You are on page 1of 108

Exam Section: Item 1 of 50 National Board of Medical Examiners

■ Mark Emergency Medicine Self-Assessment

I •
1. A 67-year-old woman comes to the emergency department 60 minutes after the sudden onset of shortness of breath. She also has had a 2-day
history of swelling of her abdomen and legs. She received the diagnosis of carcinoma of the rig ht breast 6 years ago and underwent lumpectomy
followed by radiotherapy and chemotherapy. She has type 2 diabetes mellitus well controlled with diet. On arrival, she is in severe respiratory
distress. Her pulse is 122/m in, respirations are 28/min, and blood pressure is 94/60 mm Hg. Pulse oximetry on room air shows an oxygen saturation
of 88%. Examination shows jugular venous distention. Heart sounds are distant. A pericardia! rub is heard best at the second to fourth left intercostal
spaces. The abdomen is distended and nontender. The liver is enlarged. There is 2+ edema of the lower extremities. An ECG shows sinus
tachycardia and ST-segment elevations throughout all leads. An x-ray of the chest is shown. Which of the following is the most appropriate next step
in diagnosis?
A) Bedside cardiac ultrasonograph~
B) Cardiac catheterization
C) CT scan of the chest ....

~, https://t.me/USMLENBME2CK
~ ~ r,
Next Score Report Lab Values Calculator Help Pause
Exam Section: Item 1 of 50 National Board of Medical Examiners
■ Mark Emergency Medicine Self-Assessment
...
1. A 67-year-old woman comes to the emergency department 60 minutes after the sudden onset of shortness of breath. She also has had a 2-day
history of swelling of her abdomen and legs. She received the diagnosis of carcinoma of the rig ht breast 6 years ago and underwent lumpectomy
followed by radiotherapy and chemotherapy. She has type 2 diabetes mellitus well controlled with diet. On arrival, she is in severe respiratory
distress. Her pulse is 122/m in, respirations are 28/min, and blood pressure is 94/60 mm Hg. Pulse oximetry on room air shows an oxygen saturation
of 88%. Examination shows jugular venous distention. Heart sounds are distant. A pericardia! rub is heard best at the second to fourth left intercostal
spaces. The abdomen is distended and nontender. The liver is enlarged. There is 2+ edema of the lower extremities. An ECG shows sinus
tachycardia and ST-segment elevations throughout all leads. An x-ray of the chest is shown. Which of the following is the most appropriate next step
in diagnosis?
A) Bedside cardiac ultrasonograph~
B) Cardiac catheterization
C) CT scan of the chest
D) Dopamine infusion
E) Hepari n infusion
F) Noninvasive ventilation
G) Orotracheal intubation
H) Thrombolytic infusion
Correct Answer: A.

Bedside cardiac ultrasonography is the most appropriate next step in diagnosis. This patient has a history of breast cancer and now
presents with tachycardia, jugular venous distention, pulmonary edema, and hypotension with a pericardia! friction rub , hepatomegaly,
and ST elevations on ECG. Together this is concerning for pericardia! tamponade, likely secondary to malignancy. Further assessment
with bedside ultrasonography should be the next step. Characteristic findings on echocardiography include a pericardia! effusion,
col lapse of the right atrium at the end of diastole, and a collapse of the right ventricle in the early part of diastole. Additionally, pulsus
paradoxus, which is a clinical finding defined by a drop in the systolic blood pressure of greater than 10 mm Hg with inspiration can also
be visualized on echocardiography as changes in the volume of the left and right ventricles with respiration. As a result of increased
diastolic filling pressures and equalization of diastolic pressures between the left and right heart, other potential findings include
congestive hepatopathy and tender hepatomegaly from flow reversal within the hepatic veins, and pulmonary edema.

Incorrect Answers: B, C, D, E, F, G, and H.

Cardiac catheterization (Choice B) is appropriate for patients with ST-elevation myocardial infarction (STEMI). This patient may also
have acute coronary syndrome, but an echocardiography should be performed prior to catheterization as decompression of the
pericardium via a pericardiocentesis should be performed first. ....

~, https://t.me/USMLENBME2CK
~ ~ r,
Next Score Report Lab Values Calculator Help Pause
Exam Section : Item 1 of 50 National Board of Medical Examiners
■ Mark Emergency Medicine Self-Assessment

Correct Answer: A.

Bedside cardiac ultrasonography is the most appropriate next step in diagnosis. This patient has a history of breast cancer and now
presents with tachycardia , jugular venous distention, pulmonary edema , and hypotension with a pericardia! friction rub , hepatomegaly,
and ST elevations on ECG. Together this is concerning for pericardia! tamponade, likely secondary to malignancy. Further assessment
with bedside ultrasonography should be the next step. Characteristic findings on echocardiography include a pericardia! effusion,
collapse of the right atrium at the end of diastole, and a collapse of the right ventricle in the early part of diastole. Additionally, pulsus
paradoxus, which is a clinical finding defined by a drop in the systolic blood pressure of greater than 10 mm Hg with inspiration can also
be visualized on echocardiography as changes in the volume of the left and right ventricles with respiration . As a result of increased
diastolic filling pressures and equalization of diastolic pressures between the left and right heart, other potential findings include
congestive hepatopathy and tender hepatomegaly from flow reversal within the hepatic veins, and pulmonary edema.

Incorrect Answers : B, C, D, E, F, G, and H.

Cardiac catheterization (Choice B) is appropriate for patients with ST-elevation myocardial infarction (STEMI). This patient may also
have acute coronary syndrome, but an echocardiography should be performed prior to catheterization as decompression of the
pericardium via a pericardiocentesis should be performed first.

CT scan of the chest (Choice C) is not appropriate in a borderline unstable patient with a presumed diagnosis of cardiac tamponade as
the diagnosis can easily be made at bedside with ultrasonography.

Dopamine infusion (Choice D) is appropriate in instances of cardiogenic shock, but this patient's hemodynamic status is more likely
secondary to obstruction from pericardia! tamponade, which should first be diagnosed and managed before beginning vasopressors.

Heparin infusion (Choice E) is appropriate for acute coronary syndrome, but this patient will likely require pericardiocentesis, and the
initiation of anticoagulation prior to this procedure may increase the risk for significant bleeding.

Noninvasive ventilation (Choice F) with bilevel positive pressure ventilation is sometimes used in patients with acute heart failure and
pulmonary edema. This patient's pulmonary edema is likely to improve with treatment of her pericardia! effusion.

Orotracheal intubation (Choice G) is reserved for patients who are unable to protect their airway as a result of altered mental status or
for those with respiratory failure . This patient is awake and talking and does not require intubation.

Thrombolytic infusion (Choice H) is incorrect. This patient has findings consistent with cardiac tamponade. Thrombolytics are reserved
for the treatment of STEMI in patients who do not have immediate access to a catheterization laboratory.

Educational Objective: Malignant pericardia! effusion can result in cardiac tamponade, which presents with hypotension, tachycardia ,
jugular venous distension , and pulsus paradoxus. Diagnosis can be made immediately with bedside echocardiography, which will
~, ~ F' ,.,
Next Score Report
https://t.me/USMLENBME2CK Lab Values Calculator Help Pause
Exam Section: Item 1 of 50 National Board of Medical Examiners
■ Mark Emergency Medicine Self-Assessment

Bedside cardiac ultrasonography is the most appropriate next step in diagnosis. This patient has a history of breast cancer and now
...
presents with tachycardia, jugular venous distention, pulmonary edema, and hypotension with a pericardia! friction rub, hepatomegaly,
and ST elevations on ECG. Together th is is concerning for pericardia! tamponade, likely secondary to malignancy. Fu rther assessment
with bedside ultrasonography should be the next step. Characteristic findings on echocardiography include a pericardia! effusion,
collapse of the right atrium at the end of diastole, and a collapse of the right ventricle in the early part of diastole. Additionally, pulsus
paradoxus, which is a clinical finding defined by a drop in the systolic blood pressure of greater than 10 mm Hg with inspiration can also
be visualized on echocardiography as changes in the volume of the left and right ventricles with respiration. As a result of increased
diastolic filling pressures and equalization of diastolic pressures between the left and right heart, other potential findings include
congestive hepatopathy and tender hepatomegaly from flow reversal within the hepatic veins, and pulmonary edema.

Incorrect Answers: B, C, D, E, F, G, and H.

Cardiac catheterization (Choice B) is appropriate for patients with ST-elevation myocardial infarction (STEMI). This patient may also
have acute coronary syndrome, but an echocardiography should be performed prior to catheterization as decompression of the
pericardium via a pericardiocentesis should be performed first.

CT scan of the chest (Choice C) is not appropriate in a borderline unstable patient with a presumed diagnosis of cardiac tamponade as
the diagnosis can easily be made at bedside with ultrasonography.

Dopamine infusion (Choice D) is appropriate in instances of cardiogenic shock, but this patient's hemodynamic status is more likely
secondary to obstruction from pericardia! tamponade, which should first be diagnosed and managed before beginning vasopressors.

Heparin infusion (Choice E) is appropriate for acute coronary syndrome, but this patient will likely requ ire pericardiocentesis, and the
initiation of anticoagulation prior to this procedure may increase the risk for significant bleeding.

Noninvasive ventilation (Choice F) with bilevel positive pressure ventilation is sometimes used in patients with acute heart failu re and
pulmonary edema. This patient's pulmonary edema is likely to improve with treatment of her pericardia! effusion.

Orotracheal intubation (Choice G) is reserved for patients who are unable to protect their airway as a result of altered mental status or
for those with respiratory failure. This patient is awake and talking and does not requ ire intubation.

Thrombolytic infusion (Choice H) is incorrect. This patient has findings consistent with cardiac tamponade . Thrombolytics are reserved
for the treatment of STEMI in patients who do not have immediate access to a catheterization laboratory.

Educational Objective: Malignant pericardia! effusion can result in cardiac tamponade, which presents with hypotension, tachycardia,
jugular venous distension, and pulsus paradoxus. Diagnosis can be made immediately with bedside echocardiography, which will
demonstrate right atrial and right ventricular collapse during diastole in addition to interventricular dependence and pericardia! fluid
accumulation. ....

~, https://t.me/USMLENBME2CK
~ ~ r,
Next Score Report Lab Values Calculator Help Pause
Exam Section: Item 2 of 50 National Board of Medical Examiners
■ Mark Emergency Medicine Self-Assessment

2. A 14-month-old girl is brought to the emergency department 20 minutes after having a generalized tonic-clonic seizure at home. The seizure began
after she awoke from a nap and lasted approximately 3 minutes. She was drowsy afterward. This morning, her parents noted that she had mild
rhinorrhea . She was born at term following an uncomplicated pregnancy and delivery. Development has been appropriate for age. On examination,
she is alert and responsive. Her tem perature is 39.8°C (103.6°F), pulse is 140/min, respirations are 24/min, and blood pressure is 90/60 mm Hg.
There are copious nasal secretions. The neck is supple. The lungs are clear to auscultation. Muscle strength is normal. Which of the following is the
most appropriate next step in management of this patient's seizure?

A) Reassurance
B) CT scan of the head
C) Beg in anticonvu lsant therapy
D) EEG
E) Lumbar puncture
Correct Answer: A.

This infant's generalized tonic-clonic seizure, which occurred in the setting of a febri le, upper respiratory ill ness, is consistent with a
diagnosis of simple febrile seizure. Febri le seizures typically occur in children between the ages of 6 months and 6 years and may be
simple (generalized and lasting for less than 15 minutes) or complex (focal, recurrent, or lasting longer than 15 minutes). The majority
of febrile seizures are simple, and neither require treatment nor carry a significantly increased risk for future epilepsy. A careful history
and physical examination are necessary to distinguish benign, febrile seizures from complex seizures resulting from viral or bacterial
central nervous system infections.

Incorrect Answers: B, C, D, and E.

CT scan of the head (Choice B) is useful for identifying an intracranial mass lesion or hemorrhage. This child does not display focal
neurologic signs or altered mental status to suggest an underlying intracranial pathology.

Anticonvulsant therapy (Choice C) is useful in the management of epilepsy. Simple febri le seizures are typically self-limited and do not
carry a significantly increased risk for future epilepsy.

EEG (Choice D) is useful for the diagnosis of epilepsy. Generalized tonic-clonic seizures lasting less than 15 minutes in the setting of a
febri le il lness do not require evaluation with an EEG. Recurrent seizures or those occurring outside of a period of febrile illness should
be evaluated with an EEG.

Lumbar puncture (Choice E) may be useful for the diagnosis of meningitis or encephalitis. This child lacks symptoms suggestive of ....
- - - - - - - - - - - - - - - - - - - - - -- - - - - - - -- - - - - - -- - - --- - - - - - - - -- - - -- -
r ~, ~ ~ r,
Previous Next Score Report
https://t.me/USMLENBME2CK Lab Values Calculator Help Pause
Exam Section: Item 2 of 50 National Board of Medical Examiners
■ Mark Emergency Medicine Self-Assessment
...
A) Reassurance
B) CT scan of the head
C) Beg in anticonvu lsant therapy
D) EEG
E) Lumbar pu nctu re
Correct Answer: A.

This infant's generalized tonic-clonic seizure, which occurred in the setting of a febrile, upper respiratory illness, is consistent with a
diagnosis of simple febrile seizure. Febrile seizures typically occur in children between the ages of 6 months and 6 years and may be
simple (generalized and lasting for less than 15 minutes) or complex (focal, recurrent, or lasting longer than 15 minutes). The majority
of febrile seizures are simple, and neither require treatment nor carry a significantly increased risk for future epilepsy. A careful history
and physical examination are necessary to distinguish benign, febrile seizures from complex seizures resulting from viral or bacterial
central nervous system infections.

Incorrect Answers: B, C, D, and E.

CT scan of the head (Choice B) is useful for identifying an intracranial mass lesion or hemorrhage. This child does not display focal
neurologic signs or altered mental status to suggest an underlying intracranial pathology.

Anticonvulsant therapy (Choice C) is useful in the management of epilepsy. Simple febrile seizures are typically self-limited and do not
carry a significantly increased risk for future epilepsy.

EEG (Choice D) is useful for the diagnosis of epilepsy. Generalized tonic-clonic seizures lasting less than 15 minutes in the setting of a
febrile illness do not require evaluation with an EEG. Recurrent seizures or those occurring outside of a period of febrile illness should
be evaluated with an EEG.

Lumbar puncture (Choice E) may be useful for the diagnosis of meningitis or encephalitis. This child lacks symptoms suggestive of
these diagnoses, such as altered mental status, meningismus, or focal neurologic symptoms.

Educational Objective: Febrile seizures typically occur in children between the ages of 6 months and 5 years in the setting of a febrile
illness and may be simple (generalized and lasting for less than 15 minutes) or complex (focal, recurrent, or lasting longer than 15
minutes). Simple febrile seizures do not require treatment or invasive testing.

.,.

r ~, ~ ~ r,
Previous Next Score Report
https://t.me/USMLENBME2CK Lab Values Calculator Help Pause
Exam Section: Item 3-4 of 50 National Board of Medical Examiners
■ Mark Emergency Medicine Self-Assessment

The following vignette applies to the next 2 items. The items in the set must be answered in sequential order. Once you click Proceed to Next Item, you will not
be able to add or change an answer.

A 71-year-old woman is broug ht to the emergency department by her fami ly because of a 1-day history of intermittent ag itation and confusion. Du ring the past 2
days, she has had a cough and has felt cold. She has hypertension, coronary artery disease, and Graves disease; she takes lisinopril, methimazole, and
lovastatin. She has no known allergies. On arrival , she is oriented to person but not to place or time. Her temperatu re is 39.4 °C (103°F), pulse is 150/min and
regular, respi rations are 26/min, and blood pressu re is 114/52 mm Hg. Pulse oximetry on room ai r shows an oxygen satu ration of 92%. The skin is warm and
moist to the touch. Card iac exam ination shows no murmur or gallop. Rhonchi are heard in the right lung base. The abdomen is soft and nontender. Cranial
nerves, motor function, sensation, and cerebellar function appear to be intact, but patient ag itation prevents a detailed neurolog ic exami nation. Patellar reflexes
are 3+ bilaterally.

'I 3. Item 1 of 2
In add ition to x-ray of the chest, which of the following is most likely to confi rm the diag nosis?

A) Blood cultu re
B) Complete blood count with differential
C) CT scan of the head
D) Measurement of serum calcium and magnesium concentrations
E) Thyroid fun ction testing
Correct Answer: E.

Thyroid storm is a serious potential complication of hyperthyroidism. It typically occurs in a patient with a known diagnosis of
hyperthyroidism following physiologic stress from surgery or trauma, or in the setting of infection, such as in this patient who presents
with signs and symptoms of pneumonia. Signs and symptoms of thyroid storm include hyperthermia, tachycardia, tachypnea,
hypertension, diaphoresis, agitation, tremor, hyperreflexia, nausea, and diarrhea, and may progress to high-output heart failure,
obtundation, seizure, or coma. The initial diagnosis of thyroid storm is made through a combination of clinical features and thyroid
function testing, which shows increased serum concentrations of triiodothyronine (T 3) and thyroxine (T 4), and a decreased
concentration of thyroid-stimulating hormone (TSH). Management requires supportive care, treatment of the underlying provocative
stress, as well as intravenous corticosteroids, ~-adrenergic blockers, propylthiouracil, and potassium iodide.

Incorrect Answers: A, B, C, and D.

Blood culture (Choice A) is an appropriate step for the diagnosis of bacteremia and sepsis. This patient's altered mental status,
hyperreflexia, and known history of Graves disease is more concerning for thyroid storm. ....

r ~, ~ ~ r,
Previous Next Score Report
https://t.me/USMLENBME2CK Lab Values Calculator Help Pause
Exam Section: Item 3-4 of 50 National Board of Medical Examiners
■ Mark Emergency Medicine Self-Assessment
--.. . . ...
B) Complete blood count with differential
C) CT scan of the head
D) Measurement of serum calcium and magnesium concentrations
E) Thyroid function testing
Correct Answer: E.

Thyroid storm is a serious potential complication of hyperthyroidism. It typically occurs in a patient with a known diagnosis of
hyperthyroidism following physiologic stress from surgery or trauma, or in the setting of infection, such as in this patient who presents
with signs and symptoms of pneumonia. Signs and symptoms of thyroid storm include hyperthermia, tachycardia, tachypnea,
hypertension, diaphoresis, agitation, tremor, hyperreflexia, nausea, and diarrhea, and may progress to high-output heart failure,
obtundation, seizure, or coma. The initial diagnosis of thyroid storm is made through a combination of clinical features and thyroid
function testing, which shows increased serum concentrations of triiodothyronine (T 3) and thyroxine (T 4), and a decreased
concentration of thyroid-stimulating hormone (TSH). Management requires supportive care, treatment of the underlying provocative
stress, as well as intravenous corticosteroids, ~-adrenergic blockers, propylthiouracil, and potassium iodide.

Incorrect Answers: A, B, C, and D.

Blood culture (Choice A) is an appropriate step for the diagnosis of bacteremia and sepsis. This patient's altered mental status,
hyperreflexia, and known history of Graves disease is more concerning for thyroid storm.

Complete blood count with differential (Choice B) may disclose neutrophilia in the setting of pneumonia or leukocytosis in the setting of
thyroid storm. These findings are relatively nonspecific and are less useful for the diagnosis in comparison to thyroid function testing.

CT scan of the head (Choice C) is useful for excluding the presence of intracranial mass lesions prior to performing a lumbar puncture
in patients with meningitis. Meningitis presents with fever, headache, photophobia, phonophobia, and neck stiffness, with possible
Kernig or Brudzinski signs on examination.

Measurement of serum calcium and magnesium concentrations (Choice D) may be useful for assessing potential causes of
hyperreflexia. However, this patient's vital signs and her known history of Graves disease suggest thyroid storm rather than electrolyte
derangement as the most likely cause of her hyperreflexia.

Educational Objective: Thyroid storm is a serious potential complication of hyperthyroidism. It typically occurs in patients with a known
diagnosis of hyperthyroidism fol lowing physiologic stress, such as infection (eg, pneumonia), surgery, or trauma. Thyroid storm
presents with vital sign instability, mental status changes including agitation, obtundation, or coma, tremor, seizures, hyperreflexia,
diaphoresis, nausea, diarrhea, and potential high-output heart failure.
....

r ~, ~ ~ r,
Previous Next Score Report
https://t.me/USMLENBME2CK Lab Values Calculator Help Pause
Exam Section: Item 3-4 of 50 National Board of Medical Examiners
■ Mark Emergency Medicine Self-Assessment
...

4. Item 2 of 2
Acetaminophen and 0.9% saline (2 L) are administered , and supplemental oxygen via nasal cannu la is initiated. One hour later, her temperature is
38.2°C (100.8°F), pulse is 145/min, respirations are 22/min, and blood pressure is 135/65 mm Hg. Oxygen saturation is 97%. Laboratory stud ies
show:
Leukocyte count 17,000/mm 3(17 x 10 9/L)
Segmented neutrophils 85%
Bands 3%
Lymphocytes 10%
Monocytes 2%
Serum
Na + 145 mEq/L (145 mmol/L)
Creatin ine 1.3 mg/dL (114.9 µmol/L)
Thyroid -stim ulating hormone 0.01 µIU/mL (0.01 mlU/L)
Lactic acid 2.4 mEq/L (2.4 mmol/L) [N<2 mEq/L//<2 mmol/L]
....

r ~, ~ ~ r,
Previous Next Score Report
https://t.me/USMLENBME2CK Lab Values Calculator Help Pause
4. Item 2 of 2
Acetaminophen and 0.9% saline (2 L) are administered , and supplemental oxygen via nasal cannula is initiated. One hour later, her temperature is
38.2°C (100.8°F), pulse is 145/min, respirations are 22/min, and blood pressure is 135/65 mm Hg. Oxygen saturation is 97%. Laboratory studies
show:
Leukocyte count 17,000/mm 3(17 x 10 9/L)
Segmented neutrophils 85%
Bands 3%
Lymphocytes 10%
Monocytes 2%
Serum
Na + 145 mEq/L (145 mmol/L)
Creatinine 1.3 mg/dL (114.9 µmol/L)
Thyroid -stim ulating hormone 0.01 µIU/mL (0.01 mlU/L)
Lactic acid 2.4 mEq/L (2.4 mmol/L) [N<2 mEq/L//<2 mmol/L]

X-rays of the chest are shown. In addition to antibiotic therapy, the most appropriate next step in management is administration of which of the
followi ng?

A) Adenosine
B) Diazepam
C) Diltiazem
D) Methimazole
E) Propranolol ....

r ~, ~ ~ r,
Previous Next Score Report
https://t.me/USMLENBME2CK Lab Values Calculator Help Pause
Exam Section: Item 3-4 of 50 National Board of Medical Examiners
■ Mark Emergency Medicine Self-Assessment
...
D) Methimazole
E) Propranolol
Correct Answer: E.

Thyroid storm can be urgently life-threatening by causing tachyarrhythmias, myocardial infarction, high-output heart failure, and/or
circulatory collapse. Rapid treatment of vital sign instability is of the utmost importance and includes treatment with ~-adrenergic
blockers. ~-Adrenergic blockers are useful for the management of tachycardia and high-output heart failure, and have inherent
antithyroid effects. Hyperthermia and hypotension are treated with antipyretics and cooling, and with fluid resuscitation, respectively.
Patients with thyroid storm require intensive support, typically in an intensive care unit. Once vital signs are stable, further treatment
typically requires antithyroid medications, including propylthiouracil.

Incorrect Answers: A, B, C, and D.

Adenosine (Choice A) is a purine analogue with atrioventricular nodal blockade effects that is useful for the treatment of
supraventricular tachycardia. It plays little role in the management of thyroid storm outside of the concomitant presence of a reentrant
atrioventricular tachydysrhythmia.

Diazepam (Choice B) is a benzodiazepine that may be useful for managing agitation or seizures in patients with thyroid storm. While
management of agitation is an important element of this patient's care, stabilization of vital signs is of greater immediate clinical
importance.

Diltiazem (Choice C) is a nondihydropyridine calcium channel blocker that may be used to manage tachyarrhythmia in thyroid storm for
patients with a contraindication to ~-adrenergic blockers (eg, severe asthma or ch ronic obstructive pulmonary disease), but it is not a
first-line treatment.

Methimazole (Choice D) is an antithyroid medication that inhibits thyroid peroxidase. Propylthiouracil is typically preferred over
methimazole in the management of thyroid storm because it inhibits thyroid peroxidase as does methimazole, but propylthiouracil has
the additional effect of limiting the peripheral conversion between thyroxine (T 4) and triiodothyronine (T 3).

Educational Objective: Thyroid storm can be life-threatening by causing tachyarrhythmias, myocardial infarction, high-output heart
failure, or circulatory collapse. Rapid treatment of vital sign instability is of the utmost importance and includes treatment with ~-
adrenergic blockers. ~-Adrenergic blockers are useful for the management of tachycardia and high-output heart failure, and have
inherent antithyroid effects.

End of Set
.,.

r ~, ~ ~ r,
Previous Next Score Report
https://t.me/USMLENBME2CK Lab Values Calculator Help Pause
Exam Section: Item 5 of 50 National Board of Medical Examiners
■ Mark Emergency Medicine Self-Assessment

5. An 18-year-old man is brought to the emergency department because of a 6-hour history of fever, severe headache, and neck and back pain. He
appears acutely ill and lethargic. His temperature is 40°C (104 °F). When he attempts to flex his neck, he has severe neck pain and automatically
flexes his th ighs. Examination shows small petech ial hemorrhages over his abdomen and thig hs. He is oriented to person but not to place or time.
Cerebrospinal fluid analysis in this patient is most likely to show which of the following sets of findings?

Segmented
Leukocyte Count Neutrophils Monocytes Protein Glucose
Appearance (/mm 3) (%) (%) (mg/dl) (mg/dl)
A) Clear 10 (0.01 x 10 9/L) 100 0 220 (2.2 g/L) 80 (4.44 mmol/L)
B) Clear 28 (0.03 x 10 9/L) 100 0 180 (1.8 g/L) 75 (4.16 mmol/L)
C) Clear 420 (0.42 x 109/L) 97 3 130 (1.3 g/L) 80 (4.44 mmol/L)
D) Cloudy 250 (0.25 x 109/L) 10 90 220 (2.2 g/L) 20 (1.11 mmol/L)
E) Cloudy 2600 (2.6 x 109/L) 70 30 258 (2.58 g/L) 10 (0.56 mmol/L)
Correct Answer: E.

This patient demonstrates findings of acute meningococcal meningitis including fever, severe headache, neck stiffness, and a petechial rash.
This patient also demonstrates Brudzinski sign, in which flexion of the neck induces hip flexion. Meningococcal meningitis is caused by
infection with Neisseria meningitidis, the most common cause of bacterial meningitis in teenagers and young adults. Clusters of infections may
be seen among young adults sharing close living quarters such as college dormitories or military barracks. The cerebrospinal fluid (CSF)
analysis of bacterial meningitis demonstrates increased leukocytes with a neutrophil predominance, increased protein, and decreased glucose
concentration. The appearance of the fluid is often cloudy and additional studies such as gram stain and bacterial culture will often be positive.
Thus, the CSF findings in this patient are most likely to show cloudy fluid with a leukocyte count of 2600/mm 3 with 70% segmented
neutrophils, increased protein (258 mg/dl), and decreased glucose (10 mg/dl).

Incorrect Answers: A, B, C, and D.

Clear fluid with a low leukocyte count, whether 10/mm 3 (Choice A) or 28/mm 3 (Choice B), is not generally seen in bacterial meningitis.

Clear fluid with a normal glucose concentration (Choice C) does not represent the typical findings seen in bacterial meningitis. In bacterial
meningitis, the neutrophils recruited to the CSF in response to the infection utilize glucose to perform their metabolic activities, resulting in a
decreased glucose concentration within the CSF.

Cloudy fluid with 250/mm 3 leukocytes, 90% monocytes, increased protein, and decreased glucose (Choice D) is consistent with a viral, fungal,
or mycobacterial meningitis. In contrast, bacterial meningitis will have a predominance of neutrophils.
....

r ~, ~ ~ r,
Previous Next Score Report
https://t.me/USMLENBME2CK Lab Values Calculator Help Pause
Exam Section: Item 5 of 50 National Board of Medical Examiners
■ Mark Emergency Medicine Self-Assessment
- i -- - . - i

Leukocyte Count Neutrophils Monocytes Protein Glucose


Appearance (/mm 3) (%) (%) (mg/dl) (mg/dl)
A) Clear 10 (0.01 x 10 9/L) 100 0 220 (2.2 g/L) 80 (4.44 mmol/L)
B) Clear 28 (0.03 x 10 9/L) 100 0 180 (1.8 g/L) 75 (4.16 mmol/L)
C) Clear 420 (0.42 x 109/L) 97 3 130 (1.3 g/L) 80 (4.44 mmol/L)
D) Cloudy 250 (0.25 x 109/L) 10 90 220 (2.2 g/L) 20 (1.11 mmol/L)
E) Cloudy 2600 (2.6 x 109/L) 70 30 258 (2.58 g/L) 10 (0.56 mmol/L)
Correct Answer: E.

This patient demonstrates findings of acute meningococcal meningitis including fever, severe headache, neck stiffness, and a petechial ra sh.
This patient also demonstrates Brudzinski sign, in which flexion of the neck induces hip flexion. Meningococcal meningitis is caused by
infection with Neisseria meningitidis, the most common cause of bacterial meningitis in teenagers and young adults. Clusters of infections may
be seen among young adults sharing close living quarters such as college dormitories or military barracks. The cerebrospinal fluid (CSF)
analysis of bacterial meningitis demonstrates increased leukocytes with a neutrophil predominance, increased protein, and decreased glucose
concentration. The appearance of the fluid is often cloudy and additional studies such as gram stain and bacterial culture will often be positive.
Thus, the CSF findings in this patient are most likely to show cloudy fluid with a leukocyte count of 2600/mm 3 with 70% segmented
neutrophils, increased protein (258 mg/dL), and decreased glucose (10 mg/dL).

Incorrect Answers: A, B, C, and D.

Clear fluid with a low leukocyte count, whether 10/mm 3 (Choice A) or 28/mm 3 (Choice B), is not generally seen in bacterial meningitis.

Clear fluid with a normal glucose concentration (Choice C) does not represent the typical findings seen in bacterial meningitis. In bacterial
meningitis, the neutrophils recruited to the CSF in response to the infection utilize glucose to perform their metabolic activities, resulting in a
decreased glucose concentration within the CSF.

Cloudy fluid with 250/mm 3 leukocytes, 90% monocytes, increased protein, and decreased glucose (Choice D) is consistent with a viral, fungal,
or mycobacterial meningitis. In contrast, bacterial meningitis will have a predominance of neutrophils.

Educational Objective: Cerebrospinal fluid analysis of bacterial meningitis typically demonstrates increased leukocytes with a neutrophil
predominance, increased protein, and decreased glucose concentration. The appearance of the fluid is often cloudy and additional studies
such as gram stain and bacterial culture will often be positive.

.,.

r ~, ~ ~ r,
Previous Next Score Report
https://t.me/USMLENBME2CK Lab Values Calculator Help Pause
Exam Section: Item 6 of 50 National Board of Medical Examiners
■ Mark Emergency Medicine Self-Assessment

6. A 42-year-old woman comes to the emergency department because of a 2-day history of moderate, intermittent right upper abdominal pain that
rad iates to her right shoulder blade. The pain typically occurs 30 minutes after eating and is associated with nausea and vomiting. She says she has
not eaten much today but continues to have pain. She also had a tem perature of 38.3°C (101 °F) earlier. She has no history of similar symptoms.
She has mild asthma treated with an albuterol inhaler as needed . She has no history of operative procedures and has no known drug allerg ies. Her
current temperature is 38.1 °C (100.5°F), pulse is 92/min, respirations are 16/min, and blood pressure is 136/72 mm Hg. Pulse oximetry on room air
shows an oxygen saturation of 100%. The abdomen is soft; there is tenderness to palpation of the right upper quadrant. The patient ceases
inspiration during palpation of the right upper quadrant. Laboratory studies show:
Hematocrit 38%
Leukocyte count 13,000/mm 3 (13 x 109/L)
Platelet count 284,000/mm 3 (284 x 10 9/L)
Serum
Na + 134 mEq/L (134 mmol/L)
K+ 3.5 mEq/L (3.5 mmol/L)
HCO 3- 21 mEq/L (21 mmol/L)
Bilirubin, total 2.8 mg/dL (47.88 µmol/L)
Direct 1.2 mg/dL (20.5 µmol/L)
AST 243 U/L
ALT 187 U/L
Li pase 62 U/L [N =14-280]

Urinalysis shows no abnormalities. Which of the following is the most appropriate next step in management?

A) Endoscopy and colonoscopy


B) Ketorolac therapy
C) Methylprednisolone therapy
D) Piperacillin-tazobactam therapYi
E) Placement of a nasogastric tube
Correct Answer: D.

Acute cholecystitis occurs secondary to obstruction of the cystic duct from a gallstone, which results in subsequent inflammation of the
gallbladder wall. Risk factors for developing gallstones include female gender, multiparity, increasing age, and obesity. High rates of
gallstone formation are also seen after periods of rapid weight loss, though the mechanism by which this occurs is not fully understood.

r

~,
. ....
•- ' - - . - •• .. - . - ·- • - . •• - . - • - . ·- .... ·- .. -
- -
~ ~
-• -
r,
..
Acute cholecystitis typically presents with fever, abdominal pain, and tenderness to palpation in the right upper quadrant, often in a
-· •• - ·-
Previous Next Score Report
https://t.me/USMLENBME2CK Lab Values Calculator Help Pause
Exam Section: Item 6 of 50 National Board of Medical Examiners
■ Mark Emergency Medicine Self-Assessment
...
Correct Answer: D.

Acute cholecystitis occurs secondary to obstruction of the cystic duct from a gallstone, which results in subsequent inflammation of the
gallbladder wall. Risk factors for developing gallstones include female gender, multiparity, increasing age, and obesity. High rates of
gallstone formation are also seen after periods of rapid weight loss, though the mechanism by which this occurs is not fully understood.
Acute cholecystitis typically presents with fever, abdominal pain, and tenderness to palpation in the right upper quadrant, often in a
patient with a history of biliary colic and cholelithiasis. The pain may also radiate to the right shoulder or interscapular region. It may be
distinguished from biliary colic by its constant rather than intermittent nature and associated findings of fever and leukocytosis.
Abdominal ultrasonography is the first-line diagnostic examination for the evaluation of potential acute cholecystitis. Characteristic
ultrasonography findings of acute cholecystitis include gallbladder wall thickening and hyperemia, pericholecystic fluid, presence of
gallstones, a dilated cystic duct or common bile duct, and a sonographic Murphy sign (pain with compression directly over the
gallbladder). Treatment includes supportive therapy, intravenous fluids, intravenous antibiotics, and urgent surgical resection of the
gallbladder (cholecystectomy). Intravenous antibiotic therapy should cover Gram-negative rods and anaerobes, making piperacillin-
tazobactam therapy the most appropriate next step.

Incorrect Answers: A, B, C, and E.

Endoscopy and colonoscopy (Choice A) are used to evaluate gastrointestinal bleeding, which can present with fatigue, hematemesis,
hematochezia, and melena. Endoscopic retrograde cholangiopancreatography can be performed if there is concern for
choledocholithiasis, but antibiotics are a more appropriate initial next step.

Ketorolac therapy (Choice B) may be useful in managing this patient's pain. However, it is more pertinent to start antibiotic therapy as
soon as possible to prevent complications of acute cholecystitis, such as abscess formation or bacteremia.

Methylprednisolone therapy (Choice C) is used in the treatment of allergic, autoimmune, and inflammatory conditions, such as
autoimmune hepatitis, Crohn disease, or ulcerative colitis flares. This patient has localized right upper quadrant pain, associated with
fever, leukocytosis, and mildly increased bilirubin and hepatic transaminases, making acute cholecystitis the more likely diagnosis.

Placement of a nasogastric tube (Choice E) is utilized in small bowel obstruction, which presents with nausea, emesis, abdominal pain,
distension, and constipation or obstipation, most commonly in a patient with a history of previous abdominal surgery. While this patient
is having nausea and emesis, her pain is localized to her right upper quadrant, making acute cholecystitis more likely.

Educational Objective: Acute cholecystitis occurs secondary to cystic duct obstruction by a gallstone. It presents with fever and right
upper quadrant abdominal pain, in addition to leukocytosis, and potential mild hyperbilirubinemia and transaminitis. Treatment includes
supportive therapy, intravenous fluids, antibiotics, and urgent surgical resection. Antibiotic therapy should include coverage of Gram-
negative rods and anaerobes, such as that provided by piperacillin-tazobactam therapy.

....

r ~, ~ ~ r,
Previous Next Score Report
https://t.me/USMLENBME2CK Lab Values Calculator Help Pause
Exam Section : Item 7 of 50 National Board of Medical Examiners
■ Mark Emergency Medicine Self-Assessment
....

7. A 78-year-old woman is brought to the emergency department by paramedics because of a 1-day history of generalized weakness and confusion .
Medical history includes mild dementia, hypertension, and osteoporosis. Medications are alendronate, atenolol, and donepezil. On arrival, the
patient appears ill. She is oriented to person but not to place or time. Her temperature is 35.4 °C (95.7°F), pulse is 128/min, respirations are 22/min,
and blood pressure is 76/50 mm Hg. The remainder of the physical examination shows no abnormalities. Urinalysis shows:
Leukocyte esterase 2+
WBC 50-100/hpf
RBC 10-2Wh~

A central venous catheter is inserted while the patient is lying supine. Ceftriaxone and 0.9% saline (2 L) are administered during the first hour. After
fluid resuscitation , central venous pressure is 2 mm Hg (N=3-8). Which of the following is the most likely explanation for this finding?

A) Anaphylaxis from ceftriaxone


B) Dehydration
C) Failure to place the patient in Trendelenburg position
D) Improper placement of the central venous catheter
E) Vasodilation caused by septic shock
Correct Answer: E.

This patient demonstrates persistently decreased central venous pressure despite fluid resuscitation , which is suggestive of
vasodilation caused by septic shock. The Third International Consensus Definitions for Sepsis and Septic Shock defined septic shock
by a vasopressor requirement to maintain mean arterial pressure greater than 65 mm Hg or a serum lactate concentration greater than
2 mmol/L despite fluid resuscitation. This patient presents with findings concerning for a urinary tract infection as a source, with a
urinalysis demonstrating increased WBCs and leukocyte esterase. This is the likely cause of the patient's probable bacteremia and
sepsis, which predisposed her to developing septic shock. Septic shock is characterized by profound circulatory dysregulation causing
abnormal peripheral vasodilation , which results in decreased systemic vascular resistance, decreased mean arterial pressure, and
consequent organ hypoperfusion and dysfunction.

Incorrect Answers: A, B, C, and D.

Anaphylaxis from ceftriaxone (Choice A) is typically accompanied by respiratory and cutaneous findings such as wheezing, flushing, or
urticaria.

Dehydration (Choice B) is unlikely in this patient whose central venous pressure did not respond to 2 L of intravenous 0.9% saline.
!!- - • • • • • -· ••• - • - -· ' •• • - ..... - • - · - - • ··- - - -· •• - •••• -· •• . - ·- - • ·-
,. ~, ~ P r,
Previous Next Score Report
https://t.me/USMLENBME2CK Lab Values Calculator Help Pause
Exam Section: Item 7 of 50 National Board of Medical Examiners
■ Mark Emergency Medicine Self-Assessment
...
C) Failure to place the patient in Trendelenbu rg position
D) Improper placement of the central venous catheter
E) Vasodilation caused by septic shock
Correct Answer: E.

This patient demonstrates persistently decreased central venous pressure despite fluid resuscitation, which is suggestive of
vasodilation caused by septic shock. The Third International Consensus Definitions for Sepsis and Septic Shock defined septic shock
by a vasopressor requirement to maintain mean arterial pressure greater than 65 mm Hg or a serum lactate concentration greater than
2 mmol/L despite fluid resuscitation. This patient presents with findings concerning for a urinary tract infection as a source, with a
urinalysis demonstrating increased WBCs and leukocyte esterase. This is the likely cause of the patient's probable bacteremia and
sepsis, which predisposed her to developing septic shock. Septic shock is characterized by profound circulatory dysregulation causing
abnormal peripheral vasodilation, which results in decreased systemic vascular resistance, decreased mean arterial pressure, and
consequent organ hypoperfusion and dysfunction.

Incorrect Answers: A, B, C, and D.

Anaphylaxis from ceftriaxone (Choice A) is typically accompanied by respiratory and cutaneous findings such as wheezing, flushing, or
urticaria.

Dehydration (Choice B) is unlikely in this patient whose central venous pressure did not respond to 2 L of intravenous 0.9% saline.
Refractory hypotension in a patient who has been adequately volume resuscitated is concerning for septic shock, especially in the
setting of other signs and symptoms that are suggestive of an infection and sepsis, such as fever or hypothermia (as in this case),
tachycardia, tachypnea, and pyuria.

Failure to place the patient in Trendelenburg position (Choice C) may increase the difficulty of placing a central venous catheter in the
jugular or subclavian veins. However, this positioning may not be possible in very ill patients and is not mandatory for the treatment of
shock. Positioning may skew measurement of central venous pressure, though no abnormality in positioning is suggested in this case.

Improper placement of the central venous catheter (Choice D) is unlikely given the successfu l administration of antibiotics and fluids.
This patient's refractory hypotension is more concerning for septic shock than for a misplaced central line.

Educational Objective: Septic shock is characterized by profound circulatory dysregulation causing abnormal peripheral vasodilation,
which results in decreased systemic vascu lar resistance, decreased mean arterial pressure, and consequent organ hypoperfusion and
dysfunction. Septic shock should be suspected in patients with refractory hypotension despite adequate fluid resuscitation.

.,.

r ~, ~ ~ r,
Previous Next Score Report
https://t.me/USMLENBME2CK Lab Values Calculator Help Pause
Exam Section: Item 8 of 50 National Board of Medical Examiners
■ Mark Emergency Medicine Self-Assessment

8. A 66-year-old woman is brought to the emergency department (ED) from the skilled nursing facility because of a 1-day history of shortness of
breath. Staff noted that her breathing has been more labored since yesterday; earlier today, pulse oximetry on room air done as part of routine vital
sign assessment showed an oxygen saturation of 88%. She has morbid obesity, hypertension, type 2 diabetes mellitus, coronary artery disease,
obstructive sleep apnea , and dementia. Medications are metoprolol, amlodipine, isosorbide, glyburide, metformin, aspirin, and donepezil. She has
no known allergies. BMI is 34 kg/m2 • She is somnolent but arousable and states her correct name. Her temperature is 36.2°C (97.2°F), pulse is
108/min, respirations are 24/min , and blood pressu re is 154/86 mm Hg. Pulse oximetry on 6 L/min supplemental oxygen via nasal cannula shows
an oxygen saturation of 94%. Auscultation of the lungs discloses diffuse diminished breath sounds with shallow respirations. Cardiac examination
discloses tachycardic and regular rhythm with normal heart sounds. Bilevel positive pressure ventilation is begun shortly after arrival to the ED.
Twenty minutes later, vital signs and physical examination findings are unchanged. Arterial blood gas analysis shows:
pH 7.18
Pco 2 94 mm Hg (12.5 kPa)
Po 2 66 mm Hg (8.78 kPa)
HCO 3- 36 mEq/L (36 mmol/L)

Chest x-ray shows hypoinflation. Which of the following is the most appropriate next step in treatment?

A) Adjust bilevel positive pressure ventilation settings


B) Administer albuterol
C) Administer helium and oxygen
D) Administer methylprednisolone
E) Intubate the patient
Correct Answer: E.

This patient is presenting with shortness of breath and shallow respirations with associated hypercarbic respiratory failure as seen on
arterial blood gas (ABG) analysis. This patient likely has underlying obstructive lung disease secondary to obstructive sleep apnea or
obesity hypoventilation syndrome, as evidenced by an increased HCO 3- indicating a compensatory metabolic alkalosis in the setting of
respiratory acidosis. Nevertheless, and regardless of the reason for the derangement, this patient's hypercarbia is severe, acute on
chronic, and causing persistent respiratory acidosis as evidenced by a decreased pH in spite of noninvasive and appropriate corrective
measures. Hypercarbia is the most likely underlying cause of the patient's decreased level of consciousness and is likely compounding
baseline deficits from dementia. Noninvasive positive pressure ventilation is often the first step in the management of hypercarbic
respiratory failure, but if it is insufficient, as in this case with continued altered mental status and severe acidosis, endotracheal
intubation to protect the airway and facilitate invasive positive pressure ventilation is necessary.

• • • !a • • ! : - •• •
r ~, ~ ~ r,
Previous Next Score Report
https://t.me/USMLENBME2CK Lab Values Calculator Help Pause
Exam Section : Item 8 of 50 National Board of Medical Examiners
■ Mark Emergency Medicine Self-Assessment

Chest x-ray shows hypoinflation . Which of the following is the most appropriate next step in treatment?

A) Adjust bilevel positive pressure ventilation settings


B) Administer albuterol
C) Administer helium and oxygen
D) Administer methylprednisolone
E) Intubate the patient
-- Correct Answer: E.

This patient is presenting with shortness of breath and shallow respirations with associated hypercarbic respiratory failure as seen on
arterial blood gas (ABG) analysis. This patient likely has underlying obstructive lung disease secondary to obstructive sleep apnea or
obesity hypoventilation syndrome , as evidenced by an increased HC0 3- indicating a compensatory metabolic alkalosis in the setting of
respiratory acidosis. Nevertheless, and regardless of the reason for the derangement, this patient's hypercarbia is severe , acute on
chronic, and causing persistent respiratory acidosis as evidenced by a decreased pH in spite of noninvasive and appropriate corrective
measures. Hypercarbia is the most likely underlying cause of the patient's decreased level of consciousness and is likely compounding
baseline deficits from dementia. Noninvasive positive pressure ventilation is often the first step in the management of hypercarbic
respiratory failure, but if it is insufficient, as in this case with continued altered mental status and severe acidosis, endotracheal
intubation to protect the airway and facilitate invasive positive pressure ventilation is necessary.

Incorrect Answers : A , B, C, and D.

Adjusting bilevel positive pressure ventilation settings (Choice A) would be appropriate if the patient showed a positive response to
initial treatment or if initial settings were incorrect or inadequate. However, the patient's mental status remains unchanged, making
intubation a more appropriate next step.

Administering albuterol, helium, oxygen , or methylprednisolone (Choices B, C, and D) can be helpful in the setting of a chronic
obstructive pulmonary disease exacerbation and may be helpful in this patient with hypercarbic respiratory failure . However, medical
management should follow endotracheal intubation, which is the most appropriate next step.

Educational Objective: Hypercarbic respiratory failure can present with shortness of breath , shallow respirations, and altered mental
status. If noninvasive positive pressure ventilation does not improve the patient's respiratory status, endotracheal intubation is the most
appropriate next step.

...
r ~, ~ p r,
Previous Next Score Report
https://t.me/USMLENBME2CK Lab Values Calculator Help Pause
Exam Section: Item 9 of 50 National Board of Medical Examiners
■ Mark Emergency Medicine Self-Assessment

9. A 53-year-old woman comes to the emergency department because of two episodes of dark brown emesis during the past 4 hours . She has taken
ibuprofen daily during the past 2 weeks because of chronic, intermittent low back pain. She has no history of serious illness and does not take any
other medications. She has no known allergies. She drinks two to th ree glasses of wine daily. She has smoked one pack of cigarettes daily for the
past 30 years. Her temperatu re is 36.6°C (97.9°F), pulse is 116/min, respirations are 18/min, and blood pressure is 92/54 mm Hg. Pulse oximetry on
room air shows an oxygen saturation of 97%. Lungs are clear to auscultation. Abdomen is soft, and there is mild tenderness to palpation over the
epigastric region. Which of the following is the most appropriate next step in management?

A) Administration of omeprazole
B) Administration of 0.9% saline
C) Insertion of a nasogastric tube
D) Tagged red blood cell scan
E) Upperendoscopy
Correct Answer: B.

Gastritis classically presents with left upper quadrant or epigastric pain, nausea, and vomiting. If erosive gastritis or an ulcer is present,
emesis may be blood-streaked, frankly bloody, or coffee-ground in character. If bleeding has been present for several hours, stool may
be melanotic. Gastritis common ly occurs from alcohol abuse, excess use of nonsteroidal anti-inflammatory drugs (NSAIDs), steroids,
infections, or smoking. Other causes of upper gastrointestinal (GI) bleeding include esophagitis, esophageal varices, peptic ulcer
disease, and arteriovenous malformations. Regardless of the cause of bleeding, any patient presenting with tachycardia, dry mucous
membranes, and signs of hemorrhagic shock (such as this patient's borderline hypotension) shou ld receive immediate intravenous fluid
resuscitation, typically with isotonic crystalloids such as 0.9% saline. Massive hematemesis, persistent hemodynamic instability, or
concern for active bleeding should prompt emergent blood transfusion.

Incorrect Answers: A, C, D, and E.

Omeprazole (Choice A), a proton pump inhibitor, is the first-line pharmacotherapy for patients presenting with gastroesophageal reflux
(GERO), reflux esophagitis, and peptic ulcer disease. Whi le it may be helpful in the treatment of this patient, the first step in
management of a patient who is hypotensive and tachycardic is fluid resuscitation.

Insertion of a nasogastric tube (Choice C) has a role in gastric decompression in cases of smal l bowel obstruction but does not have a
primary role in the management of undifferentiated upper gastrointestinal bleeding. It is arguably contraindicated in cases of
esophageal varices, as placement of the tube may exacerbate a variceal bleed by disrupting a thrombus or worsening the culprit
bleeding lesion.

r
.
~,
----- -------- ---- ---- • ----- --- - -- - ------- - - --- ---- -- ------
~ ~
-- - -- -
r,
--- --
....

Previous Next Score Report


https://t.me/USMLENBME2CK Lab Values Calculator Help Pause
Exam Section: Item 9 of 50 National Board of Medical Examiners
■ Mark Emergency Medicine Self-Assessment

E) Upperendoscopy
...
Correct Answer: B.

Gastritis classically presents with left upper quadrant or epigastric pain, nausea, and vomiting. If erosive gastritis or an ulcer is present,
emesis may be blood-streaked, frankly bloody, or coffee-ground in character. If bleeding has been present for several hours, stool may
be melanotic. Gastritis common ly occurs from alcohol abuse, excess use of nonsteroidal anti-inflammatory drugs (NSAIDs), steroid s,
infections, or smoking. Other causes of upper gastrointestinal (GI) bleeding include esophagitis, esophageal varices, peptic ulcer
disease, and arteriovenous malformations. Regardless of the cause of bleeding, any patient presenting with tachycardia, dry mucous
membranes, and signs of hemorrhagic shock (such as this patient's borderline hypotension) should receive immediate intravenous fluid
resuscitation, typically with isotonic crystalloids such as 0.9% saline. Massive hematemesis, persistent hemodynamic instability, or
concern for active bleeding should prompt emergent blood transfusion.

Incorrect Answers: A, C, D, and E.

Omeprazole (Choice A), a proton pump inhibitor, is the first-line pharmacotherapy for patients presenting with gastroesophageal reflux
(GERO), reflux esophagitis, and peptic ulcer disease. While it may be helpful in the treatment of this patient, the first step in
management of a patient who is hypotensive and tachycardic is fluid resuscitation.

Insertion of a nasogastric tube (Choice C) has a role in gastric decompression in cases of small bowel obstruction but does not have a
primary role in the management of undifferentiated upper gastrointestinal bleeding. It is arguably contraindicated in cases of
esophageal varices, as placement of the tube may exacerbate a variceal bleed by disrupting a thrombu s or worsening the culprit
bleeding lesion.

A tagged red blood cell scan (Choice D) can be used in the diagnosis of lower gastrointestinal bleeding to evaluate for the site of
bleeding. In this case, the patient is likely experiencing upper gastrointestinal bleeding, which can be diagnosed with upper endoscopy.
Regardless, further diagnostic tests are not appropriate prior to managing this patient's hemodynamic instability.

Upper endoscopy (Choice E) is the definitive means of diagnosis and treatment for a patient with gastrointestinal bleeding suspected to
come from the esophagus, stomach, or duodenum (upper GI tract). However, it should take place after fluid resuscitation and
stabilization of the patient's hemodynamics.

Educational Objective: Gastritis can present with epigastric pain, nausea, and emesis that may be blood-streaked, frankly bloody, or
coffee-ground in character. In patients with an upper GI bleed that are hemodynamically unstable, management should begin with
immediate intravenous fluid resuscitation, typically with isotonic crystalloids such as 0.9% saline.

.,.

r ~, ~ ~ r,
Previous Next Score Report
https://t.me/USMLENBME2CK Lab Values Calculator Help Pause
Exam Section: Item 10 of 50 National Board of Medical Examiners
■ Mark Emergency Medicine Self-Assessment
....

10. A 23-year-old man with major depressive disorder is brought to the emergency department by paramedics after jumping from a third-story
apartment window in a suicide attempt. The fall was witnessed by the patient's girlfriend , who says that he has been despondent since losing his
job. En route to the hospital, he lost consciousness and bag-valve-mask ventilation was begun. A cervical collar is in place. The patient withdraws
to painful stimuli but does not open his eyes. His temperature is 36°C (96.8°F), pulse is 100/min, respirations are 7/min, and blood pressure is
90/50 mm Hg. Pu lse oximetry on room air shows an oxygen saturation of 96%. Exami nation shows contusions and lacerations over the scalp.
Which of the following is the most appropriate next step in management?

A) CT scan of the head


B) Determ ination of Glasgow Coma Scale score
C) Full neurologic examination
D) Orotracheal intubation
E) X-rays of the cervical spine
Correct Answer: D.

The initial evaluation of a critically ill patient includes assessing the airway, breathing , circulation, central nervous system , and any
external threats to morbidity or mortality. Airway management should take place prior to further management. In acute trauma , airway
and breathing must be managed before circulation and central nervous system lesions. A patient with a compromised airway and
insufficient respiratory drive is at immediate risk of decompensation and death. Examples of airway compromise include facial or neck
trauma, airway foreign bodies , expanding neck hematomas, uncontrolled emesis, or loss of airway protective reflexes (altered mental
status). Orotracheal intubation and ventilation is the most appropriate next step for this patient with a depressed Glasgow Coma Scale
score, bradypnea , and obtundation or coma.

Incorrect Answers: A , B, C, and E.

CT scan of the head (Choice A) will be critical in the diagnosis and management of this patient with head trauma. However, any
diagnostic imaging should be performed after the evaluation and management of the patient's airway, breathing , and circulation.

Determination of the Glasgow Coma Scale score (Choice B) is a part of the primary survey after airway, breathing , and circulation.
While the Glasgow Coma Scale score can help determine the need for intubation, this patient will already require airway management
caused by a decreased respiratory rate and altered mental status.

Full neurologic examination (Choice C) is appropriate in the setting of head trauma and altered mental status; however, a detailed
examination and further secondary survey should be performed after managing the patient's airway, breathing , and circulation.

- - - - - -- - - -- -- - - -- -- - - - - - - -- - - - -- -- - - - - -- - - - - - -- - - - - - - - -- - - - - - - - - - -
r ~, ~ p r,
Previous Next Score Report
https://t.me/USMLENBME2CK Lab Values Calculator Help Pause
Exam Section : Item 10 of 50 National Board of Medical Examiners
■ Mark Emergency Medicine Self-Assessment
...
A) CT scan of the head
B) Determination of Glasgow Coma Scale score
C) Full neurologic examination
D) Orotracheal intubation
E) X-rays of the cervical spine
Correct Answer: D.

The initial evaluation of a critical ly ill patient includes assessing the airway, breathing, circu lation, central nervous system, and any
external threats to morbidity or mortality. Airway management should take place prior to further management. In acute trauma, airway
and breathing must be managed before circulation and central nervous system lesions. A patient with a compromised airway and
insufficient respiratory drive is at immediate risk of decompensation and death. Examples of airway compromise include facial or neck
trauma, airway foreign bodies, expanding neck hematomas, uncontrolled emesis, or loss of airway protective reflexes (altered mental
status). Orotracheal intubation and ventilation is the most appropriate next step for this patient with a depressed Glasgow Coma Scale
score, bradypnea, and obtundation or coma.

Incorrect Answers: A, B, C, and E.

CT scan of the head (Choice A) wil l be critical in the diagnosis and management of this patient with head trauma. However, any
diagnostic imaging should be performed after the evaluation and management of the patient's airway, breathing, and circu lation.

Determination of the Glasgow Coma Scale score (Choice B) is a part of the primary survey after airway, breathing, and circulation.
While the Glasgow Coma Scale score can help determine the need for intubation, this patient wi ll already require airway management
caused by a decreased respiratory rate and altered mental status.

Full neurologic examination (Choice C) is appropriate in the setting of head trauma and altered mental status; however, a detailed
examination and further secondary survey should be performed after managing the patient's airway, breathing, and circu lation.

X-rays of the cervical spine (Choice E) may be appropriate in the setting of trauma; however, any diagnostic imaging shou ld be
attempted after securing the patient's airway in the setting of altered mental status and head trauma.

Educational Objective: In the setting of acute head trauma, patients who present with bradypnea and obtundation are at risk for airway
compromise. Endotracheal intubation should be the next step in management to secure the patient's airway.

.,.

r ~, ~ ~ r,
Previous Next Score Report
https://t.me/USMLENBME2CK Lab Values Calculator Help Pause
Exam Section : Item 11 of 50 National Board of Medical Examiners
■ Mark Emergency Medicine Self-Assessment

11. A 25-year-old man comes to the emergency department because of a 1-week history of right knee pain and swelling and a 2-day history of pain
and swel ling in his left elbow. He has not had any other symptoms. He has no history of serious illness and takes no med ications. His temperature
is 38°C (100.4 °F), pulse is 80/min, respirations are 16/min, and blood pressure is 140/70 mm Hg. Pulse oximetry on room ai r shows an oxygen
saturation of 100%. Physical exam ination discloses a swollen and erythematous left elbow. Passive and active range of motion of the elbow is
limited. Rad ial pulses are 2+ bi laterally. Examination of the right knee discloses swell ing and erythema. Active and passive range of motion of the
knee is limited. Arthrocentesis of the knee is done, and results of synovial fluid analysis show a leukocyte cou nt of 75,000/mm 3 (75 x 10 9/L). Gram
stain of the fluid shows no organisms. Microscopic exami nation is negative for crystals. In add ition to culturing the synovial fluid of the elbow, which
of the following is the most appropriate next step in management?

A) Cu ltures of urethral swab


B) lntraarticular injection of triamci nolone
C) Oral azith romycin therapy
D) Oral colchicine therapy
E) Serum antistreptolysin O test
Correct Answer: A.

Neisseria gonorrhoeae is a sexually transmitted Gram-negative diplococci and can cause pharyngitis, urethritis, epididymitis, vaginitis,
cervicitis, and pelvic inflammatory disease. It can also cause disseminated infection if it persists untreated. When patients present with
disseminated infection, there are often no proceeding symptom1s suggesting a sexually transmitted infection. These patients present
with fevers, malaise, polyarthritis or oligoarthritis, tenosynovitis, and vesiculopustular dermatitis. However, patients may also present
solely with purulent arthritis of one or more extremity joints (eg, knees, elbows, ankles), as in this patient. Further evaluation of a septic
joint (as evidenced in this patient by swollen, erythematous joints with limited passive and active ranges of motion) includes
arthrocentesis for synovial fluid analysis. In the setting of gonococcal arthritis, the synovial fluid leukocyte count will be greater than
20,000 to 50,000/mm 3 but no organisms will be visible on Gram stain because Neisseria gonorrhoeae is an intracellular organism.
Blood cu ltures and cultures of the synovial fluid should also be obtained. Therefore, in this patient whose arthralgias are likely caused
by disseminated gonorrhea, cultures of a urethral swab should also be collected to diagnose an underlying gonococcal infection.

Incorrect Answers: B, C, D, and E.

lntraarticular injections of triamcinolone (Choice B) can be used in the management of some causes of joint pain, such as that caused
by rheumatoid arthritis or osteoarthritis. Injection of glucocorticoids into a septic joint is contraindicated , as it can lead to worsening of
the infection.

Oral azithromycin therapy (Choice C) may be warranted in the treatment of this patient. It is effective against Chlamydia trachomatis, ....
- - - - - - - -- -- - - - - - - - - - - -- -- - - .. - - - - - - - - - - - - - - - - - - - - - - -- - - - - - - - -- - - - - - -
r ~, ~ ~ r,
Previous Next Score Report
https://t.me/USMLENBME2CK Lab Values Calculator Help Pause
Exam Section : Item 11 of 50 National Board of Medical Examiners
■ Mark Emergency Medicine Self-Assessment
...
Correct Answer: A.

Neisseria gonorrhoeae is a sexually transmitted Gram-negative diplococci and can cause pharyngitis, urethritis, epididymitis, vaginitis,
cervicitis, and pelvic inflammatory disease. It can also cause disseminated infection if it persists untreated. When patients present with
disseminated infection, there are often no proceeding symptom1s suggesting a sexually transmitted infection. These patients present
with fevers, malaise, polyarthritis or oligoarthritis, tenosynovitis, and vesiculopustular dermatitis. However, patients may also present
solely with purulent arthritis of one or more extremity joints (eg, knees, elbows, ankles), as in this patient. Further evaluation of a septic
joint (as evidenced in this patient by swollen, erythematous joints with limited passive and active ranges of motion) includes
arthrocentesis for synovial fluid analysis. In the setting of gonococcal arthritis, the synovial fluid leukocyte count will be greater than
20,000 to 50,000/mm 3 but no organisms will be visible on Gram stain because Neisseria gonorrhoeae is an intracellular organism.
Blood cultures and cultures of the synovial fluid should also be obtained. Therefore, in this patient whose arthralgias are likely caused
by disseminated gonorrhea, cu ltures of a urethral swab should also be collected to diagnose an underlying gonococcal infection.

Incorrect Answers: B, C, D, and E.

lntraarticular injections of triamcinolone (Choice B) can be used in the management of some causes of joint pain, such as that caused
by rheumatoid arthritis or osteoarthritis. Injection of glucocorticoids into a septic joint is contraindicated, as it can lead to worsening of
the infection.

Oral azithromycin therapy (Choice C) may be warranted in the treatment of this patient. It is effective against Chlamydia trachomatis,
which is a common coinfection in patients with Neisseria gonorrhoeae. However, prior to treatment with antibiotics, a urethral culture
should be obtained to confirm a gonococcal infection.

Oral colchicine therapy (Choice D) can be used in the treatment of acute gout flares, which similarly present with arthralgia, although it
is more commonly limited to a sing le joint. Additionally, synovial fluid analysis will show crystals.

Serum antistreptolysin O test (Choice E) can be used in the diagnosis of acute rheumatic fever, which presents with fever, arthritis,
subcutaneous nodules, erythema marginatum, pancarditis, and Sydenham chorea following a streptococcal infection. This patient is
presenting solely with oligoarthritis, making gonococcal infection more likely.

Educational Objective: Neisseria gonorrhoeae is a sexually transmitted disease that can cause septic arthritis when disseminated, even
with an asymptomatic primary infection. Gonococcal arthritis presents with oligo- or polyarthritis associated with erythema, tenderness,
and limited active and passive ranges of motion. Synovial fluid analysis will show an increased white blood cell count, but Gram stain
may be negative as Neisseria gonorrhoeae is an intracellular organism.

.,.

r ~, ~ ~ r,
Previous Next Score Report
https://t.me/USMLENBME2CK Lab Values Calculator Help Pause
Exam Section : Item 12 of 50 National Board of Medical Examiners
■ Mark Emergency Medicine Self-Assessment

12. A 37-year-old woman comes to the emergency department because of a 9-hour history of severe headache and vomiting that began sudden ly.
She describes the headache as the "worst of my life ." It began while she was working on her computer. She has not had fever, trauma to the head,
or sensitivity to light or sound. She has no history of serious illness and takes no medications. Her temperature is 37°C (98.6°F), pulse is 92/min,
respirations are 16/min, and blood pressure is 124/78 mm Hg. Pulse oximetry on room air shows an oxygen saturation of 99%. Exam ination shows
no neck pain or rigidity. Neurologic examination shows no focal findings. A CT scan of the head shows no abnormalities. Which of the following is
the most appropriate next step in evaluation?

A) Carotid ultrasonography
B) EEG
C) Lumbar puncture
D) Measurement of erythrocyte sed imentation rate
E) MRI of the brain
Correct Answer: C.

Noncontrast CT scan of the head is a highly sensitive, but not perfectly sensitive method for the detection of subarachnoid hemorrhage
(SAH). The sensitivity of a CT scan for SAH decreases over time from the onset of bleeding or with smaller volume hemorrhages.
Patients with a presenting history concerning for SAH who have a negative CT scan should undergo emergent lumbar puncture.
Lumbar puncture is useful for identifying findings of SAH, including xanthochromia, increased opening pressure, or an increased
cerebrospinal fluid concentration of red blood cells that does not clear across several tubes of collected fluid. CT scan is most sensitive
to detect SAH when performed within 6 hours from the onset of symptoms and may reveal hyperdense blood products in the
subarachnoid space or ventricles.

Incorrect Answers: A, B, D, and E.

Carotid ultrasonography (Choice A) is useful for the diagnosis of carotid stenosis but has little role in the diagnosis of SAH.

EEG (Choice B) is useful for the evaluation of seizures but has little role in the diagnosis of SAH.

Measurement of erythrocyte sedimentation rate (Choice D) is useful for the diagnosis of temporal arteritis. Temporal arteritis is rare in
patients younger than age 50 years. It typically presents with unilateral or bilateral temporal headaches of subacute onset that are often
accompanied by jaw claudication, amaurosis fugax, or sudden vision loss. This patient's history of severe, acute headache and
vomiting is more consistent with SAH.

r ~,
- - -- - - -- - - - -- - - - - - - - -- - - -- -- - - - - - - - - - - - .
MRI of the brain (Choice E) has high sensitivity for the detection of SAH but is a time-consuming study and will result in a delay in
~ ~ r,
....

Previous Next Score Report


https://t.me/USMLENBME2CK Lab Values Calculator Help Pause
Exam Section : Item 12 of 50 National Board of Medical Examiners
■ Mark Emergency Medicine Self-Assessment
...
A) Carotid ultrasonography
B) EEG
C) Lumbar puncture
D) Measurement of erythrocyte sed imentation rate
E) MRI of the brain
Correct Answer: C.

Noncontrast CT scan of the head is a highly sensitive, but not perfectly sensitive method for the detection of subarachnoid hemorrhage
(SAH). The sensitivity of a CT scan for SAH decreases over time from the onset of bleeding or with smaller volume hemorrhages.
Patients with a presenting history concerning for SAH who have a negative CT scan should undergo emergent lumbar puncture.
Lumbar puncture is useful for identifying findings of SAH, including xanthochromia, increased opening pressure, or an increased
cerebrospinal fluid concentration of red blood cells that does not clear across several tubes of collected fluid. CT scan is most sensitive
to detect SAH when performed within 6 hours from the onset of symptoms and may reveal hyperdense blood products in the
subarachnoid space or ventricles.

Incorrect Answers: A, B, D, and E.

Carotid ultrasonography (Choice A) is useful for the diagnosis of carotid stenosis but has little role in the diagnosis of SAH.

EEG (Choice B) is useful for the evaluation of seizures but has little role in the diagnosis of SAH.

Measurement of erythrocyte sedimentation rate (Choice D) is useful for the diagnosis of temporal arteritis. Temporal arteritis is rare in
patients younger than age 50 years. It typically presents with unilateral or bilateral temporal headaches of subacute onset that are often
accompanied by jaw claudication, amaurosis fugax, or sudden vision loss. This patient's history of severe, acute headache and
vomiting is more consistent with SAH.

MRI of the brain (Choice E) has high sensitivity for the detection of SAH but is a time-consuming study and will result in a delay in
treatment. Lumbar puncture allows for the more immediate detection of SAH.

Educational Objective: Noncontrast CT scan of the head has a high but not perfect rate of sensitivity for the detection of SAH. Patients
with a presenting history concerning for SAH but who have a negative CT scan should undergo emergent lumbar puncture, which may
disclose xanthochromia, increased opening pressure, and/or a persistent cerebrospinal fluid concentration of red blood cells.

.,.

r ~, ~ ~ r,
Previous Next Score Report
https://t.me/USMLENBME2CK Lab Values Calculator Help Pause
Exam Section: Item 13 of 50 National Board of Medical Examiners
■ Mark Emergency Medicine Self-Assessment

13. A 62-year-old woman is brought to the emergency department by her husband because of a 2-day history of generalized weakness. The husband
reports that the patient has had a poor appetite during this time and has not gotten out of bed often. Today, he noticed that she was not able to
provide coherent answers to questions. The patient's med ical history includes hypertension, type 2 diabetes mellitus, coronary artery disease with
prior stenting, and bipolar disorder. He notes that she has not taken any of her daily med ications for the past 5 days. Her prescribed med ications
are lisinopri l, hydroch lorothiazide, metformin, glyburide, and lamotrigine. She has no known allerg ies. She does not drink alcoholic beverages,
smoke cigarettes, or use illicit drugs. On arrival , she is drowsy but responds to her name. She is oriented to person on ly and is unable to engage in
a coherent conversation. Her temperatu re is 37 .3°C (99.2°F), pulse is 128/min, respirations are 20/min, and blood pressure is 178/94 mm Hg.
Pulse oximetry on room air shows an oxygen saturation of 97%. Physical examination shows dry mucous membranes. Lungs are clear to
auscultation. Heart sounds are normal. The abdomen is soft and nontender. She moves al l four extrem ities spontaneously but does not comply
with a ful l neurolog ic examination. Fingerstick blood glucose concentration is determined to be too hig h to calculate w ith a glucometer. Serum
studies show:
Na + 128 mEq/L (128 mmol/L)
K+ 3.0 mEq/L (3 mmol/L)
HCO 3- 20 mEq/L (20 mmol/L)
Glucose 950 mg/dl (52. 72 mmol/L)

In addition to 0.9% saline, wh ich of the following is the most appropriate to administer?

A) Calcium gluconate
B) Hypertonic saline
C) Insulin
D) Potassium
E) Sodium bicarbonate
Correct Answer: D.

Potassium should be administered in conjunction with 0.9% saline in this patient with hyperglycemic hyperosmolar syndrome (HHS)
and hypokalemia. HHS is a consequence of profound hyperglycem ia and manifests with increased glucose concentration (often above
600 mg/dl), altered mental status, and hypovolemia in the absence of ketosis. It is more common than diabetic ketoacidosis (OKA) in
patients with type 2 diabetes mellitus. Glucose is an osmotically active substrate and can, at high concentrations, lead to a dramatic
increase in the osmolality of the serum with consequent mental status changes. Other common laboratory findings include
pseudohyponatremia, increased serum osmolality, and hypokalemia without a metabolic acidosis. The treatment is with intravenous
hyd ration and insu li n. Intravenous potassium supplementation should always be given before insulin if the serum potassium

r
..
~,
-· . - - - •- -. ·- .. .... -· ..... ·- ..... -· -·
-
~ ~
- -· .... r..., . .
concentration is below 3.5 mEq/L as insu li n causes potassium to shift intracellularly, which can lead to life-threatening hypokalemia with
-· .... - - •• -

Previous Next Score Report


https://t.me/USMLENBME2CK Lab Values Calculator Help Pause
Exam Section: Item 13 of 50 National Board of Medical Examiners
■ Mark Emergency Medicine Self-Assessment
• - I "' I

E) Sodium bicarbonate
Correct Answer: D.

Potassium should be administered in conjunction with 0.9% saline in this patient with hyperglycemic hyperosmolar syndrome (HHS)
and hypokalemia. HHS is a consequence of profound hyperglycem ia and manifests with increased glucose concentration (often above
600 mg/dl), altered mental status, and hypovolemia in the absence of ketosis. It is more common than diabetic ketoacidosis (OKA) in
patients with type 2 diabetes mellitus. Glucose is an osmotically active substrate and can, at high concentrations, lead to a dramatic
increase in the osmolality of the serum with consequent mental status changes. Other common laboratory findings include
pseudohyponatremia, increased serum osmolality, and hypokalemia without a metabolic acidosis. The treatment is with intravenous
hydration and insu li n. Intravenous potassium supplementation should always be given before insulin if the serum potassium
concentration is below 3.5 mEq/L as insu li n causes potassium to shift intracellularly, which can lead to life-threatening hypokalemia with
consequent cardiac arrhythmias and death. The potassium concentration should be monitored frequently during intravenous insu li n
administration and should be repleted when it falls below 3.5 mEq/L.

Incorrect Answers: A, B, C, and E.

Calcium gluconate (Choice A) should be given to patients with hypocalcemia or acute hyperkalemia. HHS does not typically resu lt in
hypocalcemia or acute hyperkalemia, so administration is not necessary unless the measured value is low.

Hypertonic saline (Choice B) is not appropriate for this patient and could potentially worsen her condition. Hyperton ic saline is typically
reserved for patients with increased intracranial pressure and those with severe, symptomatic hyponatremia .

Insulin (Choice C) is a necessary part of the treatment algorithm for HHS, but when given before repleting potassium, it can result in
life-threaten ing hypokalemia secondary to the transcellular shift of potassium into cells. Thus, potassium should be repleted first.

Sodium bicarbonate (Choice E) is occasionally indicated for patients with severe acidosis. This patient's bicarbonate is normal, so she
does not requi re sodium bicarbonate.

Educational Objective: HHS may occur in patients with type 2 diabetes mellitus as either an initial presentation of the disease, or in
patients with uncontrolled disease who fail to take their medications appropriately. It presents with profound hyperglycemia and altered
mental status from increased plasma osmolality and is treated with intravenous crystalloids and insulin. Potassium should always be
repleted first if less than 3.5 mEq/L, as insulin therapy causes a transcellular shift of potassium into the cell and can result in life-
threatening hypokalemia.

.,.

r ~, ~ ~ r,
Previous Next Score Report
https://t.me/USMLENBME2CK Lab Values Calculator Help Pause
Exam Section : Item 14 of 50 National Board of Medical Examiners
■ Mark Emergency Medicine Self-Assessment

14. A 20-year-old man is brought to the emergency department 45 minutes after he developed nausea, unsteady gait, muscle cramps, and headache
wh ile ru nning in a marathon. On arrival, he is confused and disoriented. His tem perature is 40°C (104 °F), pulse is 155/min, and blood pressure is
100/60 mm Hg. Examination shows warm skin and diaphoresis. Neurologic exam ination shows no focal findings. Which of the following is the most
likely explanation for these findings?

A) Depletion of total body magnesium


B) Depletion of total body potassium
C) High-output cardiac failure
D) Inadequate dissipation of body heat
E) Release of creatine kinase from muscle cells
Correct Answer: D.

Athletes are at an increased risk for heat exhaustion or heat stroke as a result of strenuous exercise in hot temperatures and increased
humidity. Other risk factors include lack of acclimatization, obesity, dehydration, and poor physical fitness. During exertion, the body is
able to dissipate heat by increasing blood flow to skin and producing sweat to facilitate evaporative cooling. However, when ambient
temperature is increased and humidity is high, these mechanisms are less effective at dissipating heat, potentially resulting in heat
exhaustion or heat stroke. Heat exhaustion is characterized by exercise intolerance, increased core body temperature, and the
absence of neurologic disturbances (eg, altered mental status, confusion, or seizure). Heat stroke is characterized by hyperthermia,
altered mental status or neurologic deficits, and variable physical signs and symptoms depending on severity. In this case, this patient
was running a marathon and was susceptible to overheating from the environment. His examination shows tachycardia, hypotension,
and warm, flushed skin. Laboratory work-up may disclose hypernatremia, hyperchloremia, hyperkalemia, along with increased serum
creatinine, blood urea nitrogen, and creatine kinase concentrations. Management of heat-related illness involves removing the
individual from the heat, removing excess clothing, and initiating cooling measures such as immersion in cool water or evaporative
cooling.

Incorrect Answers: A, B, C, and E.

While exercise can lead to the depletion of magnesium (Choice A), it would not explain the patient's hyperthermia and altered mental
status.

Exercise typically causes hyperkalemia, not hypokalemia (Choice B) as a result of potassium release from muscle cells. Potassium
derangement would generally only cause muscle cramps and ECG abnormalities and would not completely explain the clinical picture
in this case.
....
-- - - -- - - - - - - - -- - - -- - - - - - - ---- - - -- - ---- ---- -- - - -- - - - - -- - ----- --- -- --- -
r ~, ~ ~ r,
Previous Next Score Report
https://t.me/USMLENBME2CK Lab Values Calculator Help Pause
Exam Section : Item 14 of 50 National Board of Medical Examiners
■ Mark Emergency Medicine Self-Assessment

Correct Answer: D. ...

Athletes are at an increased risk for heat exhaustion or heat stroke as a result of strenuous exercise in hot temperatures and increased
humidity. Other risk factors include lack of acclimatization, obesity, dehydration, and poor physical fitness. During exertion, the body is
able to dissipate heat by increasing blood flow to skin and producing sweat to facilitate evaporative cooling. However, when ambient
temperature is increased and humidity is high, these mechanisms are less effective at dissipating heat, potentially resulting in heat
exhaustion or heat stroke. Heat exhaustion is characterized by exercise intolerance, increased core body temperature, and the
absence of neurologic disturbances (eg, altered mental status, confusion, or seizure). Heat stroke is characterized by hyperthermia,
altered mental status or neurologic deficits, and variable physical signs and symptoms depending on severity. In this case, this patient
was running a marathon and was susceptible to overheating from the environment. His examination shows tachycardia, hypotension,
and warm, flushed skin. Laboratory work-up may disclose hypernatremia, hyperchloremia, hyperkalemia, along with increased serum
creatinine, blood urea nitrogen, and creatine kinase concentrations. Management of heat-related illness involves removing the
individual from the heat, removing excess clothing, and initiating cooling measures such as immersion in cool water or evaporative
cooling.

Incorrect Answers: A, B, C, and E.

While exercise can lead to the depletion of magnesium (Choice A), it would not explain the patient's hyperthermia and altered mental
status.

Exercise typically causes hyperkalemia, not hypokalemia (Choice B) as a result of potassium release from muscle cells. Potassium
derangement wou ld generally on ly cause muscle cramps and ECG abnormalities and would not completely explain the clinical picture
in this case.

High-output cardiac fai lure (Choice C) presents with shortness of breath, orthopnea, paroxysmal nocturnal dyspnea, and peripheral
edema, often in the setting of an arteriovenous fistula, cirrhosis, sepsis, or severe anemia. In an athlete presenting after running a
marathon, heat stroke is more likely.

Creatine kinase is an enzyme found primarily in muscle cells, which is released in states of myocyte destruction or turnover and serves
as an accurate surrogate marker for myocyte injury. Intense exercise can lead to myocyte injury and rhabdomyolysis, characterized by
myalgias, myoglobinuria, and acute kidney injury. While exercise can release creatine kinase from muscle cells (Choice E), it would not
be sufficient to explain the patient's hyperthermia and altered mental status.

Educational Objective: Heat stroke is characterized by exercise intolerance, increased core body temperature, dehydration,
tachycardia, lightheadedness, muscle cramps, and neurologic disturbances. It occurs secondary to the body's inability to dissipate heat
during periods of intense exercise.

....

r ~, ~ ~ r,
Previous Next Score Report
https://t.me/USMLENBME2CK Lab Values Calculator Help Pause
Exam Section: Item 15 of 50 National Board of Medical Examiners
■ Mark Emergency Medicine Self-Assessment

15. A previously healthy 3-year-old boy is brought to the emergency department 1 hour after he fell off a boat during a lake party. He was submerged
for approximately 1 minute and initially had difficulty breathing and a severe cough when pulled from the water. He currently is asymptomatic. His
temperature is 36.7°C (98.1 °F), pulse is 110/min, respirations are 30/min, and blood pressure is 90/56 mm Hg. Pulse oximetry on room air shows
an oxygen saturation of 92%. Physical examination shows no abnormalities. Which of the following is the most appropriate next step in
management?

A) Discharge home and recommend follow up with his primary care provider in 1 week
B) Intravenous administration of antibiotics
C) Intravenous administration of corticosteroids
D) Observation for serial examination
E) Rapid sequence intubation
Correct Answer: D.

Near-drowning in children can result in a variety of clinical presentations depending on the severity of the drowning incident. Patients
can present with aspiration, hypoxemia, respiratory distress, and laryngospasm. Hypoxemia can lead to ischemic neuronal damage
resulting in cerebral edema. In the case of drowning, aspirated fluid causes local inflammation in the lung, along with the dilution of
surfactant, leading to impaired compliance and airspace infiltration. This can result in persistent and severe hypoxia despite
management with positive end-expiratory pressure and increased fraction of inspired oxygen. Asymptomatic patients should be
observed in the emergency department and admitted to the hospital if any clinical deterioration occurs. Symptomatic patients or
patients who have continued hypoxia should be admitted to the hospital and be monitored until symptoms and hypoxia resolve. Point-
of-time evaluation is generally insufficient in these cases, as near-drowning can result in progressively worsening inflammation and
hypoxia over hours to days, such that an initially normal evaluation may be falsely reassuring.

Incorrect Answers: A, B, C, and E.

Discharge home and follow up in 1 week (Choice A) is inappropriate in this patient who has continued mild hypoxia on room air. The
patient should be admitted for observation, as aspirated fluid can cause delayed inflammation and the patient is at risk for potential
clinical deterioration and respiratory distress.

Intravenous administration of antibiotics (Choice B) or corticosteroids (Choice C) would not be appropriate. There is no strong evidence
to support the use of prophylactic antibiotics or routine steroids in near-drowning victims. Antibiotics should only be used in patients
who later develop a pulmonary infection.

Rapid sequence intubation (Choice E) would be appropriate in patients with severe respiratory distress, hypoxic and/or hypercarbic ....
--- - - - - - -- - -- - -- - - - -- - - -- -- - -- - --- - - - - --- --- - -- -- - -- - -- -- -- --
r ~, ~ ~ r,
Previous Next Score Report
https://t.me/USMLENBME2CK Lab Values Calculator Help Pause
Exam Section: Item 15 of 50 National Board of Medical Examiners
■ Mark Emergency Medicine Self-Assessment
...
B) Intravenous adm inistration of antibiotics
C) Intravenous adm inistration of corticosteroids
D) Observation for serial examination
E) Rapid sequence intubation
Correct Answer: D.

Near-drowning in chi ldren can resu lt in a variety of clinical presentations depending on the severity of the drowning incident. Patients
can present with aspiration, hypoxemia, respiratory distress, and laryngospasm. Hypoxemia can lead to ischemic neuronal damage
resu lting in cerebral edema. In the case of drowning, aspirated fluid causes local inflammation in the lung, along with the dilution of
surfactant, leading to impaired compliance and airspace infiltration. This can result in persistent and severe hypoxia despite
management with positive end-expiratory pressure and increased fraction of inspired oxygen. Asymptomatic patients should be
observed in the emergency department and admitted to the hospital if any clinical deterioration occurs. Symptomatic patients or
patients who have continued hypoxia should be admitted to the hospital and be monitored until symptoms and hypoxia resolve. Point-
of-time evaluation is generally insufficient in these cases, as near-drowning can result in progressively worsening inflammation and
hypoxia over hours to days, such that an initially normal evaluation may be falsely reassuring.

Incorrect Answers: A, B, C, and E.

Discharge home and follow up in 1 week (Choice A) is inappropriate in this patient who has continued mild hypoxia on room air. The
patient should be admitted for observation, as aspirated fluid can cause delayed inflammation and the patient is at risk for potential
clinical deterioration and respiratory distress.

Intravenous administration of antibiotics (Choice B) or corticosteroids (Choice C) would not be appropriate. There is no strong evidence
to support the use of prophylactic antibiotics or routine steroids in near-drowning victims. Antibiotics should only be used in patients
who later develop a pulmonary infection.

Rapid sequence intubation (Choice E) would be appropriate in patients with severe respiratory distress, hypoxic and/or hypercarbic
respiratory failure, or neurologic deterioration. This patient is not in respiratory distress and intubation is not currently indicated.

Educational Objective: Aspirated fluid in a drowning incident can cause local inflammation in the lung, along with the dilution of
surfactant, leading to impaired compliance and airspace infiltration. Patients can present with respiratory distress and hypoxia.
Asymptomatic patients shou ld be monitored and admitted if clinical deterioration occurs.

.,.

r ~, ~ ~ r,
Previous Next Score Report
https://t.me/USMLENBME2CK Lab Values Calculator Help Pause
Exam Section : Item 16 of 50 National Board of Medical Examiners
■ Mark Emergency Medicine Self-Assessment

16. A 23-year-old woman with asthma comes to the emergency department because of a 24-hour history of abdominal pain and mild nausea. She
says the pain was initially generalized but is now concentrated in the right lower abdomen. She has not had vomiting, urinary symptoms, or vaginal
discharge or bleeding. Menses occur at regular intervals and last 4 days. Her last menstrual period was 3 weeks ago. She is sexually active with
one male partner; they use condoms consistently. A home pregnancy test was negative. On arrival, she appears uncomfortable. Her temperatu re
is 38.8°C (101.8°F), pulse is 113/min, respirations are 22/min, and blood pressure is 119/54 mm Hg. Pu lse oximetry on room air shows an oxygen
saturation of 98%. Abdominal examination shows tenderness to palpation in the rig ht lower quadrant with rebound tenderness and guarding. Pelvic
exam ination shows no abnormalities. Wh ile awaiting surg ical evaluation, which of the following is the most appropriate pain management?

A) Intravenous ketorolac
B) Intravenous morphine
C) Oral acetaminophen
D) Oral oxycodone
E) Withhold medication prior to surgical evaluation
Correct Answer: B.

This patient is presenting with fever, abdominal pain, and tenderness in the right lower quadrant, raising suspicion for appendicitis.
Appendicitis typical ly presents with right lower quadrant abdominal pain that is often periumbilical at onset, along with nausea,
anorexia, fever, leukocytosis, and tenderness to palpation in the right lower quadrant on examination. This patient also has signs of
complicated or ruptured appendicitis presenting with peritoneal signs such as rebound tenderness and guarding on examination.
Patients with appendicitis and other intra-abdominal infections typically have acute, severe pain that should be treated appropriately.
Morphine is an opioid analgesic used to manage moderate to severe pain. It acts on µ-opioid receptors to provide pain relief and is
available in oral, intravenous, intramuscular, and suppository formulations. It is a potent analgesic that would be effective in treating this
patient's pain.

Incorrect Answers: A, C, D, and E.

Intravenous ketorolac (Choice A) can be used to treat moderate pain, but this patient likely has severe pain from a complicated
appendicitis with peritoneal signs on examination. It would likely be inadequate in treating this patient's pain. Additionally, this patient
wi ll require surgery, and ketorolac, a nonsteroidal anti-inflammatory drug, presents an adverse effect of bleeding because of its effect
on prostaglandin pathways.

Oral acetaminophen (Choice C) and oral oxycodone (Choice D) shou ld not be given to this patient awaiting a surgical consultation. Oral
medications should be avoided in patients who may require em1ergent surgery, such as in this patient, because of the risk for aspiration
while under anesthesia. ....

r ~, ~ ~ r,
Previous Next Score Report
https://t.me/USMLENBME2CK Lab Values Calculator Help Pause
Exam Section : Item 16 of 50 National Board of Medical Examiners
■ Mark Emergency Medicine Self-Assessment
i ..,:f.• · ·· .. .. . .
C) Oral acetaminophen
D) Oral oxycodone
E) Withhold med ication prior to surgical evaluation
Correct Answer: B.

This patient is presenting with fever, abdominal pain, and tenderness in the right lower quadrant, raising suspicion for appendicitis.
Appendicitis typical ly presents with right lower quadrant abdominal pain that is often periumbilical at onset, along with nausea,
anorexia, fever, leukocytosis, and tenderness to palpation in the right lower quadrant on examination. This patient also has signs of
complicated or ruptured appendicitis presenting with peritoneal signs such as rebound tenderness and guarding on examination.
Patients with appendicitis and other intra-abdominal infections typically have acute, severe pain that should be treated appropriately.
Morphine is an opioid analgesic used to manage moderate to severe pain. It acts on µ-opioid receptors to provide pain relief and is
available in oral, intravenous, intramuscular, and suppository formulations. It is a potent analgesic that would be effective in treating this
patient's pain.

Incorrect Answers: A, C, D, and E.

Intravenous ketorolac (Choice A) can be used to treat moderate pain, but this patient likely has severe pain from a complicated
appendicitis with peritoneal signs on examination. It would likely be inadequate in treating this patient's pain. Additionally, this patient
wi ll require surgery, and ketorolac, a nonsteroidal anti-inflammatory drug, presents an adverse effect of bleeding because of its effect
on prostaglandin pathways.

Oral acetaminophen (Choice C) and oral oxycodone (Choice D) shou ld not be given to this patient awaiting a surgical consultation. Oral
medications should be avoided in patients who may require em1ergent surgery, such as in this patient, because of the risk for aspiration
while under anesthesia.

Withholding medication prior to surgical evaluation (Choice E) is not recommended, as pain should always be adequately treated. As
well, pain control with opioid analgesics has not been shown to obscure abdominal examination findings concerning for appendicitis or
other surgical emergencies. This is in contrast to classic teaching, which formerly suggested that pain medication administration can
confound the abdominal examination.

Educational Objective: Patients should always receive adequate pain management. Morphine is a potent opioid analgesic that is an
appropriate option for treating severe, acute pain.

.,.

r ~, ~ ~ r,
Previous Next Score Report
https://t.me/USMLENBME2CK Lab Values Calculator Help Pause
Exam Section : Item 17 of 50 National Board of Medical Examiners
■ Mark Emergency Medicine Self-Assessment

17. A 45-year-old woman comes to the emergency department because of a 2-day history of pain, swelling , and redness in her right knee. She is
unable to bear weight on the knee. She has not had fever or trauma. The patient has hypertension treated with hydrochloroth iazide. Her
temperature is 37°C (98 .6°F), pulse is 90/min, respirations are 16/min, and blood pressure is 140/70 mm Hg. Pulse oximetry on room air shows an
oxygen saturation of 99%. The right knee is swollen, erythematous, and warm to touch. Passive range of motion of the knee is fu ll, but active
range of motion elicits mild pain. Dorsalis pedis pulses are 2+ bilaterally. Measurement of which of the following is most likely to confirm the
diagnosis?

A) Serum C-reactive protein concentration


B) Serum o-dimer assay
C) Serum uric acid concentration
D) Synovial fluid crystal analysis
E) Synovial fluid lactic acid concentration
Correct Answer: D.

Synovial fluid crystal analysis is the most appropriate test to confirm the diagnosis. This patient with right knee pain, swelling, and
erythema most likely has gout and is experiencing an acute flare. Gout is an inflammatory joint disease that is caused by the
precipitation of uric acid crystals in the joint space, which are highly inflammatory and lead to swelling, erythema, and pain. The most
commonly affected joints are the first metatarsophalangeal (MTP) joints, but knees, elbows, wrists, and other joints can be involved.
The diagnosis is made definitively by analysis of the joint aspirate, which will demonstrate needle-shaped crystals that are negatively
birefringent under polarized light. Aside from synovial fluid crystal analysis, it is also important to perform a cell count and culture on the
joint aspirate to rule out infection as septic arthritis and acute gout attacks can exist simultaneously in the same patient and in the same
joint. Treatment for acute gout attacks includes the use of nonsteroidal anti-inflammatory drugs (NSAIDs), colchicine, or steroids. Long-
term urate lowering therapy is indicated in patients with kidney disease, tophaceous gout, and those with frequent gout attacks.

Incorrect Answers: A, B, C, and E.

Serum C-reactive protein concentration (Choice A) is an inflammatory marker that, while potentially increased in acute gout attacks, is
nonspecific. It would not be helpful in making a diagnosis in this case.

Serum d-dimer assay (Choice B) is incorrect. The d-dimer is a useful tool for patients who are thought to have a low to intermediate
probability of pulmonary embolism (PE) or deep venous thrombosis (DVT). When negative, it effectively rules out a thrombus. This
patient does not have signs or symptoms concerning for a PE or DVT.

Serum uric acid concentration (Choice C) may be deceivingly low during acute gout flares and is not used to make a diagnosis of gout. ....
- - -- - - -- -- - - -- -- - -- -- ----- - - - - -- - -- - - - - - - - - - - - - -- - - - - - - - - -- ----- - -- -
r ~, r,
...

~ ~
Previous Next Score Report
https://t.me/USMLENBME2CK Lab Values Calculator Help Pause
Exam Section : Item 17 of 50 National Board of Medical Examiners
■ Mark Emergency Medicine Self-Assessment
...
D) Synovial fluid crystal analysis
E) Synovial fluid lactic acid concentration
Correct Answer: D.

Synovial fluid crystal analysis is the most appropriate test to confirm the diagnosis. This patient with right knee pain, swelling, and
erythema most likely has gout and is experiencing an acute flare. Gout is an inflammatory joint disease that is caused by the
precipitation of uric acid crystals in the joint space, which are highly inflammatory and lead to swelling, erythema, and pain. The most
commonly affected joints are the first metatarsophalangeal (MTP) joints, but knees, elbows, wrists, and other joints can be involved.
The diagnosis is made definitively by analysis of the joint aspirate, which wi ll demonstrate needle-shaped crystals that are negatively
birefringent under polarized light. Aside from synovial fluid crystal analysis, it is also important to perform a cell count and culture on the
joint aspirate to rule out infection as septic arthritis and acute gout attacks can exist simultaneously in the same patient and in the same
joint. Treatment for acute gout attacks includes the use of nonsteroidal anti-inflammatory drugs (NSAIDs), colchicine, or steroids. Long-
term urate lowering therapy is indicated in patients with kidney disease, tophaceous gout, and those with frequent gout attacks.

Incorrect Answers: A, B, C, and E.

Serum C-reactive protein concentration (Choice A) is an inflammatory marker that, while potentially increased in acute gout attacks, is
nonspecific. It would not be helpful in making a diagnosis in this case.

Serum d-dimer assay (Choice B) is incorrect. The d-dimer is a useful tool for patients who are thought to have a low to intermediate
probability of pulmonary embolism (PE) or deep venous thrombosis (DVT). When negative, it effectively rules out a thrombus. This
patient does not have signs or symptoms concerning for a PE or DVT.

Serum uric acid concentration (Choice C) may be deceivingly low during acute gout flares and is not used to make a diagnosis of gout.
While many patients with gout have an increased uric acid concentration, there are also many patients with increased uric acid
concentrations that do not have gout.

Synovial fluid lactic acid concentration (Choice E) is not useful in the diagnosis of gout. Lactate may be high in states of infection but is
not routinely used for diagnosis.

Educational Objective: Acute gout attacks common ly present with one or more red, swollen, and painful joints. Classical ly, gout affects
the first MTP joint, but it can also affect the knees, wrists, elbows, and other joints as well. A definitive diagnosis is made by analysis of
the synovial fluid, which shows needle-shaped negatively birefringent crystals under polarized light.

.,.

r ~, ~ ~ r,
Previous Next Score Report
https://t.me/USMLENBME2CK Lab Values Calculator Help Pause
Exam Section : Item 18 of 50 National Board of Medical Examiners
■ Mark Emergency Medicine Self-Assessment

18. A previously healthy 36-year-old man comes to the emergency department because of a 1-hour history of nausea and sharp, intermittent pain in his
rig ht midback that rad iates to his right flank. He has had two episodes of vom iting during th is time. He takes no medications. His temperature is
37.0°C (98.6°F), pulse is 90/min, respirations are 20/min, and blood pressure is 140/60 mm Hg. Pulse oximetry on room air shows an oxygen
saturation of 100%. Physical exam ination shows tenderness to percussion at the right costovertebral angle. No abdom inal tenderness is noted . A
CT scan of the abdomen is shown. The most appropriate next step is adm inistration of which of the following?
A) Allopu rinol
B) Aspirin
C) Ceftriaxone
D) Esmolol
E) Tamsulosin
Correct Answer: E.

Renal calculi typically cause symptoms of unilateral acute flank or back pain, nausea, vomiting, and hematuria. There are three main ....

r ~, ~ ~ r,
Previous Next Score Report
https://t.me/USMLENBME2CK Lab Values Calculator Help Pause
Exam Section : Item 18 of 50 National Board of Medical Examiners
■ Mark Emergency Medicine Self-Assessment
...
B) Aspirin
C) Ceftriaxone
D) Esmolol
E) Tamsulosin
Correct Answer: E.

Renal calculi typically cause symptoms of unilateral acute flank or back pain, nausea, vomiting, and hematuria. There are three main
sites of anatomic narrowing along the ureter that may obstruct the passage of a calculus: at the ureteropelvic junction of the renal pelvis
and the ureter, the ureterovesical junction as the ureter enters the posterior bladder, and at the pelvic inlet when the ureter crosses the
common iliac vessel bifurcation. This patient's CT scan demonstrates a ureteral calculus at the site of the right ureterovesical junction.
The initial management of ureterolithiasis includes oral fluid hydration, analgesia, and pharmacologic therapy to facilitate the passage
of the calculus. a-Adrenergic blockers, such as tamsulosin, cause ureteral smooth muscle relaxation, which encourages passage of the
calculus.

Incorrect Answers: A, B, C, and D.

Allopurinol (Choice A) is useful for the prevention of nephrolithiasis in patients with a history of uric acid stones. Uric acid stones are
often radiolucent on CT scans.

Aspirin (Choice B) is a nonsteroidal anti-inflammatory drug (NSAID) that is useful as an analgesic. However, its antiplatelet effects
contraindicate its use in this setting, as it increases the risk for bleeding if this patient requires extracorporeal shock wave lithotripsy or
surgical ureteral stenting via cystoscopy.

Ceftriaxone (Choice C) is useful for the management of pyelonephritis. Pyelonephritis also presents with acute-onset nausea, vomiting,
and flank pain, but is typically accompanied by fever.

Esmolol (Choice D) is a cardioselective ~-adrenergic blocker that is useful for the management of supraventricular tachycardia and
other tachyarrhythmias. It does not play a significant role in the management of nephrolithiasis or ureterolithiasis.

Educational Objective: The initial management of ureterolithiasis includes oral fluid hydration, analgesia, and pharmacologic therapy to
facilitate the passage of the calculus. a-Adrenergic blockers, such as tamsu losin, cause ureteral smooth muscle relaxation, which
encourages passage of the calcu lus.

.,.

r ~, ~ ~ r,
Previous Next Score Report
https://t.me/USMLENBME2CK Lab Values Calculator Help Pause
Exam Section : Item 19 of 50 National Board of Medical Examiners
■ Mark Emergency Medicine Self-Assessment
...

19. A 45-year-old man is brought to the emergency department 4 hours after the sudden onset of severe shortness of breath. He has coronary artery
disease, chron ic obstructive pulmonary disease (COPD), recu rrent pneumonia, and congestive heart failure. His medications are albuterol,
pred nisone, aspirin, and hydralazine. He has no known allergies. On arrival, he appears pale and has labored breathing. His temperatu re is 37.5°C
(99.5°F), pulse is 124/min, respirations are 28/min, and blood pressure is 235/103 mm Hg. Pulse oximetry on room air shows an oxygen saturation
of 85%. Card iac exam ination discloses no abnormalities. Diffuse crackles are heard throughout the lung fiel ds. Breath sounds and air movement
are decreased. There is moderate pitting edema of the lower extremities. An x-ray is shown. Wh ich of the following is the most likely diagnosis?

A) Acute bi lateral pneumonia


B) Cardiogenic pulmonary edema
C) COPD exacerbation
D) Large, left-sided pleural effusion
E) Pulmonary embolism ....

r ~, ~ ~ r,
Previous Next Score Report
https://t.me/USMLENBME2CK Lab Values Calculator Help Pause
Exam Section : Item 19 of 50 National Board of Medical Examiners
■ Mark Emergency Medicine Self-Assessment

E) Pulmonary embolism
...
Correct Answer: B.

Cardiogenic pulmonary edema resu lts from increased fi lling pressures or impaired forward flow in the heart. This increases hydrostatic
pressure in the pu lmonary venous circulation, resu lting in the transudation of fluid from the pu lmonary capillaries into the pu lmonary
interstitial and alveolar spaces, impairing gas exchange. Cardiogenic pulmonary edema can be brought on suddenly by an increase in
blood pressure (known as "flash" pulmonary edema), which is likely the cause in this case. Patients with cardiogenic pulmonary edema
typically experience progressive dyspnea on exertion and then at rest, paroxysmal nocturnal dyspnea, and orthopnea. Physical
examination findings include hypoxia, crackles (representing fluid-filled alveoli opening suddenly), and decreased breath sounds.
Initially, crackles will be present only in the dependent regions of the lung, but with increasing severity of edema, will spread to involve
the lung fields diffusely. This patient's lower extremity edema further suggests a status of volume overload. The chest x-ray also
demonstrates diffuse interstitial infiltrates, suggesting pulmonary edema.

Incorrect Answers: A, C, D, and E.

Acute bilateral pneumonia (Choice A) would not explain this patient's chest x-ray findings or peripheral edema. A patient with an acute
infection may also be hypotensive as a feature of sepsis rather than severely hypertensive, as in this case.

COPD exacerbation (Choice C) is characterized by shortness of breath and shallow respirations with wheezing present on physical
examination. Accumu lation of carbon dioxide as a result of the inability to fully expire will also cause a respiratory acidosis, which may
present with altered mental status. The patient's physical examination findings are more consistent with cardiogenic pulmonary edema.

Large, left-sided pleural effusion (Choice D) would demonstrate opacification of the left hemithorax with blunting of the left costophrenic
angle. Physical examination wou ld demonstrate dullness to percussion and decreased breath sounds on the left side on ly. This
patient's physical examination and x-ray findings instead suggest cardiogenic pulmonary edema.

Pulmonary embolism (Choice E) presents with acute onset dyspnea and may present with pleuritic chest pain. Asymmetric lower
extremity edema may be present, indicating the presence of a deep venous thrombosis. However, diffuse crackles and diffuse
interstitial infiltrates are not characteristic. Additionally, hypotension and hemodynamic instability would be more likely than
hypertension, especially in the setting of a large enough embolus to resu lt in tachycardia and a new oxygen requirement.

Educational Objective: Cardiogenic pulmonary edema results from increased hydrostatic pressure in the pulmonary venous circu lation,
leading to the transudation of fluid from the pulmonary capil laries into the alveolar space. It is a feature of acute congestive heart failure
exacerbation and can be triggered by severe hypertension.

.,.

r ~, ~ ~ r,
Previous Next Score Report
https://t.me/USMLENBME2CK Lab Values Calculator Help Pause
Exam Section: Item 20 of 50 National Board of Medical Examiners
■ Mark Emergency Medicine Self-Assessment

20. A 22-year-old woman is brought to the emergency department because of a 6-hour history of severe right lower abdom inal pain and near loss of
consciousness. She also has a 2-week history of vaginal bleed ing. Her last menstrual period was 45 days ago. Menses previously occurred at
reg ular 28-day intervals . She has no history of serious illness and takes no medications. She is sexual ly active with one male partner; they use
condoms inconsistently. Her tem perature is 37°C (98.6°F), pu lse is 104/min, and blood pressure is 85/55 mm Hg. Abdominal exam ination shows
right lower quadrant tenderness on deep palpation. Pelvic exam ination shows a small amount of dark blood in the vaginal vault. The cervical os is
closed, and there is no active bleed ing. The uterus is soft and nontender. Laboratory studies show a hemoglobin concentration of 10 g/dl (100 g/L)
and a leukocyte count of 13,000/mm 3 (13 x 10 9/L). A urine pregnancy test is positive. Which of the following is the most appropriate next step to
confirm the diagnosis?

A) CT scan of the abdomen and pelvis


B) Measurement of serum ~-hCG concentration
C) Ultrasonography of the pelvis
D) Urinalysis
E) X-ray of the abdomen
Correct Answer: C.

Pregnancy is suspected when there is a missed or delayed menstrual period. Ectopic pregnancy is a nonviable, abnormal pregnancy in
which the fertilized ovum implants in the fal lopian tube (most common), on the ovary, within the peritoneal cavity, or in any non-
endometrial location. An ectopic pregnancy often presents with vaginal bleeding and abdominopelvic pain early in pregnancy caused by
the restricted area for fetal development. Physical examination may demonstrate adnexal tenderness on the side of implantation, as
well as tenderness to palpation of the lower abdomen. Ectopic pregnancy can be complicated by rupture, bleeding, and
hemoperitoneum, which can present with signs of hemodynamic instability, such as tachycardia and hypotension, as in this patient.
Ultrasonography can be used to evaluate for an intrauterine pregnancy and may potentially visualize the location of an ectopic
pregnancy, making it the most appropriate next step in the management of this patient. Ectopic pregnancy can be managed medical ly,
using methotrexate for smal l, early pregnancies, but may require salpingectomy or evacuation with laparoscopy for larger pregnancies,
in cases of medical treatment fai lure, or in any case of complication such as rupture.

Incorrect Answers: A, B, D, and E.

CT scan of the abdomen and pelvis (Choice A) may disclose the diagnosis of ectopic pregnancy. However, it exposes the patient and
her fetus (if it is a viable intrauterine pregnancy) to unnecessary radiation. Ultrasonography shou ld first be utilized to evaluate this
patient.

Measurement of the serum ~-hCG concentration (Choice B) is used in the evaluation of a potential ectopic pregnancy in ....
~

r ~, r,
- - -- - - - - -- - - - - - - - - - - - - - - - - - - - - - - - - - - - - - - - - - - - - - - - - - - - - - - - - - - - - -- - - - - A •

~ ~
Previous Next Score Report
https://t.me/USMLENBME2CK Lab Values Calculator Help Pause
Exam Section: Item 20 of 50 National Board of Medical Examiners
■ Mark Emergency Medicine Self-Assessment
...
Correct Answer: C.

Pregnancy is suspected when there is a missed or delayed menstrual period. Ectopic pregnancy is a nonviable, abnormal pregnancy in
which the fertilized ovum implants in the fallopian tube (most common), on the ovary, within the peritoneal cavity, or in any non-
endometrial location. An ectopic pregnancy often presents with vaginal bleeding and abdominopelvic pain early in pregnancy caused by
the restricted area for fetal development. Physical examination may demonstrate adnexal tenderness on the side of implantation, as
well as tenderness to palpation of the lower abdomen. Ectopic pregnancy can be complicated by rupture, bleeding, and
hemoperitoneum, which can present with signs of hemodynamic instability, such as tachycardia and hypotension, as in this patient.
Ultrasonography can be used to evaluate for an intrauterine pregnancy and may potentially visualize the location of an ectopic
pregnancy, making it the most appropriate next step in the management of this patient. Ectopic pregnancy can be managed medically,
using methotrexate for small, early pregnancies, but may require salpingectomy or evacuation with laparoscopy for larger pregnancies,
in cases of medical treatment failure, or in any case of complication such as rupture.

Incorrect Answers: A, B, D, and E.

CT scan of the abdomen and pelvis (Choice A) may disclose the diagnosis of ectopic pregnancy. However, it exposes the patient and
her fetus (if it is a viable intrauterine pregnancy) to unnecessary radiation. Ultrasonography should first be utilized to evaluate this
patient.

Measurement of the serum ~-hCG concentration (Choice B) is used in the evaluation of a potential ectopic pregnancy in
hemodynamically stable patients with a pregnancy of unknown location. ~-hCG concentration should double approximately every 48
hours in a normal intrauterine pregnancy but does not increase as expected in an ectopic pregnancy. This patient is hemodynamically
unstable and should be evaluated with ultrasonography first to determine whether emergent surgical intervention is warranted.

Urinalysis (Choice D) can be used to diagnose urinary tract infections, such as cystitis or pyelonephritis. These present with dysuria,
frequency, and urgency, as well as fever and flank pain in pyelonephritis. These would not explain this patient's hemodynamic instability
in the setting of her positive pregnancy test and vaginal bleeding.

X-ray of the abdomen (Choice E) is used in the evaluation of small bowel obstruction, and occasionally, nephrolithiasis. It would not be
useful in the diagnosis of ectopic pregnancy.

Educational Objective: Ectopic pregnancy occurs as a result of the nonviable implantation of an embryo in the fallopian tube, ovary, or
peritoneal cavity, and presents with abdominopelvic pain and vaginal bleeding in the setting of a positive pregnancy test. It can be
potentially complicated by rupture, hemoperitoneum, and hemodynamic instability. Initial evaluation should include ultrasonography of
the pelvis to aid in diagnosis, as a ruptured ectopic pregnancy is a surgical emergency.

.,.

r ~, ~ ~ r,
Previous Next Score Report
https://t.me/USMLENBME2CK Lab Values Calculator Help Pause
Exam Section : Item 21 of 50 National Board of Medical Examiners
■ Mark Emergency Medicine Self-Assessment

21. A 70-year-old woman with a 20-year history of hyperl ipidemia and hypertension is brought to the emergency department 30 minutes after an
episode of facial drooping and difficu lty speaking. Her symptoms lasted 30 minutes and have now resolved. Medications include lisinopril and
pravastatin. She has smoked one pack of cigarettes daily for 40 years. Vital signs are within normal limits. Physical examination, including
neurologic examination, shows no abnormal ities. An ECG shows a normal sinus rhythm at a rate of 80/min. Noncontrast CT scan of the head
shows no evidence of hemorrhage or infarct. Wh ich of the following drugs is most appropriate to admin ister at this time?

A) Aspirin
B) Clopidogrel
C) Dipyridamole
D) Tissue plasminogen activator
E) Warfarin
Correct Answer: A.

Transient ischemic attack (TIA) is defined as a transient episode of neurologic dysfunction lasting less than 24 hours caused by focal
brain, spinal cord, or retinal ischemia without evidence of infarction. This patient has developed focal neurologic deficits that resolved
after 30 minutes, qualifying as a TIA. TIA can be induced by either an embolus arising from a proximal site or from temporarily
decreased blood flow to a watershed area in the brain. A "low-flow" TIA typically lasts minutes while an embolic TIA lasts longer, as in
this case. An embolic ischemic stroke involves an event in which a clot breaks off and travels to the brain, occluding blood flow within a
cerebral artery. Emboli most commonly originate from proximal vessels, such as the internal carotid artery, or from an atrial thrombus. If
the embolus successfully undergoes physiologic thrombolysis, blood flow is restored to the affected area and reverses the clinical
symptoms. After a TIA, measures for the prevention of an ischemic stroke should be enacted. In this case, the most appropriate
pharmacotherapy to administer for secondary prophylaxis of a cerebral infarction is aspirin. Hypertension and hyperlipidemia are also
stroke risk factors which should be optimized, though in this case the patient is already receiving treatment for these comorbidities.

Incorrect Answers: B, C, D, and E.

Clopidogrel (Choice B) or dipyridamole (Choice C) may be used as combination therapy with aspirin in patients with a TIA or stroke who
are already taking aspirin. However, as this patient's only previous medications are lisinopril and pravastatin, aspirin is the first-line
choice for secondary prophylaxis of a cerebral infarction.

Tissue plasminogen activator (Choice D) is used for its thrombolytic properties to treat ischemic stroke provided that it can be initiated
within 4.5 hours of symptom onset. Because this patient's symptoms have resolved, she does not require tissue plasminogen activator.

Oral anticoagulation with warfarin (Choice E) may be indicated in a patient with atrial fibrillation who develops a TIA. However, in this ....
-- -- - --- - - - - -- - - - -- - - - -- - - - -- - - - - - - - - -- - - - -- - - - - - - -- - - - - - -- -- -- - -- ----- - - -----
r ~, ~ ~ r,
Previous Next Score Report
https://t.me/USMLENBME2CK Lab Values Calculator Help Pause
Exam Section : Item 21 of 50 National Board of Medical Examiners
■ Mark Emergency Medicine Self-Assessment
i •• i i i -

C) Dipyridamole
D) Tissue plasminogen activator
E) Warfarin
Correct Answer: A.

Transient ischemic attack (TIA) is defined as a transient episode of neurologic dysfunction lasting less than 24 hours caused by focal
brain, spinal cord, or retinal ischemia without evidence of infarction. This patient has developed focal neurologic deficits that resolved
after 30 minutes, qualifying as a TIA. TIA can be induced by either an embolus arising from a proximal site or from temporarily
decreased blood flow to a watershed area in the brain. A "low-flow" TIA typically lasts minutes while an embolic TIA lasts longer, as in
this case. An embolic ischemic stroke involves an event in which a clot breaks off and travels to the brain, occluding blood flow within a
cerebral artery. Emboli most common ly originate from proximal vessels, such as the internal carotid artery, or from an atrial thrombus. If
the embolus successfu lly undergoes physiologic thrombolysis, blood flow is restored to the affected area and reverses the clinical
symptoms. After a TIA, measures for the prevention of an ischemic stroke should be enacted. In this case, the most appropriate
pharmacotherapy to administer for secondary prophylaxis of a cerebral infarction is aspirin. Hypertension and hyperlipidemia are also
stroke risk factors which should be optimized, though in this case the patient is already receiving treatment for these comorbidities.

Incorrect Answers: B, C, D, and E.

Clopidogrel (Choice B) or dipyridamole (Choice C) may be used as combination therapy with aspirin in patients with a TIA or stroke who
are already taking aspirin. However, as this patient's only previous medications are lisinopril and pravastatin, aspirin is the first-line
choice for secondary prophylaxis of a cerebral infarction.

Tissue plasminogen activator (Choice D) is used for its thrombolytic properties to treat ischemic stroke provided that it can be initiated
within 4.5 hours of symptom onset. Because this patient's symptoms have resolved, she does not require tissue plasminogen activator.

Oral anticoagulation with warfarin (Choice E) may be indicated in a patient with atrial fibrillation who develops a TIA. However, in this
patient whose only comorbidities are hyperlipidemia and hypertension, warfarin is not indicated and carries an unnecessary increased
risk for bleeding.

Educational Objective: Transient ischemic attack (TIA) is defined as a transient episode of neurologic dysfunction lasting less than 24
hours caused by focal brain, spinal cord, or retinal ischemia without evidence of infarction. TIA is most commonly caused by a cerebral
embolic event. Aspirin is the first-line medication for secondary prophylaxis of a cerebral infarction.

.,.

r ~, ~ ~ r,
Previous Next Score Report
https://t.me/USMLENBME2CK Lab Values Calculator Help Pause
Exam Section: Item 22 of 50 National Board of Medical Examiners
■ Mark Emergency Medicine Self-Assessment

22. A 26-year-old woman comes to the emergency department because of a 3-day history of lower abdominal pain and yellow vaginal discharge. She
says her pain has increased today and that she has a fever. She has not had flank pain, urinary urgency, or vom iting. She became sexual ly active
with a new male partner 2 weeks ago; they use condoms inconsistently. The patient has no history of serious illness and takes no medications.
She has no known al lergies. She is lying on a stretcher and is in mild distress. BM I is 22 kg/m 2. Her temperatu re is 38.1 °C (100.6°F), pulse is
96/min, respirations are 14/min, and blood pressure is 124/74 mm Hg. Pulse oximetry on room air shows an oxygen saturation of 97%.
Cardiopulmonary exam ination shows no abnormalities. There is tenderness to palpation of the lower abdomen bilateral ly; there is no guarding.
Bowel sounds are normal. Pelvic examination shows a friable cervix, cervical motion tenderness, and adnexal tenderness bilaterally. Wh ich of the
following is the most appropriate pharmacotherapy?

A) Azithromycin and metronidazole


B) Ceftriaxone and doxycycline
C) Ciprofloxacin and vancomycin
0) Cl indamycin and ampicil lin
E) Penicillin and tetracycline
Correct Answer: B.

Chlamydia trachomatis and Neisseria gonorrhoeae can cause vaginitis, cervicitis, and pelvic inflammatory disease (PIO) in women.
Risk factors for PIO and other sexually transmitted infections include adolescence, multiple sexual partners, and not using barrier
contraception such as condoms. Vaginitis typically presents with mucopurulent discharge and pruritus. Cervicitis presents with simi lar
discharge, but also with an erythematous, friable cervix and cervical motion tenderness on examination. When infection spreads to the
fallopian tubes and ovaries, pelvic inflammatory disease with purulent cervical discharge, uterine and adnexal tenderness, and systemic
symptoms such as fever can result. If left untreated, PIO can become complicated by a tube-ovarian or intraabdominal abscess,
presenting with focal tenderness and a fluctuant mass. Treatment of PIO involves intravenous antibiotics such as cephalosporins and
tetracyclines to cover potential infectious organisms, making ceftriaxone and doxycycline the most appropriate pharmacotherapy.

Incorrect Answers: A, C, 0, and E.

Azithromycin and metronidazole (Choice A) would treat an infection from Chlamydia trachomatis or trichomoniasis, as well as bacterial
vaginosis. However, it does not provide coverage for Neisseria gonorrhoeae, which is a common cause of PIO.

Ciprofloxacin and vancomycin (Choice C) would not appropriately cover the causative organisms of PIO. Whi le ciprofloxacin was
previously used to treat Neisseria gonorrhoeae, resistance in the United States is now common. Vancomycin is most commonly used to
treat methicillin-resistant Staphylococcus aureus.

r ~,
- - - -- - - - - - - -- - - - -- -- . - - -- - - - - - - - - - - - - -- - - - -- - - -
~
- --- -
~
- - -- --
r,
- - - - - - - - ---
....

Previous Next Score Report


https://t.me/USMLENBME2CK Lab Values Calculator Help Pause
Exam Section: Item 22 of 50 National Board of Medical Examiners
■ Mark
. . . ' .. ..
..
E) Penicillin and tetracycline
Emergency Medicine Self-Assessment

Correct Answer: B.

Chlamydia trachomatis and Neisseria gonorrhoeae can cause vaginitis, cervicitis, and pelvic inflammatory disease (PIO) in women.
Risk factors for PIO and other sexually transmitted infections include adolescence, multiple sexual partners, and not using barrier
contraception such as condoms. Vaginitis typically presents with mucopurulent discharge and pruritus. Cervicitis presents with simi lar
discharge, but also with an erythematous, friable cervix and cervical motion tenderness on examination. When infection spreads to the
fallopian tubes and ovaries, pelvic inflammatory disease with purulent cervical discharge, uterine and adnexal tenderness, and systemic
symptoms such as fever can result. If left untreated, PIO can become complicated by a tuba-ovarian or intraabdominal abscess,
presenting with focal tenderness and a fluctuant mass. Treatment of PIO involves intravenous antibiotics such as cephalosporins and
tetracyclines to cover potential infectious organisms, making ceftriaxone and doxycycline the most appropriate pharmacotherapy.

Incorrect Answers: A, C, 0, and E.

Azithromycin and metronidazole (Choice A) would treat an infection from Chlamydia trachomatis or trichomoniasis, as well as bacterial
vaginosis. However, it does not provide coverage for Neisseria gonorrhoeae, which is a common cause of PIO.

Ciprofloxacin and vancomycin (Choice C) would not appropriately cover the causative organisms of PIO. Whi le ciprofloxacin was
previously used to treat Neisseria gonorrhoeae, resistance in the United States is now common. Vancomycin is most commonly used to
treat methicillin-resistant Staphylococcus aureus.

Clindamycin and ampicillin (Choice 0) would treat infections caused primarily by Gram-positive bacteria, such as Streptococcus and
Staphylococcus. Both Chlamydia trachomatis and Neisseria gonorrhoeae are Gram-negative organisms, and thus would not be
properly treated with clindamycin or ampicillin. Both of these agents do have partial Gram-negative coverage but are insufficient in
breadth to cover PIO.

Penicil lin and tetracycline (Choice E) wou ld not appropriately cover the causative organisms of PIO. While tetracycline can be used in
the treatment of Chlamydia trachomatis, this pairing of antibiotics does not offer appropriate coverage for Neisseria gonorrhoeae.

Educational Objective: Pelvic inflammatory disease is caused by an ascending sexually transmitted infection, such as Chlamydia
trachomatis or Neisseria gonorrhoeae. It presents with purulent cervical discharge, uterine and adnexal tenderness, cervical motion
tenderness, and systemic symptoms such as fever. Treatment includes antibiotic therapy with agents that cover potential infectious
organisms, such as ceftriaxone and doxycycline.

.,.

r ~, ~ ~ r,
Previous Next Score Report
https://t.me/USMLENBME2CK Lab Values Calculator Help Pause
Exam Section: Item 23 of 50 National Board of Medical Examiners
■ Mark Emergency Medicine Self-Assessment
...

23. A 72-year-old man is brought to the emergency department 25 minutes after a motor vehicle collision in which he was the restrained driver. He had
no loss of consciousness following the collision. On arrival, he is alert and fully oriented; he reports severe abdominal pain. His pulse is 110/min,
respirations are 26/min, and blood pressure is 130/70 mm Hg. There are large ecchymoses over the left shoulder, chest, and lower abdomen.
Breath sounds are normal bilaterally. Cardiac examination shows no abnormalities. Abdominal examination shows severe left upper quadrant
tenderness with rebound. There is no posterior midline cervical spine tenderness. Abdominal ultrasonography for trauma (FAST) is shown. Which
of the following is the most likely diagnosis?
A) Duodenal hematoma
B) Laceration of the inferior vena cava
C) Laceration of the left kidney
D) Perforated viscus
E) Splenic rupture ....

r ~, ~ ~ r,
Previous Next Score Report
https://t.me/USMLENBME2CK Lab Values Calculator Help Pause
Exam Section: Item 23 of 50 National Board of Medical Examiners
■ Mark Emergency Medicine Self-Assessment
...
Correct Answer: E.

The spleen is one of the most common organs damaged during blunt abdominal trauma, with injury patterns such as laceration and
rupture. Ultrasonography demonstrates the left upper quadrant of the abdomen, with hypoechoic fluid in between the spleen and the
left kidney, raising concern for acute blood products in the setting of trauma. Splenic rupture can cause large amounts of blood to leak
into the peritoneal and retroperitoneal spaces, which can result in hemodynamic instability characterized by tachycardia and if severe,
hypotension. Patients with splenic injuries may have left upper quadrant tenderness, peritonitis from blood products irritating the
peritoneum, or referred pain to the left shoulder secondary to diaphragmatic irritation. In hemodynamically stable patients, minor splenic
injuries may be treated nonoperatively with observation and reassessment. In unstable patients, exploratory laparotomy is required for
repair of the injury or splenectomy, depending on the nature and severity of the splenic injury.

Incorrect Answers: A, B, C, and D.

Duodenal hematoma (Choice A) can occur after blunt abdominal trauma to the epigastric region, especially in children (eg, seat belt
injury). The presentation can be subtle with nonspecific symptoms such as nausea, vomiting, and decreased oral intake. Patients may
demonstrate a midline mass on examination along with increased amylase and lipase concentrations on laboratory examination caused
by associated pancreatitis. Duodenal hematomas can be diagnosed on CT scan of the abdomen, but not on a focused assessment with
sonography for trauma (FAST) examination, which evaluates for free fluid in the abdomen.

Laceration of the inferior vena cava (IVC) (Choice B) is an uncommon injury because of its retroperitoneal position and protection by
surrounding structures. An IVC injury can lead to severe hemorrhage and high mortality. Because of its retroperitoneal location, free
fluid may not appear on a FAST examination, which primarily evaluates for intraperitoneal blood and fluid.

As a result of its retroperitoneal location, laceration of the left kidney (Choice C) would not cause intraperitoneal free fluid. A FAST
examination showing intraperitoneal free fluid in the left upper quadrant is more likely caused by a splenic injury. Kidney injury can
present with hematuria and is best evaluated with a CT scan.

Perforated viscus (Choice D) can present after blunt abdominal trauma with abdominal pain, distention, tenderness to palpation,
peritonitis, anemia, leukocytosis, and x-ray evidence of pneumoperitoneum. Peritonitis requires an emergent surgical laparotomy to
identify and repair any perforation, lavage the peritoneum, and prevent sepsis, exsanguination, and bowel necrosis.

Educational Objective: The spleen is one of the most commonly injured organs in the setting of blunt abdominal trauma . It can present
with abdominal tenderness and if severe, hemodynamic instability from large volume hemoperitoneum. Abdominal ultrasonography with
FAST can evaluate for intraperitoneal free fluid.

.,.

r ~, ~ ~ r,
Previous Next Score Report
https://t.me/USMLENBME2CK Lab Values Calculator Help Pause
Exam Section: Item 24 of 50 National Board of Medical Examiners
■ Mark Emergency Medicine Self-Assessment

24. A 48-year-old man is broug ht to the emergency department by paramed ics 30 minutes after he sustained a gunshot wound to the chest. He is
anxious and has severe shortness of breath . His tem peratu re is 37°C (98.6°F), pulse is 122/min, respirations are 20/min, and blood pressu re is
82/55 mm Hg. Exam ination shows an open gunshot wou nd on the right side of the chest. There is jugular venous distention. Breath sou nds are
decreased, and there is palpable crepitus over the right hemithorax. Peripheral pulses are weak throughout. Which of the following is the most
appropriate next step in management?

A) Chest x-ray
B) CT scan of the chest
C) Needle thoracostomYi
D) Tracheal intu bation and mechan ical ventilation
E) Transfusion of packed red blood cells
Correct Answer: C.

Tension pneumothorax can be a complication of a penetrating thoracic trauma and occurs when air is able to enter the pleural space
but is prevented from exiting. Increasing amounts of trapped air in the pleural space compresses the lung parenchyma, creating an
inability for the lung to expand and causing it to collapse. As the pneumothorax progresses and continues to increase in size, it can
resu lt in tracheal or mediastinal deviation away from the affected hemithorax. It can also result in decreased venous return to the heart,
which manifests with hypotension and obstructive shock. Tension pneumothorax is characterized by absent breath sounds and
hyperresonance to percussion as a result of the increased air in the pleural space displacing normal lung parenchyma along with
hemodynamic instability. In cases of acute tension pneumothorax, evacuating the accumulated air via needle decompression and
aspiration permits expansion of the lung and eliminates tension on the mediastinum. Immediate needle thoracostomy for
decompression is followed by tube thoracostomy, which prevents re-accumulation and promotes repair of the pleural defect.

Incorrect Answers: A, B, D, and E.

Chest x-ray (Choice A) and CT scan of the chest (Choice B) are both diagnostic imaging modalities that can disclose a traumatic
pneumothorax or hemothorax, rib fractures, pulmonary contusions, or other injuries. However, in patients who are hemodynamically
unstable, management should precede diagnostic imaging. In this case, a tension pneumothorax is a clinical diagnosis that should be
treated prior to further diagnostic imaging.

Tracheal intubation and mechanical venti lation (Choice D) would not be indicated in this patient prior to emergent needle thoracostomy.
Intubation wou ld increase positive pressure and air entry into the pleural space, increasing the severity of mediastinal deviation,
hypotension, and obstructive shock. A thoracostomy must be performed to relieve the tension pneumothorax and allow air to escape
the pleural space. ....

r ~, ~ ~ r,
Previous Next Score Report
https://t.me/USMLENBME2CK Lab Values Calculator Help Pause
Exam Section: Item 24 of 50 National Board of Medical Examiners
■ Mark Emergency Medicine Self-Assessment
...
C) Needle thoracostomYi
D) Tracheal intu bation and mechanical ventilation
E) Transfusion of packed red blood cells
Correct Answer: C.

Tension pneumothorax can be a complication of a penetrating thoracic trauma and occurs when air is able to enter the pleural space
but is prevented from exiting. Increasing amounts of trapped air in the pleural space compresses the lung parenchyma, creating an
inability for the lung to expand and causing it to collapse. As the pneumothorax progresses and continues to increase in size, it can
resu lt in tracheal or mediastinal deviation away from the affected hemithorax. It can also result in decreased venous return to the heart,
which manifests with hypotension and obstructive shock. Tension pneumothorax is characterized by absent breath sounds and
hyperresonance to percussion as a result of the increased air in the pleural space displacing normal lung parenchyma along with
hemodynamic instability. In cases of acute tension pneumothorax, evacuating the accumulated air via needle decompression and
aspiration permits expansion of the lung and eliminates tension on the mediastinum. Immediate needle thoracostomy for
decompression is followed by tube thoracostomy, which prevents re-accumulation and promotes repair of the pleural defect.

Incorrect Answers: A, B, D, and E.

Chest x-ray (Choice A) and CT scan of the chest (Choice B) are both diagnostic imaging modalities that can disclose a traumatic
pneumothorax or hemothorax, rib fractures, pulmonary contusions, or other injuries. However, in patients who are hemodynamically
unstable, management should precede diagnostic imaging. In this case, a tension pneumothorax is a clinical diagnosis that should be
treated prior to further diagnostic imaging.

Tracheal intubation and mechanical venti lation (Choice D) would not be indicated in this patient prior to emergent needle thoracostomy.
Intubation wou ld increase positive pressure and air entry into the pleural space, increasing the severity of mediastinal deviation,
hypotension, and obstructive shock. A thoracostomy must be performed to relieve the tension pneumothorax and allow air to escape
the pleural space.

Transfusion of packed red blood cel ls (Choice E) wou ld be indicated in the setting of hemorrhagic shock. However, the cause of this
patient's shock is obstructive secondary to a tension pneumothorax preventing venous return to the heart.

Educational Objective: Tension pneumothorax occurs when air is able to enter the pleural space but is prevented from exiting. This
leads to an increasing amount of air trapped in the pleural space with subsequent collapse of the lung parenchyma and mass effect on
the mediastinum. Management involves emergent needle decompression followed by tube thoracostomy.

.,.

r ~, ~ ~ r,
Previous Next Score Report
https://t.me/USMLENBME2CK Lab Values Calculator Help Pause
Exam Section: Item 25 of 50 National Board of Medical Examiners
■ Mark Emergency Medicine Self-Assessment

25. A 78-year-old man is brought to the emergency department by paramed ics 30 minutes after a friend found him confused and disoriented in his
home. The patient lives alone and appeared normal when the friend last saw him 3 days ago. He has type 1 diabetes mellitus and
hypercholesterolemia treated with insulin and simvastatin. His friend says the patient drinks alcohol ic beverages social ly and has smoked one-half
pack of cigarettes daily for the past 35 years. He has no known medication allergies. The patient appears drowsy but is easily aroused and follows
commands. His temperature is 37.6°C (99.7°C), pulse is 118/min, respirations are 18/min, and blood pressure is 92/54 mm Hg. Pulse oximetry on
room air shows an oxygen saturation of 97%. Physical examination shows decreased skin turgor, dry mucous membranes, and flat neck veins.
Laboratory studies show:
Hemoglobin 15.8 g/dL (158 g/L)
Hematocrit 46%
Leukocyte count 9,200/mm 3 (9.2 x 109/L)
Platelet count 227,000/mm 3 (227 x 109/L)
Serum
Na + 143 mEq/L (143 mmol/L)
CI- 104 mEq/L (104 mmol/L)
K+ 3.9 mEq/L (3.9 mmol/L)
HC0 3- 26 mEq/L (26 mmol/L)
Ca 2+ 11.6 mg/dL (2.9 mmol/L)
Urea nitrogen 68 mg/dL (24.3 mmol/L)
Glucose 562 mg/dL (31.19 mmol/L)
Creatin ine 3.6 mg/dL (318.24 µmol/L)

A basic metabolic profile obtained 3 months ago was within the reference ranges. Which of the following is the most appropriate next step in
management?

A) Administration of insulin
B) Administration of metoprolol
C) Administration of 0.9% saline
D) CT scan of the head
E) Hemodialysis
F) Ultrasonography of the kidneys and bladder
Correct Answer: C.

This patient's laboratory panel suggests severe dehydration as the primary aberrancy driving his clinical picture. Features of severe ....

r ~, ~ ~ r,
Previous Next Score Report
https://t.me/USMLENBME2CK Lab Values Calculator Help Pause
Exam Section: Item 25 of 50 National Board of Medical Examiners
■ Mark Emergency Medicine Self-Assessment
Correct Answer: C. ...

This patient's laboratory panel suggests severe dehydration as the primary aberrancy driving his clinical picture. Features of severe
dehydration include hypotension, tachycardia, decreased skin turgor, dry mucous membranes, and flat neck veins. His severely
increased creatinine concentrations suggests acute prerenal kidney injury, which is caused by ineffective perfusion of the kidneys,
whether caused by hypovolemia, as in this case, or hypotension. His hyperglycemia also suggests a high serum osmolality induced by
dehydration as does his hypercalcemia. Treatment of dehydration and prerenal kidney injury is with intravenous isotonic fluids, such as
0.9% saline. This will expand his intravascular volume and improve perfusion to the kidneys. However, if decreased perfusion to the
kidneys persists to the point of ischemia, then acute tubular necrosis, a form of intrinsic kidney injury, can develop. Prerenal and
intrinsic kidney injury can be distinguished by the fractional excretion of sodium, which is <1 % in prerenal and >2% in intrinsic kidney
inJury.

Incorrect Answers: A, B, D, E, and F.

Administration of insulin (Choice A) is not the best first step as the primary cause of his hyperglycemia is dehydration. Administering
insulin would be critical if the patient were demonstrating signs of diabetic ketoacidosis. However, he has a normal serum bicarbonate
and anion gap at this time. Thus, addressing his dehydration will likely resolve the majority of his hyperglycemia and should be the first
step in management.

Administration of metoprolol (Choice B) is not appropriate in this case as the patient is tachycardic as a result of dehydration, not
because of a primary arrhythmia. The patient's decreased intravascular volume and consequent decreased stroke volume require
tachycardia to maintain cardiac output and hemodynamic stability. If metoprolol were administered, his heart rate would decrease, and
he would likely begin to suffer further complications of inadequate perfusion and end-organ ischemia.

CT scan of the head (Choice D) may be warranted as a component of this patient's work-up but is not the most appropriate first step in
management. If the patient is still disoriented after correcting his metabolic disturbances, then a CT scan of the head is warranted.

Hemodialysis (Choice E) is indicated when acute kidney injury causes severe acidemia, electrolyte disturbances such as hyperkalemia,
volume overload, or uremia. None of these are present in this case. This patient's prerenal kidney injury will likely correct with
intravenous fluid administration.

Ultrasonography of the kidneys and bladder (Choice F) is indicated in patients whose acute kidney injury is suspected to be caused by
postrenal obstruction causing hydronephrosis, such as nephrolithiasis or an enlarged prostate. This patient's clinical and laboratory
picture are more consistent with a prerenal cause.

Educational Objective: Acute kidney injury, defined as a rapid decrease in renal function, may be caused by prerenal, intrinsic, or
postrenal causes. Dehydration is a common cause of prerenal kidney injury and should be first treated with intravenous isotonic fluids,
such as 0.9% saline. ....

r ~, ~ ~ r,
Previous Next Score Report
https://t.me/USMLENBME2CK Lab Values Calculator Help Pause
Exam Section: Item 26 of 50 National Board of Medical Examiners
■ Mark Emergency Medicine Self-Assessment

26. A 36-year-old woman with a 15-year history of intravenous drug use is brought to the emergency department because of a 4-day history of
progressive redness and pain of her left arm at the site of a recent injection of heroin. Her tem peratu re is 39°C (102.2°F), pulse is 110/min,
respirations are 24/mi n, and blood pressure is 100/58 mm Hg. Physical exami nation shows a 10 x 10-cm area of erythema , warmth , and
tenderness of the left antecubital fossa. Multiple attempts at peripheral intravenous access are unsuccessful, and she requ ires placement of a
central venous catheter. Which of the following is necessary to discuss with the patient in order to obtai n informed consent for this procedure?

A) Advantages of using a subclavian catheter over a femoral catheter


B) Anticipated length of time intravenous access will be needed
C) Number of prior central venous catheters placed by the physician
D) Risk of pneumothorax associated with placement of a subclavian catheter.
E) Informed consent is not needed in th is situation
Correct Answer: D.

Ensuring informed consent has been obtained is a physician's duty before performing a procedure in a non-emergent setting. Informed
consent fulfills the ethics principle of autonomy, which dictates that patients should be able to make their own decisions about their
healthcare, including whether to accept or decline a procedure. Informed consent also benefits the physician-patient relationship and
encourages physicians to thoroughly review the indications, benefits, and risks of the procedure, and whether alternatives could be
considered. Informed consent in this case must include an explanation of the proposed procedure, alternative options, the risks and
benefits of the procedure, and the risk of refusing the procedure. In this case, the informed consent process should include discussion
of the risk of pneumothorax associated with placement of a subclavian catheter.

Incorrect Answers: A, B, C, and E.

Advantages of using a subclavian catheter over a femoral catheter (Choice A) shou ld be considered by the physician when determining
the appropriate management of this patient but does not need to be described to the patient as part of the informed consent process.

Anticipated length of time intravenous access will be needed (Choice B) is not likely known at this point in the patient's course and does
not need to be included in the informed consent.

Number of prior central venous catheters placed by the physician (Choice C) is not necessary to disclose as part of the informed
consent process. However, it is necessary to disclose to the patient whether trainees such as residents and medical students will
participate in the procedure.

Informed consent is necessary before any procedure occurs in a non-emergent setting. Thus, it is required in this situation (Choice E) ....
- - - - - - - - - - -- - - - - - - - - - - -- --- --- -- - - -- - -- -- - - --
r ~, ~ ~ r,
Previous Next Score Report
https://t.me/USMLENBME2CK Lab Values Calculator Help Pause
Exam Section: Item 26 of 50 National Board of Medical Examiners
■ Mark Emergency Medicine Self-Assessment
- . -· . ' ' - - - ·· . - . -. - ,. -- - . - -- - - . - - . - .. ...
A) Advantages of using a subclavian catheter over a femoral catheter
B) Anticipated length of time intravenous access will be needed
C) Number of prior central venous catheters placed by the physician
D) Risk of pneumothorax associated with placement of a subclavian catheter.
E) Informed consent is not needed in th is situation
Correct Answer: D.

Ensuring informed consent has been obtained is a physician's duty before performing a procedure in a non-emergent setting. Informed
consent fulfills the ethics principle of autonomy, which dictates that patients should be able to make their own decisions about their
healthcare, including whether to accept or decline a procedure. Informed consent also benefits the physician-patient relationship and
encourages physicians to thoroughly review the indications, benefits, and risks of the procedure, and whether alternatives could be
considered. Informed consent in this case must include an explanation of the proposed procedure, alternative options, the risks and
benefits of the procedure, and the risk of refusing the procedure. In this case, the informed consent process should include discussion
of the risk of pneumothorax associated with placement of a subclavian catheter.

Incorrect Answers: A, B, C, and E.

Advantages of using a subclavian catheter over a femoral catheter (Choice A) should be considered by the physician when determining
the appropriate management of this patient but does not need to be described to the patient as part of the informed consent process.

Anticipated length of time intravenous access will be needed (Choice B) is not likely known at this point in the patient's course and does
not need to be included in the informed consent.

Number of prior central venous catheters placed by the physician (Choice C) is not necessary to disclose as part of the informed
consent process. However, it is necessary to disclose to the patient whether trainees such as residents and medical students will
participate in the procedure.

Informed consent is necessary before any procedure occurs in a non-emergent setting. Thus, it is required in this situation (Choice E)
and to forego informed consent would be negligent on the part of the physician.

Educational Objective: Informed consent fulfills the ethics principle of autonomy. When obtaining informed consent for a procedure, the
proposed procedure, alternative treatment options, the risks and benefits of the procedure, and the risk of refusing the procedure
should all be explained to the patient in a way that they can understand.

.,.

r ~, ~ ~ r,
Previous Next Score Report
https://t.me/USMLENBME2CK Lab Values Calculator Help Pause
Exam Section: Item 27 of 50 National Board of Medical Examiners
■ Mark Emergency Medicine Self-Assessment

27. A 49-year-old woman comes to the emergency department because of the sudden onset of rig ht wrist pain 5 hours ago, when she tripped, fell, and
landed on her outstretched right hand. She did not sustain any other injury. She has hypothyroidism treated with levothyroxine. She has no known
allergies. She does not drink alcoholic beverages, smoke cigarettes, or use illicit drugs. Vital signs are within normal limits. Examination of the rig ht
wrist shows mild diffuse swelling without gross deformity. There is tenderness over the radial aspect of the wrist including the snuffbox. Active and
passive range of motion is limited by pain. She is able to extend, flex, abduct, and adduct her fingers, and sensation is intact to light touch. X-rays
of the wrist are shown. Which of the following is the most appropriate next step in management?
A) Activity as tolerated
B) Arthrocentesis
C) Elastic bandage and ice packs for comfort
D) Fracture red uction
E) Thumb spica splint
Correct Answer: E.

This patient's history of wrist pain overlying the snuffbox following a fall on an outstretched hand, as wel l as her radiographic findings,
- • 81 •1 , I - -818 I - • 1• -118 I - II -1 . 8 - • - • - II 81 -1 8 • 1•1 1-11 a• - • 8

r ~, ~ ~ r,
Previous Next Score Report
https://t.me/USMLENBME2CK Lab Values Calculator Help Pause
Exam Section: Item 27 of 50 National Board of Medical Examiners
■ Mark
. - " -- ... '' . . ... .
- - . -·· .. - - - -.
Emergency Medicine Self-Assessment
- .. - -
A) Activity as tolerated
B) Arthrocentesis
C) Elastic bandage and ice packs for comfort
D) Fracture reduction
E) Thumb spica splint
-- Correct Answer: E.

This patient's history of wrist pain overlying the snuffbox following a fall on an outstretched hand, as well as her radiographic findings,
are consistent with a scaphoid fracture. The scaphoid is at significant risk for fracture after falling on an outstretched hand because of
its narrow and fragile central waist. Scaphoid fractures can have significant and long-lasting effects as a result of the risk for nonunion
and proximal osteonecrosis of the bone , which receives its vascular supply in a distal-to-proximal fashion. Patients with non-displaced
scaphoid fractures should undergo thumb spica splinting until union has been demonstrated with follow-up x-rays . Patients with
displaced fractures, carpal instability, or delayed presentation should be referred for surgical repair.

Incorrect Answers : A , B, C, and D.

Activity as tolerated (Choice A) prior to union of the fracture increases the patient's risk for non-union and of prolonged functional
deficits.

Arthrocentesis (Choice B) is useful for the diagnosis of septic or inflammatory arthritis. This patient's lack of fever and her presentation
immediately following a fall are more suggestive of a traumatic fracture of the scaphoid.

Elastic bandage and ice packs (Choice C) will result in insufficient restriction of mobility in comparison to a thumb spica splint and are
not recommended for the treatment of a scaphoid fracture.

Fracture reduction (Choice D) is recommended for displaced scaphoid fractures, carpal instability, or patients with a delayed
presentation. This patient's x-rays are more consistent with a non-displaced acute scaphoid fracture.

Educational Objective: The scaphoid is at significant risk for fracture after falling on an outstretched hand as a result of its narrow and
fragile central waist. Patients with non-displaced scaphoid fractures should undergo thumb spica splinting until union has been
demonstrated with follow-up x-rays.

...
r ~, ~ p r,
Previous Next Score Report
https://t.me/USMLENBME2CK Lab Values Calculator Help Pause
Exam Section: Item 28 of 50 National Board of Medical Examiners
■ Mark Emergency Medicine Self-Assessment

28. A 30-year-old woman is brought to the emergency department because of a 1-hour history of sharp, left-sided chest pain and shortness of breath .
She has not had fever, cough, nausea, or vomiting. Her symptoms started while she was at rest at home. She has hypercholesterolem ia but has
declined pharmacotherapy for this cond ition. Her only current medication is a combination oral contraceptive. She leads a sedentary lifestyle and
eats foods that have a high salt content. She has smoked one pack of cigarettes daily for 10 years. She does not use illicit drugs. BMI is 32 kg/m 2.
Her temperature is 36. 7°C (98°F), pulse is 110/min, respirations are 24/min, and blood pressure is 160/70 mm Hg. Pulse oximetry on room air
shows an oxygen saturation of 94%. Physical examination shows tenderness and edema of the left calf. Wh ich of the following is the strongest
predisposing factor for this patient's current condition?

A) High-sodium diet
B) Lack of exercise
C) Uncontrolled hypertension
D) Untreated hypercholesterolemia
E) Use of an oral contraceptive
Correct Answer: E.

Use of an oral contraceptive is the strongest risk factor for this patient's current condition, which is most consistent with a deep venous
thrombosis (DVT) of the left lower leg with likely embolization to the pulmonary arteries (pulmonary embolism). The cause of venous
thromboembolism (VTE) is considered provoked if it occurs after surgery, malignancy, or prolonged immobilization, but is considered
unprovoked in situations where there does not appear to be an inciting factor. While this distinction can sometimes be useful in
estimating the risk for subsequent VTE events, it is not always useful in clinical practice. Regardless of the cause, all patients should
receive chronic anticoagulant medications such as warfarin, low molecular weight heparin, or one of the direct oral anticoagulant
medications. Additionally, underlying risk factors for VTE should be addressed. This patient's cigarette use increases her risk for VTE,
but the use of oral contraceptives, especially in a patient who smokes cigarettes, confers a substantially higher risk for VTE. This
patient should be started on an anticoagulant medication and her oral contraceptives should be discontinued. Additionally, she should
receive counseling on tobacco cessation.

Incorrect Answers: A, B, C, and D.

High-sodium diet (Choice A), in addition to obesity, can contribute to the development of hypertension, which is apparent in this patient,
but a high-sodium diet does not predispose to VTE.

Lack of exercise (Choice B) is not a risk factor for VTE, although prolonged periods of immobility such as during international flights
may confer some increased risk.
....
--- - - -- - -- --- -- -- -- -- ----- -- - - -- - - - - - - - - -- --- - - - - - --- -- -- ---- -- - -
r ~, ~ ~ r,
Previous Next Score Report
https://t.me/USMLENBME2CK Lab Values Calculator Help Pause
Exam Section: Item 28 of 50 National Board of Medical Examiners
■ Mark Emergency Medicine Self-Assessment
i • ,:: • - •- -

C) Uncontrolled hypertension
D) Untreated hypercholesterolemia
E) Use of an oral contraceptive
Correct Answer: E.

Use of an oral contraceptive is the strongest risk factor for this patient's current condition, which is most consistent with a deep venous
thrombosis (DVT) of the left lower leg with likely embolization to the pulmonary arteries (pulmonary embolism). The cause of venous
thromboembolism (VTE) is considered provoked if it occurs after surgery, malignancy, or prolonged immobilization, but is considered
unprovoked in situations where there does not appear to be an inciting factor. While this distinction can sometimes be useful in
estimating the risk for subsequent VTE events, it is not always useful in clinical practice. Regardless of the cause, all patients should
receive chronic anticoagulant medications such as warfarin, low molecular weight heparin, or one of the direct oral anticoagulant
medications. Additionally, underlying risk factors for VTE should be addressed. This patient's cigarette use increases her risk for VTE,
but the use of oral contraceptives, especially in a patient who smokes cigarettes, confers a substantially higher risk for VTE. This
patient should be started on an anticoagulant medication and her oral contraceptives should be discontinued. Additionally, she should
receive counseling on tobacco cessation.

Incorrect Answers: A, B, C, and D.

High-sodium diet (Choice A), in addition to obesity, can contribute to the development of hypertension, which is apparent in this patient,
but a high-sodium diet does not predispose to VTE.

Lack of exercise (Choice B) is not a risk factor for VTE, although prolonged periods of immobility such as during international flights
may confer some increased risk.

Uncontrolled hypertension (Choice C) increases the risk for myocardial infarction and stroke but does not increase the risk for VTE.

Untreated hypercholesterolemia (Choice D) increases the risk for coronary artery disease and myocardial infarction but does not
increase the risk for VTE.

Educational Objective: Patients who develop signs or symptoms of a VTE should be treated with anticoagulant medications and
assessed for the presence of risk factors. The use of an oral contraceptive, especially in a patient who also smokes cigarettes,
substantially increases the risk for VTE.

.,.

r ~, ~ ~ r,
Previous Next Score Report
https://t.me/USMLENBME2CK Lab Values Calculator Help Pause
Exam Section: Item 29 of 50 National Board of Medical Examiners
■ Mark Emergency Medicine Self-Assessment

29. A previously healthy 28-year-old woman comes to the emergency department


2 days after boiling water splashed on her left foot while she was brewing beer
and caused a painful red area on her toes. Five minutes after the incident, a
blister developed on one portion of the area. She has no other symptoms. She
takes no medications. Her on ly allergy is to wasp stings. Her temperature is 37°C
(98.6°F), pu lse is 60/min, respirations are 14/min, and blood pressure is
120/60 mm Hg. Pulse oximetry on room air shows an oxygen satu ration of 100%.
A photograph of the foot is shown. Which of the following is the most likely
diagnosis?
A) Burn wound infection
B) Contact dermatitis
C) Full-thickness burn
D) Partial-thickness burn
E) Superficial-thickness burn
Correct Answer: D.

This patient's physical examination findings suggest a diagnosis of


partial-thickness (second-degree) burn. Burns may be caused by
heat, as in this case, electrical discharge, chemicals, radiation, or
friction, and are classified based on their depth. When heat is the
cause of a burn, the depth of injury is related to the temperature and
the duration of contact with the causative agent. A superficial (first-
degree) burn is characterized by dry, blanchable erythema, which is
painful and takes several days to heal. A partial-thickness (second-
degree) burn demonstrates blisters and redness, which may weep
serous fluid. Partial-thickness burns are typical ly painful to
temperature, air, and touch, and take 1 to 3 weeks to heal. A full-
thickness (third-degree) burn may be white, gray, or black, and have a
leathery or charred appearance. It does not blanch with pressure and
only deep pressure causes pain. Full-thickness burns do not typically
heal unless surgical ly treated. Treatment of this partial-thickness
wound should include pain management with acetaminophen or
nonsteroidal anti-inflammatory drugs, cleaning and debridement of ....

r ~, ~ ~ r,
Previous Next Score Report
https://t.me/USMLENBME2CK Lab Values Calculator Help Pause
Exam Section : Item 29 of 50 National Board of Medical Examiners
■ Mark Emergency Medicine Self-Assessment

Correct Answer: D.

This patient's physical examination findings suggest a diagnosis of


partial-thickness (second-degree) burn. Burns may be caused by
heat, as in this case, electrical discharge, chemicals, radiation, or
friction, and are classified based on their depth. When heat is the
cause of a burn, the depth of injury is related to the temperature and
the duration of contact with the causative agent. A superficial (first-
degree) burn is characterized by dry, blanchable erythema, which is
painful and takes several days to heal. A partial-thickness (second-
degree) burn demonstrates blisters and redness, which may weep
serous fluid. Partial-thickness burns are typical ly painful to
temperature, air, and touch, and take 1 to 3 weeks to heal. A full-
thickness (third-degree) burn may be white, gray, or black, and have a
leathery or charred appearance. It does not blanch with pressure and
only deep pressure causes pain. Full-thickness burns do not typically
heal unless surgical ly treated. Treatment of this partial-thickness
wound should include pain management with acetaminophen or
nonsteroidal anti-inflammatory drugs, cleaning and debridement of
ruptured blisters (though any intact blisters should generally not be
ruptured intentionally), and basic dressing of the area with a layer of
nonadherent gauze fol lowed by dry gauze. Topical antibiotics should
generally be employed to prevent secondary infection (eg, si lver
sulfadiazine, bacitracin).

Incorrect Answers: A, B, C, and E.

This examination does not demonstrate signs of burn wound infection


(Choice A). Ful l thickness burns are at high risk for infection from both
common and opportunistic organisms, but partial-thickness burns
have less risk.

Contact dermatitis (Choice B), when acute and severe, may cause
small vesicles to develop in the distribution of exposure to the
allergen. This patient's erythema and bu llae have a sharp cut-off
where her shoe protected her foot. The combination of this finding
and the history of a thermal burn make contact dermatitis unlikely.
....

r ~, ~ ~ r,
Previous Next Score Report
https://t.me/USMLENBME2CK Lab Values Calculator Help Pause
Exam Section: Item 29 of 50 National Board of Medical Examiners
■ Mark Emergency Medicine Self-Assessment

thickness (third-degree) burn may be white, gray, or black, and have a ...
leathery or charred appearance. It does not blanch with pressure and
only deep pressure causes pain. Full-thickness burns do not typically
heal unless surgical ly treated. Treatment of this partial-thickness
wound should include pain management with acetaminophen or
nonsteroidal anti-inflammatory drugs, cleaning and debridement of
ruptured blisters (though any intact blisters should generally not be
ruptured intentionally), and basic dressing of the area with a layer of
nonadherent gauze fol lowed by dry gauze. Topical antibiotics should
generally be employed to prevent secondary infection (eg, si lver
sulfadiazine, bacitracin).

Incorrect Answers: A, B, C, and E.

This examination does not demonstrate signs of burn wound infection


(Choice A). Ful l thickness burns are at high risk for infection from both
common and opportunistic organisms, but partial-thickness burns
have less risk.

Contact dermatitis (Choice B), when acute and severe, may cause
small vesicles to develop in the distribution of exposure to the
allergen. This patient's erythema and bu llae have a sharp cut-off
where her shoe protected her foot. The combination of this finding
and the history of a thermal burn make contact dermatitis unlikely.

Full-thickness burn (Choice C) causes a white, gray, or black


appearance. Superficial-thickness burn (Choice E) causes painful,
dry, blanchable erythema without blisters. This patient's examination
demonstrates erythema and bullae, consistent with a partial-thickness
burn.

Educational Objective: Thermal burns are classified by the depth of


injury. A superficial (first-degree) burn is characterized by painful, dry,
blanchable erythema. A partial-thickness (second-degree) burn
demonstrates blisters and redness, which may weep serous fluid. A
full-thickness (third-degree) burn may be white, gray, or black, and
have a leathery or charred appearance.
....

r ~, ~ ~ r,
Previous Next Score Report
https://t.me/USMLENBME2CK Lab Values Calculator Help Pause
Exam Section: Item 30 of 50 National Board of Medical Examiners
■ Mark Emergency Medicine Self-Assessment

30. A 44-year-old woman who is a firefighter is brought to the emergency department by paramed ics because of smoke inhalation. She was fig hting a
house fire when her personal protective equipment failed approximately 15 minutes ago. She has no history of serious illness and takes no
med ications. Her temperatu re is 37°C (98.6°F), pulse is 115/min, respirations are 24/min, and blood pressure is 110/50 mm Hg. Pulse oximetry
wh ile breathing 100% oxygen via nonrebreather mask shows an oxygen saturation of 100%. Physical exam ination shows soot in the airway.
Auscu ltation of the neck and lungs discloses strider on inspiration. In add ition to continued oxygen admin istration, wh ich of the following is the
most appropriate next step in management?

A) Administration of amyl nitrite


B) Administration of hydroxocobalamin
C) Arterial blood gas analysis
D) Endotracheal intubation
E) X-ray of the chest
Correct Answer: D.

Endotracheal intubation is the most appropriate next step in management of the patient's airway in the setting of acute smoke
inhalation injury. Smoke inhalation injury should be suspected in patients presenting with facial burns, hoarse voice, wheezing, soot in
the airway, strider, or carbonaceous sputum. The inhalation of smoke and additional chemicals in the setting of a fire presents the risk
for injury to the pharyngeal, laryngeal, and tracheal mucosa. Resultant inflammation drives the rapid development of mucosa! edema,
which can cause acute airway obstruction. Airway obstruction is an immediate life-threatening condition that must be managed at or
before the time it occurs. If care is not taken to immediately maintain an unobstructed path for air to flow into the tracheobronchial tree,
rapid hypoxia, hypercapnia, and death can occur if significant airway edema ensues. Emergent endotracheal intubation is the most
appropriate initial step and shou ld be performed immediately to prevent airway obstruction.

Incorrect Answers: A, B, C, and E.

Cyanide inhibits the electron transport chain, leading to lactic acidosis, compensatory respiratory alkalosis, hypotension, lethargy, and
bradycardia. The management of cyanide poisoning includes supportive care with intravenous fluids and vasopressors, sodium
thiosulfate, which increases the conversion of cyanide to thiocyanate for renal excretion, amyl nitrites (Choice A) to induce
methemoglobinemia, which binds with cyanide and prevents it from acting on the electron transport chain, and hydroxocobalamin
(Choice B), which combines with cyanide to create nontoxic cyanocobalamin. This patient may require these interventions but not prior
to securing the airway.

Arterial blood gas analysis (Choice C) and an x-ray of the chest (Choice E) are not helpfu l at this time to make further decisions about
management. This patient is at risk for imminent airway obstruction as a result of signs of smoke inhalation injury and strider on ....
--- - -- --- - - -- - - - - -- - - - - - -- - - - - - -- -- - ---- ----- - -- --
r ~, ~ ~ r,
Previous Next Score Report
https://t.me/USMLENBME2CK Lab Values Calculator Help Pause
Exam Section: Item 30 of 50 National Board of Medical Examiners
■ Mark Emergency Medicine Self-Assessment
...
A) Administration of amyl nitrite
B) Administration of hyd roxocobalamin
C) Arterial blood gas analysis
D) Endotracheal intubation
E) X-ray of the chest
Correct Answer: D.

Endotracheal intubation is the most appropriate next step in management of the patient's airway in the setting of acute smoke
inhalation injury. Smoke inhalation injury should be suspected in patients presenting with facial burns, hoarse voice, wheezing, soot in
the airway, strider, or carbonaceous sputum. The inhalation of smoke and additional chemicals in the setting of a fire presents the risk
for injury to the pharyngeal, laryngeal, and tracheal mucosa. Resultant inflammation drives the rapid development of mucosal edema,
which can cause acute airway obstruction. Airway obstruction is an immediate life-threatening condition that must be managed at or
before the time it occurs. If care is not taken to immediately maintain an unobstructed path for air to flow into the tracheobronchial tree,
rapid hypoxia, hypercapnia, and death can occur if significant airway edema ensues. Emergent endotracheal intubation is the most
appropriate initial step and shou ld be performed immediately to prevent airway obstruction.

Incorrect Answers: A, B, C, and E.

Cyanide inhibits the electron transport chain, leading to lactic acidosis, compensatory respiratory alkalosis, hypotension, lethargy, and
bradycardia. The management of cyanide poisoning includes supportive care with intravenous fluids and vasopressors, sodium
thiosulfate, which increases the conversion of cyanide to thiocyanate for renal excretion, amyl nitrites (Choice A) to induce
methemoglobinemia, which binds with cyanide and prevents it from acting on the electron transport chain, and hydroxocobalamin
(Choice B), which combines with cyanide to create nontoxic cyanocobalamin. This patient may require these interventions but not prior
to securing the airway.

Arterial blood gas analysis (Choice C) and an x-ray of the chest (Choice E) are not helpfu l at this time to make further decisions about
management. This patient is at risk for imminent airway obstruction as a result of signs of smoke inhalation injury and strider on
inspiration, and the next step in management of this patient is endotracheal intubation.

Educational Objective: Smoke inhalation injury should be suspected in patients presenting with facial burns, hoarse voice, wheezing,
soot in the airway, strider, or carbonaceous sputum. Emergent endotracheal intubation is the most appropriate initial step in
management and should be performed immediately to prevent airway obstruction.

.,.

r ~, ~ ~ r,
Previous Next Score Report
https://t.me/USMLENBME2CK Lab Values Calculator Help Pause
Exam Section : Item 31 of 50 National Board of Medical Examiners
■ Mark Emergency Medicine Self-Assessment

31. A previously healthy 23-year-old man comes to the emergency department because of fever,
muscle aches, pain in both of his knees, pain in his left forearm and wrist, and a diffuse rash . His
current temperature is 38.2°C (100.8°F), pulse is 90/min, respirations are 16/min, and blood
pressure is 115/75 mm Hg. Examination shows effusion of the knees bilaterally and a diffuse
rash. A photograph of one of the lesions is shown. Which of the following is the most likely causal
pathogen?
A) Chlamydia trachomatis
B) Herpes simplex virus
C) Molluscum contagiosum virus
D) Neisseria gonorrhoeae
E) Treponema pallidum
Correct Answer: D.

Infection of the genital and urinary tracts with Neisseria gonorrhoeae may lead to
bacteremia and disseminated gonococcal infection. Disseminated gonococcal
infection presents with signs and symptoms including fever, chil ls, septic polyarthritis,
tenosynovitis, and dermatitis characterized by a pustular rash. Symptoms are
typically acute in onset and may follow genitourinary infectious symptoms such as
dysuria, pruritus, or mucopuru lent discharge. The affected joints display erythema,
swelling, and warmth. Arthrocentesis and synovial fluid analysis may show Gram-
negative diplococci and a markedly increased concentration of neutrophi ls. N.
gonorrhoeae may be isolated from genital swabs or from blood cultures. Treatment
includes antibiotic therapy with intravenous ceftriaxone. Recently, treatment has
been complicated by the emergence of multidrug resistant strains of N. gonorrhoeae.

Incorrect Answers: A, B, C, and E.

Chlamydia trachomatis (Choice A) is a common sexually transmitted infection that


can cause cervicitis, pelvic inflammatory disease, epididymitis, and urethritis. C.
trachomatis does not typically cause septic arthritis or a disseminated rash.

Herpes simplex virus (Choice B) can cause herpetic whitlow, which presents with
painful, non-purulent vesicles that occur on the fingers. Herpetic whitlow typically ....

r ~, ~ ~ r,
Previous Next Score Report
https://t.me/USMLENBME2CK Lab Values Calculator Help Pause
Exam Section : Item 31 of 50 National Board of Medical Examiners
■ Mark Emergency Medicine Self-Assessment
...
Correct Answer: D.

Infection of the genital and urinary tracts with Neisseria gonorrhoeae may lead to
bacteremia and disseminated gonococcal infection. Disseminated gonococcal
infection presents with signs and symptoms including fever, chills, septic polyarthritis,
tenosynovitis, and dermatitis characterized by a pustular rash . Symptoms are
typically acute in onset and may follow genitourinary infectious symptoms such as
dysuria, pruritus, or mucopurulent discharge. The affected joints display erythema,
swelling, and warmth. Arthrocentesis and synovial fluid analysis may show Gram-
negative diplococci and a markedly increased concentration of neutrophils. N.
gonorrhoeae may be isolated from genital swabs or from blood cultures. Treatment
includes antibiotic therapy with intravenous ceftriaxone. Recently, treatment has
been complicated by the emergence of multidrug resistant strains of N. gonorrhoeae.

Incorrect Answers: A, B, C, and E.

Chlamydia trachomatis (Choice A) is a common sexually transm itted infection that


can cause cervicitis, pelvic inflammatory disease, epididymitis, and urethritis. C.
trachomatis does not typically cause septic arthritis or a disseminated rash.

Herpes simplex virus (Choice B) can cause herpetic whitlow, which presents with
painful, non-purulent vesicles that occur on the fingers. Herpetic whitlow typically
occurs in dentists as a result of oral-cutaneous inoculation. It is not associated with a
diffuse rash, tenosynovitis, or polyarthritis.

Molluscum contagiosum (Choice C) refers to a viral infection of the skin caused by


poxvirus. Lesions are characteristically round, smooth, and exhibit a central
umbilication. It is transm itted through close contact with infected individuals and thus,
is commonly seen in multiple children within a family or in children who play contact
sports. Molluscum contagiosum infection does not typically lead to tenosynovitis or
polyarthritis.

Treponema pal/idum (Choice E) is the causative spirochete bacterium of syphilis,


which presents in multiple stages with varying symptoms, including primary with a
painless chancre, secondary with fever, lymphadenopathy, and condylomata lata,
and tertiary with tabes dorsalis, aortitis, and gummas. The rash of secondary syphilis
is characterized by scaly, light brown macules and patches.
....

r ~, ~ ~ r,
Previous Next Score Report
https://t.me/USMLENBME2CK Lab Values Calculator Help Pause
Exam Section : Item 31 of 50 National Board of Medical Examiners
■ Mark Emergency Medicine Self-Assessment

y ' •• • •y p y p
typically acute in onset and may follow genitourinary infectious symptoms such as
dysuria, pruritus, or mucopurulent discharge. The affected joints display erythema,
swelling, and warmth. Arthrocentesis and synovial fluid analysis may show Gram-
negative diplococci and a markedly increased concentration of neutrophils. N.
gonorrhoeae may be isolated from genital swabs or from blood cultures. Treatment
includes antibiotic therapy with intravenous ceftriaxone. Recently, treatment has
been complicated by the emergence of multidrug resistant strains of N. gonorrhoeae.

Incorrect Answers: A, B, C, and E.

Chlamydia trachomatis (Choice A) is a common sexually transmitted infection that


can cause cervicitis, pelvic inflammatory disease, epididymitis, and urethritis. C.
trachomatis does not typically cause septic arthritis or a disseminated rash.

Herpes simplex virus (Choice B) can cause herpetic whitlow, which presents with
painful, non-purulent vesicles that occur on the fingers. Herpetic whitlow typically
occurs in dentists as a result of oral-cutaneous inoculation. It is not associated with a
diffuse rash, tenosynovitis, or polyarthritis.

Molluscum contagiosum (Choice C) refers to a viral infection of the skin caused by


poxvirus. Lesions are characteristically round, smooth, and exhibit a central
umbilication. It is transmitted through close contact with infected individuals and thus,
is commonly seen in multiple children within a family or in children who play contact
sports. Molluscum contagiosum infection does not typically lead to tenosynovitis or
polyarthritis.

Treponema pal/idum (Choice E) is the causative spirochete bacterium of syphilis,


which presents in multiple stages with varying symptoms, including primary with a
painless chancre, secondary with fever, lymphadenopathy, and condylomata lata,
and tertiary with tabes dorsalis, aortitis, and gummas. The rash of secondary syphilis
is characterized by scaly, light brown macules and patches.

Educational Objective: Disseminated gonococcal infection presents with signs and


symptoms including fever, chills, septic polyarthritis, tenosynovitis, and dermatitis
characterized by a pustular rash. Treatment includes antibiotic therapy with
intravenous ceftriaxone.
....

r ~, ~ ~ r,
Previous Next Score Report
https://t.me/USMLENBME2CK Lab Values Calculator Help Pause
Exam Section: Item 32 of 50 National Board of Medical Examiners
■ Mark Emergency Medicine Self-Assessment

32. A 41-year-old man who works as a research assistant comes to the emergency department 15 minutes after pricking himself with a visibly
contaminated need le left in an examination room . He was wearing gloves at the time and washed his hands with soap and water immediately
afterward. He is unsure when he received his last tetanus booster. He was vaccinated for hepatitis B 2 years ago. Medical history is unremarkable.
Vital signs are within normal limits . Physical examination shows a small puncture wound consistent with a needlestick injury; the wound is not
bleeding. The patient is concerned about transmissible diseases and , after a discussion of the risks and benefits, requests HIV prophylaxis. This
patient shou ld also receive which of the following?

A) Amoxicillin/clavulanate on ly
B) Hepatitis B immune globulin (HBIG) and amoxicillin/clavulanate
C) HBIG only
D) Tetanus booster and amoxicillin/clavulanate
E) Tetanus booster on lYi
F) Tetanus booster and HBIG
G) Tetanus booster, HBIG, and amoxicillin/clavulanate
H) No additional treatment is necessary at this time
Correct Answer: E.

Needlestick injuries are a common occupational hazard for healthcare workers. These injuries put the worker at risk for inoculation of
bloodborne pathogens and subsequent infection with hepatitis B, hepatitis C, and HIV. Needlestick injuries also put the injured worker
at risk for tetanus infection. Following such an injury, the worker should be asked about their previous vaccinations (including hepatitis
Band tetanus) and should be counseled on the risks and benefits of HIV prophylaxis. If the patient had not received his hepatitis B
vaccine series, he should be treated with vaccination as well as with hepatitis B immune globulin (HBIG). If a patient has received their
tetanus vaccine, they shou ld receive a tetanus booster if their last dose was more than 10 years ago for a clean or minor wound or if
there has been more than 5 years ago for all other wounds. If they have not received their vaccination for tetanus, they shou ld receive
the tetanus immune globulin only for large or dirty wounds. This patient has been vaccinated against hepatitis B, making HBIG
unnecessary, and has been vaccinated against tetanus. Since he is unsure when his last tetanus vaccination was performed, he
should receive a tetanus booster.

Incorrect Answers: A, B, C, D, F, G, and H.

Treatment with amoxici llin/clavulanate (Choices A, B, D, and G) is utilized in the management of skin and soft tissue infections, such as
those sustained from a bite wound. As this was an overall clean wound without inoculation of the joint capsu le, antibiotic therapy is not
warranted unless the patient develops signs and symptoms of infection. ....

r ~, ~ ~ r,
Previous Next Score Report
https://t.me/USMLENBME2CK Lab Values Calculator Help Pause
Exam Section : Item 32 of 50 National Board of Medical Examiners
■ Mark Emergency Medicine Self-Assessment
A .. , I • • • A • , J,

F) Tetanus booster and HBIG


G) Tetanus booster, HBIG, and amoxicillin/clavulanate
H) No additional treatment is necessary at this time
Correct Answer: E.

Needlestick injuries are a common occupational hazard for healthcare workers. These injuries put the worker at risk for inoculation of
bloodborne pathogens and subsequent infection with hepatitis B, hepatitis C, and HIV. Needlestick injuries also put the injured worker
at risk for tetanus infection. Following such an injury, the worker should be asked about their previous vaccinations (including hepatitis
Band tetanus) and should be counseled on the risks and benefits of HIV prophylaxis. If the patient had not received his hepatitis B
vaccine series, he should be treated with vaccination as well as with hepatitis B immune globulin (HBIG). If a patient has received their
tetanus vaccine, they should receive a tetanus booster if their last dose was more than 10 years ago for a clean or minor wound or if
there has been more than 5 years ago for all other wounds. If they have not received their vaccination for tetanus, they should receive
the tetanus immune globulin only for large or dirty wounds. This patient has been vaccinated against hepatitis B, making HBIG
unnecessary, and has been vaccinated against tetanus. Since he is unsure when his last tetanus vaccination was performed, he
should receive a tetanus booster.

Incorrect Answers: A, B, C, D, F, G, and H.

Treatment with amoxicillin/clavulanate (Choices A, B, D, and G) is utilized in the management of skin and soft tissue infections, such as
those sustained from a bite wound. As this was an overall clean wound without inoculation of the joint capsule, antibiotic therapy is not
warranted unless the patient develops signs and symptoms of infection.

Hepatitis B immune globulin (Choices B, C, F, and G) is not warranted, as this patient has been previously vaccinated against hepatitis
B.

No additional treatment is necessary at this time (Choice H) is incorrect, as this patient could be at risk for the development of tetanus
given the unknown date of his last tetanus vaccination. Failure to prophylactically immunize this patient could lead to tetanus infection
and unnecessary morbidity and mortality.

Educational Objective: Needlestick injuries are common in healthcare, and those affected should be asked about their vaccination
history, including hepatitis B and tetanus, as well as counseled about the risks and benefits of HIV prophylaxis. If the patient has not
been immunized against hepatitis B, he or she should receive the hepatitis B vaccine series and HBIG. The patient should also receive
a tetanus booster if it has been more than 5 years since the last dose for a dirty wound or more than 10 years since the last dose for a
clean wound.

.,.

r ~, ~ ~ r,
Previous Next Score Report
https://t.me/USMLENBME2CK Lab Values Calculator Help Pause
Exam Section: Item 33 of 50 National Board of Medical Examiners
■ Mark Emergency Medicine Self-Assessment
....

33. A 55-year-old man is brought to the emergency department (ED) because of a 1-day history of subjective fever and altered mental status. He was
seen in the ED 3 days ago because of painful joint swelling . At that time, he was presumptively diagnosed with gouty arthritis and discharged with
a prescription for indomethacin. He has no history of serious illness and takes no other medications. The patient is alert but confused. He is
oriented to person only. His temperature is 38.5°C (101.3°F), pulse is 116/min , respirations are 23/min, and blood pressure is 87/54 mm Hg. Pulse
oximetry on room air shows an oxygen saturation of 98%. The skin is cool and clammy. The left ankle is swollen, erythematous, and warm.
Movement of the ankle elicits pain. The remainder of the physical examination shows no abnormalities. In addition to joint fluid analysis, which of
the following is most likely to predict this patient's clinical course?

A) Blood cultures
B) Complete blood count
C) Serum C-reactive protein concentration
D) Serum lactate concentration
E) Serum uric acid concentration
Correct Answer: D.

The Third International Consensus Definitions for Sepsis and Septic Shock defined sepsis as life-threatening organ dysfunction caused
by a dysregulated host response to infection, and septic shock as a subcategory of sepsis characterized by a vasopressor requirement
to maintain mean arterial pressure greater than 65 mm Hg or a serum lactate concentration greater than 2 mmol/L despite fluid
resuscitation. Serum lactate concentration is reflective of organ hypoperfusion and dysfunction. The measurement of serum lactate is
therefore of prognostic utility for this patient with a clinical presentation concerning for septic arthritis and sepsis, as septic shock is
associated with an approximate four-fold increase in mortality (40%) as compared to sepsis alone (10%). These new criteria replaced
the previously adopted Systemic Inflammatory Response Syndrome (SIRS) Criteria , although the SIRS criteria remain widely used in
clinical practice.

Incorrect Answers: A , B, C, and E.

Blood cultures (Choice A) are useful for the identification of the causative pathogen and for guiding antimicrobial therapy but are not of
prognostic significance.

Complete blood count (Choice B) may disclose leukocytosis in patients with sepsis. This finding is nonspecific, however, and does not
necessarily suggest a dysregulated, life-threatening host response.

Serum C-reactive protein concentration (Choice C) is indicative of an acute phase inflammatory response but is nonspecific and of little
prognostic significance in patients with septic shock.
r ~, ~ p r,
Previous Next Score Report
https://t.me/USMLENBME2CK Lab Values Calculator Help Pause
Exam Section: Item 33 of 50 National Board of Medical Examiners
■ Mark Emergency Medicine Self-Assessment
...
B) Complete blood count
C) Serum C-reactive protein concentration
D) Serum lactate concentration
E) Serum uric acid concentration
Correct Answer: D.

The Third International Consensus Definitions for Sepsis and Septic Shock defined sepsis as life-threatening organ dysfunction caused
by a dysregulated host response to infection, and septic shock as a subcategory of sepsis characterized by a vasopressor requirement
to maintain mean arterial pressure greater than 65 mm Hg or a serum lactate concentration greater than 2 mmol/L despite fluid
resuscitation. Serum lactate concentration is reflective of organ hypoperfusion and dysfunction. The measurement of serum lactate is
therefore of prognostic utility for this patient with a clinical presentation concerning for septic arthritis and sepsis, as septic shock is
associated with an approximate four-fold increase in mortality (40%) as compared to sepsis alone (10%). These new criteria replaced
the previously adopted Systemic Inflammatory Response Syndrome (SIRS) Criteria, although the SIRS criteria remain widely used in
clinical practice.

Incorrect Answers: A, B, C, and E.

Blood cultures (Choice A) are useful for the identification of the causative pathogen and for guiding antimicrobial therapy but are not of
prognostic significance.

Complete blood count (Choice B) may disclose leukocytosis in patients with sepsis. This finding is nonspecific, however, and does not
necessarily suggest a dysregulated, life-threatening host response.

Serum C-reactive protein concentration (Choice C) is indicative of an acute phase inflammatory response but is nonspecific and of little
prognostic significance in patients with septic shock.

Serum uric acid concentration (Choice E) is useful for the evaluation of gout. This patient's clinical presentation, with worsening fever,
hypotension, joint swelling, and erythema, is more suggestive of septic arthritis than of gouty arthritis.

Educational Objective: The Third International Consensus Definitions for Sepsis and Septic Shock defined septic shock by a
vasopressor requirement to maintain mean arterial pressure greater than 65 mm Hg or a serum lactate concentration greater than 2
mmol/L despite fluid resuscitation. The measurement of serum lactate is of prognostic utility, as septic shock is associated with an
approximately four-fold increase in mortality as compared to sepsis alone.

.,.

r ~, ~ ~ r,
Previous Next Score Report
https://t.me/USMLENBME2CK Lab Values Calculator Help Pause
Exam Section: Item 34 of 50 National Board of Medical Examiners
■ Mark Emergency Medicine Self-Assessment

34. A 35-year-old woman is brought to the emergency department by ambulance after the patient cal led 911 to say that she had taken an overdose
and that she wanted to die. When the ambu lance arrived 9 minutes later, the paramedics say they found her on the sofa. She was obtunded and
had vomitus on her. Her pulse was 110/min and thready, respirations were 20/min, and blood pressure was 90/46 mm Hg. She was diaphoretic
and was given intravenous naloxone. Now, in the emergency department, vital signs are unchanged. Physical exam ination shows no abnormalities
except for obtundation. She is responsive to deep pain on ly; reflexes are 4+ and symmetric. ECG shows sinus tachycard ia at 110/min and a
prolonged, corrected QT interval. Her med ical record lists her current medications as imipram ine, atenolol, an oral contraceptive, and
acetaminophen. Past medications incl ude paroxetine and triazolam , which were discontinued 3 months ago. After inserting a large-bore
intravenous catheter for fluid adm inistration, which of the following is the most appropriate initial management?

A) Administer activated charcoal through a nasogastric tube


B) Administer 50% dextrose solution, intravenously
C) Intubate the patient
D) Perform emergency hemodialysis
E) Perform gastric lavage
Correct Answer: C.

This patient has attempted suicide by overdose likely with imipramine, a tricyclic antidepressant. Features of tricyclic antidepressant
overdose include confusion, delirium, cardiac arrhythmias, hypotension, and anticholinergic toxicity. This patient is obtunded and
unable to protect her airway. Thus, the first step in stabi lizing this patient is to intubate her, even before addressing the underlying
cause of her symptoms. Following intubation of this patient, further resuscitation includes the administration of sodium bicarbonate to
treat the patient's cardiac toxicity (demonstrated in this case by her prolonged corrected QT interval) and the administration of
intravenous fluids to correct her hypotension. A dose of activated charcoal may be given to bind any remaining imipramine but may only
be used if the airway is protected. The mainstay of treatment for tricyclic antidepressant overdose is sod ium bicarbonate to treat the
associated cardiac toxicity and prevent further arrhythmias. A broad set of laboratory analyses shou ld be sent to assess for end-organ
dysfunction and co-ingestion.

Incorrect Answers: A, B, D, and E.

Administer activated charcoal through a nasogastric tube (Choice A) shou ld not be performed prior to intubation in a patient who is
unable to protect their airway. In this patient, activated charcoal can be used following intubation but not before.

Administer 50% dextrose solution, intravenously (Choice B) is not indicated prior to intubation outside of suspected cases of profound
hypoglycemia. Following intubation, an isotonic fluid such as 0.9% saline shou ld be used for fluid resuscitation to address the patient's

r ~,
- - - - - - - -- - - - - -- - - .
hypotension. The administration of excess dextrose in a presumed euglycemic is not necessary and would not assist in improving the
-- - - -- - - ---- - -- - - --- - -- -- - --- -- -- - -
~
-- -- -- --
~ r,
....

Previous Next Score Report


https://t.me/USMLENBME2CK Lab Values Calculator Help Pause
Exam Section: Item 34 of 50 National Board of Medical Examiners
■ Mark Emergency Medicine Self-Assessment
...
D) Perform emergency hemodialysis
E) Perform gastric lavage
Correct Answer: C.

This patient has attempted suicide by overdose likely with imipramine, a tricyclic antidepressant. Features of tricyclic antidepressant
overdose include confusion, delirium, cardiac arrhythmias, hypotension, and anticholinergic toxicity. This patient is obtunded and
unable to protect her airway. Thus, the first step in stabi lizing this patient is to intubate her, even before addressing the underlying
cause of her symptoms. Following intubation of this patient, further resuscitation includes the administration of sodium bicarbonate to
treat the patient's cardiac toxicity (demonstrated in this case by her prolonged corrected QT interval) and the administration of
intravenous fluids to correct her hypotension. A dose of activated charcoal may be given to bind any remaining imipramine but may only
be used if the airway is protected. The mainstay of treatment for tricyclic antidepressant overdose is sodium bicarbonate to treat the
associated cardiac toxicity and prevent further arrhythmias. A broad set of laboratory analyses shou ld be sent to assess for end-organ
dysfunction and co-ingestion.

Incorrect Answers: A, B, D, and E.

Administer activated charcoal through a nasogastric tube (Choice A) shou ld not be performed prior to intubation in a patient who is
unable to protect their airway. In this patient, activated charcoal can be used following intubation but not before.

Administer 50% dextrose solution, intravenously (Choice B) is not indicated prior to intubation outside of suspected cases of profound
hypoglycemia. Following intubation, an isotonic fluid such as 0.9% saline shou ld be used for fluid resuscitation to address the patient's
hypotension. The administration of excess dextrose in a presumed euglycemic is not necessary and would not assist in improving the
patient's hemodynamics. A point-of-care glucose analysis should be obtained on any obtunded patient.

Perform emergency hemodialysis (Choice D) is not yet indicated as the patient should be stabi lized first and then treated with sodium
bicarbonate. Additionally, only select solutes are dialyzable.

Gastric lavage (Choice E) is generally contraindicated in the treatment of tricyclic antidepressant overdose as it can worsen the
associated cardiac arrhythmias by increasing vagal tone. Gastrointestinal decontamination may be performed through a sing le dose of
activated charcoal, but only after the patient's airway is protected.

Educational Objective: In patients presenting with tricyclic antidepressant overdose, initial management should include airway
protection, the administration of sodium bicarbonate for cardiac toxicity, and intravenous fluids for hemodynamic resuscitation.

.,.

r ~, ~ ~ r,
Previous Next Score Report
https://t.me/USMLENBME2CK Lab Values Calculator Help Pause
Exam Section: Item 35 of 50 National Board of Medical Examiners
■ Mark Emergency Medicine Self-Assessment

35. A 60-year-old man in cardiac arrest is brought to the emergency department by paramedics. Bystanders initiated cardiopu lmonary resuscitation
(CPR) within mi nutes of his col lapse. Paramedics say his initial rhythm was pulseless electrical activity (PEA) when they arrived at the scene. En
route to the hospital, paramedics continued CPR and obtained a supraglottic airway. Three doses of epinephrine were adm inistered . No med ical
history is available. On arrival, CPR is continued, and the first rhythm is PEA. The supraglottic airway is functioning appropriately. Auscultation of
the lungs discloses coarse breath sounds bilaterally. On a brief secondary survey, an arteriovenous graft is noted in the patient's left arm, and he
has had a right-sided below-knee amputation. There are no signs of trauma. Fingerstick blood glucose concentration is 312 mg/dl (17.32 mmol/L).
Which of the following interventions is most likely to improve th is patient's condition?

A) Administration of atropine
B) Administration of calcium chloride
C) Administration of insulin
D) Administration of vasopressin (ADH)
E) Endotracheal intubation
F) Needle thoracostomy
Correct Answer: B.

Cardiac arrest can be caused by a variety of conditions, including hypovolemia, hypoxia, acidosis, hypothermia, hyperkalemia,
hypokalemia, overdose, cardiac tamponade, acute coronary syndrome, pulmonary embolism, hypoglycemia, and tension
pneumothorax among others. In this patient with evidence of an upper extremity arteriovenous graft presenting in pulseless electrical
activity (PEA) arrest, there is high suspicion for hyperkalemia as a cause of cardiac arrest as a result of underlying kidney injury.
Patients with end-stage kidney disease requiring dialysis are prone to electrolyte abnormalities, especially hyperkalemia. Severe
hyperkalemia leads to reduced myocardial excitability resulting in repolarization abnormalities, conduction dysfunction, heart block, and
bradycardia. These abnormalities can manifest on an ECG with peaked T waves, P-R interval prolongation, loss of P waves, and if
severe, widened ORS complexes, bradycardia, and cardiac arrest. Hyperkalemia resulting in ECG changes or cardiac arrest requires
the emergent administration of calcium gluconate or calcium chloride to induce cardiac cell membrane stabilization. Further treatment
after calcium may involve insulin and glucose, ~-adrenergic agonists, or bicarbonate to shift potassium into cells, loop diuretics to
facilitate the urinary excretion of potassium, and sodium polystyrene sulfonate to promote binding of potassium within the lumen of the
gut. For severe or refractory cases, dialysis may be necessary.

Incorrect Answers: A, C, D, E, and F.

Administration of atropine (Choice A), a cholinergic inhibitor, is used in the management of symptomatic bradycardia and in
organophosphate toxicity. It has no role in the management of PEA arrest outside of these cases.
....

r ~, ~ ~ r,
Previous Next Score Report
https://t.me/USMLENBME2CK Lab Values Calculator Help Pause
Exam Section: Item 35 of 50 National Board of Medical Examiners
■ Mark Emergency Medicine Self-Assessment

Correct Answer: B. ...

Cardiac arrest can be caused by a variety of conditions, including hypovolemia, hypoxia, acidosis, hypothermia, hyperkalemia,
hypokalemia, overdose, cardiac tamponade, acute coronary syndrome, pulmonary embolism, hypoglycemia, and tension
pneumothorax among others. In this patient with evidence of an upper extremity arteriovenous graft presenting in pulseless electrical
activity (PEA) arrest, there is high suspicion for hyperkalem ia as a cause of cardiac arrest as a result of underlying kidney injury.
Patients with end-stage kidney disease requiring dialysis are prone to electrolyte abnormalities, especially hyperkalem ia. Severe
hyperkalemia leads to reduced myocardial excitability resulting in repolarization abnormalities, conduction dysfunction, heart block, and
bradycardia. These abnormalities can manifest on an ECG with peaked T waves, P-R interval prolongation, loss of P waves, and if
severe, widened ORS complexes, bradycardia, and cardiac arrest. Hyperkalemia resu lting in ECG changes or cardiac arrest requires
the emergent administration of calcium gluconate or calcium chloride to induce cardiac cell membrane stabilization. Further treatment
after calcium may involve insu li n and glucose, ~-adrenergic agonists, or bicarbonate to shift potassium into cells, loop diuretics to
facilitate the urinary excretion of potassium, and sodium polystyrene sulfonate to promote binding of potassium within the lumen of the
gut. For severe or refractory cases, dialysis may be necessary.

Incorrect Answers: A, C, D, E, and F.

Administration of atropine (Choice A), a cholinergic inhibitor, is used in the management of symptomatic bradycardia and in
organophosphate toxicity. It has no role in the management of PEA arrest outside of these cases.

Administration of insulin (Choice C) would be helpful for the treatment of hyperkalemia but should be given after the administration of
calcium chloride or calcium gluconate for stabilization of the cardiac membrane. Insulin will shift potassium intracellularly to decrease
serum potassium concentrations.

Vasopressin (ADH) (Choice D) can be used as a vasopressor due to its vasoconstrictive properties, which increase systemic vascular
resistance. It can be used in cases of distributive shock, such as septic shock, but is typically not a first-line vasopressor. It would not
be helpful in this patient who is pulseless and in cardiac arrest.

While endotracheal intubation (Choice E) can be helpful in cases of cardiac arrest to secure the airway and prevent or treat hypoxia,
the cause of this patient's cardiac arrest is likely hyperkalemia, which should be treated emergently with calcium chloride or gluconate.
As well, this patient has a supraglottic airway which is functioning appropriately and will sufficiently supply oxygenation and ventilation
to the patient. Endotracheal intubation can be attempted at a later time for a definitive airway, if necessary.

Needle thoracostomy (Choice F) would be helpful if this patient's arrest was caused by a tension pneumothorax. Tension pneumothorax
can be a complication of penetrating thoracic trauma and occurs when air is able to enter the pleural space but is prevented from
exiting. Tension pneumothorax is characterized by absent breath sounds and hyperresonance to percussion on the affected side.

Educational Objective: Hyperkalemia leads to reduced myocardial excitability resulting in repolarization abnormalities, conduction ....

r ~, ~ ~ r,
Previous Next Score Report
https://t.me/USMLENBME2CK Lab Values Calculator Help Pause
Exam Section: Item 35 of 50 National Board of Medical Examiners
■ Mark Emergency Medicine Self-Assessment

Cardiac arrest can be caused by a variety of conditions, including hypovolemia, hypoxia, acidosis, hypothermia, hyperkalemia, ...
hypokalemia, overdose, cardiac tamponade, acute coronary syndrome, pulmonary embolism, hypoglycemia, and tension
pneumothorax among others. In this patient with evidence of an upper extremity arteriovenous graft presenting in pulseless electrical
activity (PEA) arrest, there is high suspicion for hyperkalem ia as a cause of cardiac arrest as a result of underlying kidney injury.
Patients with end-stage kidney disease requiring dialysis are prone to electrolyte abnormalities, especially hyperkalem ia. Severe
hyperkalemia leads to reduced myocardial excitability resulting in repolarization abnormalities, conduction dysfunction, heart block, and
bradycardia. These abnormalities can manifest on an ECG with peaked T waves, P-R interval prolongation, loss of P waves, and if
severe, widened ORS complexes, bradycardia, and cardiac arrest. Hyperkalemia resu lting in ECG changes or cardiac arrest requires
the emergent administration of calcium gluconate or calcium chloride to induce cardiac cell membrane stabilization. Further treatment
after calcium may involve insu li n and glucose, ~-adrenergic agonists, or bicarbonate to shift potassium into cells, loop diuretics to
facilitate the urinary excretion of potassium, and sodium polystyrene sulfonate to promote binding of potassium within the lumen of the
gut. For severe or refractory cases, dialysis may be necessary.

Incorrect Answers: A, C, D, E, and F.

Administration of atropine (Choice A), a cholinergic inhibitor, is used in the management of symptomatic bradycardia and in
organophosphate toxicity. It has no role in the management of PEA arrest outside of these cases.

Administration of insulin (Choice C) would be helpful for the treatment of hyperkalemia but should be given after the administration of
calcium chloride or calcium gluconate for stabilization of the cardiac membrane. Insulin will shift potassium intracellularly to decrease
serum potassium concentrations.

Vasopressin (ADH) (Choice D) can be used as a vasopressor due to its vasoconstrictive properties, which increase systemic vascular
resistance. It can be used in cases of distributive shock, such as septic shock, but is typically not a first-line vasopressor. It would not
be helpful in this patient who is pulseless and in cardiac arrest.

While endotracheal intubation (Choice E) can be helpful in cases of cardiac arrest to secure the airway and prevent or treat hypoxia,
the cause of this patient's cardiac arrest is likely hyperkalemia, which should be treated emergently with calcium chloride or gluconate.
As well, this patient has a supraglottic airway which is functioning appropriately and will sufficiently supply oxygenation and ventilation
to the patient. Endotracheal intubation can be attempted at a later time for a definitive airway, if necessary.

Needle thoracostomy (Choice F) would be helpful if this patient's arrest was caused by a tension pneumothorax. Tension pneumothorax
can be a complication of penetrating thoracic trauma and occurs when air is able to enter the pleural space but is prevented from
exiting. Tension pneumothorax is characterized by absent breath sounds and hyperresonance to percussion on the affected side.

Educational Objective: Hyperkalemia leads to reduced myocardial excitability resulting in repolarization abnormalities, conduction
dysfunction, heart block, bradycardia, and potentially cardiac arrest. Emergent administration of calcium gluconate or calcium chloride
is indicated to induce cardiac cell membrane stabilization. ....

r ~, ~ ~ r,
Previous Next Score Report
https://t.me/USMLENBME2CK Lab Values Calculator Help Pause
Exam Section: Item 36 of 50 National Board of Medical Examiners
■ Mark Emergency Medicine Self-Assessment
....

36. A 43-year-old woman comes to the emergency department because of a 1-day history of upper abdominal pain that radiates to the back and
several episodes of vomiting. She also notes a 1-month history of epigastric and right upper quadrant pain that lasts between 30 minutes and 1
hour and is triggered by eating, particularly fatty foods. These episodes subside with time and are associated with nausea but no vomiting . She
has heavy menstrual bleeding treated with oral contraceptive therapy. Vital signs are within normal limits. Abdominal examination discloses
epigastric tenderness to palpation with voluntary guarding. Which of the following laboratory results are most likely in this patient?

A) Decreased serum alkaline phosphatase activity


B) Decreased serum urea nitrogen:creatinine ratio
C) Increased serum indirect bilirubin concentration
D) Increased serum lipase activity
E) Positive Helicobacter pylori stool antigen
Correct Answer: D.

Increased serum lipase activity is commonly seen in acute pancreatitis. Acute pancreatitis presents with epigastric abdominal pain that
radiates to the back, along with nausea and emesis, often in a patient with a history of gallstones, alcoholism , trauma ,
hypertriglyceridemia, or hypercalcemia. Gallstone pancreatitis may be preceded by other signs of cholelithiasis including biliary colic, as
seen in this case . Acute pancreatitis can be complicated by necrosis, hemorrhage, abscess, or the formation of pseudocysts.
Laboratory evaluation often shows increased serum amylase and lipase concentrations. Treatment of uncomplicated acute pancreatitis
is largely supportive and should include intravenous fluids , either 0.9% saline or lactated Ringer solution , bowel rest, and pain control.
Surgical intervention may be required in cases of complications.

Incorrect Answers : A , B, C, and E.

Increased, not decreased serum alkaline phosphatase activity (Choice A), is seen in some complications of gallstones such as
choledocholithiasis, ascending cholangitis, and gallstone pancreatitis.

Acute pancreatitis can cause a severe inflammatory response accompanied by hypotension leading to a prerenal acute kidney injury.
Increased, not decreased serum urea nitrogen:creatinine ratio (Choice B), characterizes this type of kidney injury.

Direct, not indirect (Choice C) , bilirubin concentration is increased by obstruction of the biliary tree . Direct bilirubin is secreted into the
bile canaliculi of the liver and eventually drains into the common bile duct before entering the duodenum. Obstruction of the flow of bile
in the setting of choledocholithiasis or ascending cholangitis causes direct bilirubin to leak into the surrounding interstitium and
vasculature .

• -- - - ------ - - --- -- --- ---- ----- --- --- ---- - -- ------ -- ---- - ------ - -

r ~, ~ p r,
Previous Next Score Report
https://t.me/USMLENBME2CK Lab Values Calculator Help Pause
Exam Section: Item 36 of 50 National Board of Medical Examiners
■ Mark Emergency Medicine Self-Assessment
...
C) Increased serum indirect bi lirubin concentration
D) Increased serum lipase activitYi
E) Positive Helicobacter pylori stool antigen
Correct Answer: D.

Increased serum lipase activity is commonly seen in acute pancreatitis. Acute pancreatitis presents with epigastric abdominal pain that
radiates to the back, along with nausea and emesis, often in a patient with a history of gallstones, alcoholism, trauma,
hypertriglyceridemia, or hypercalcemia. Gallstone pancreatitis may be preceded by other signs of cholelithiasis including biliary colic, as
seen in this case. Acute pancreatitis can be complicated by necrosis, hemorrhage, abscess, or the formation of pseudocysts.
Laboratory evaluation often shows increased serum amylase and lipase concentrations. Treatment of uncomplicated acute pancreatitis
is largely supportive and should include intravenous fluids, either 0.9% saline or lactated Ringer solution, bowel rest, and pain control.
Surgical intervention may be required in cases of complications.

Incorrect Answers: A, B, C, and E.

Increased, not decreased serum alkaline phosphatase activity (Choice A), is seen in some complications of gallstones such as
choledocholithiasis, ascending cholangitis, and gallstone pancreatitis.

Acute pancreatitis can cause a severe inflammatory response accompanied by hypotension leading to a prerenal acute kidney injury.
Increased, not decreased serum urea nitrogen:creatinine ratio (Choice B), characterizes this type of kidney injury.

Direct, not indirect (Choice C), bilirubin concentration is increased by obstruction of the biliary tree. Direct bilirubin is secreted into the
bile canaliculi of the liver and eventually drains into the common bile duct before entering the duodenum. Obstruction of the flow of bile
in the setting of choledocholithiasis or ascending cholangitis causes direct bilirubin to leak into the surrounding interstitium and
vasculature.

Positive Helicobacter pylori stool antigen (Choice E) is seen in patients with peptic ulcer disease induced by Helicobacter pylori
infection. While gastric or duodenal ulcers are also a cause of epigastric pain, this patient's history of biliary colic is more suggestive of
gallstone pancreatitis.

Educational Objective: Acute pancreatitis presents with epigastric abdominal pain that radiates to the back, along with nausea and
emesis, often in a patient with a history of gallstones, alcoholism, trauma, hypertriglyceridemia, or hypercalcemia. Laboratory evaluation
often shows increased serum amylase and lipase concentrations.

.,.

r ~, ~ ~ r,
Previous Next Score Report
https://t.me/USMLENBME2CK Lab Values Calculator Help Pause
Exam Section : Item 37 of 50 National Board of Medical Examiners
■ Mark Emergency Medicine Self-Assessment

37. An otherwise healthy 25-year-old woman comes to the emergency department because of a 5-day history of low-grade fever and pain, red ness,
and swel ling of an area on her left thigh. There is no history of recent trauma to the thigh. Her temperature is 37°C (98.6°F), pulse is 85/min,
respirations are 14/min, and blood pressure is 110/78 mm Hg. Physical examination shows tenderness of the left th igh. There is no fluctuance. A
photograph of the left thigh is shown. No other abnormalities are noted. Which of the following is the most appropriate next step in management?
A) Complete blood count with differential
B) CT scan of the lower extremity with contrast
C) Empiric antibiotic therapYi
D) Incision and drainage of the affected area
E) Needle aspiration
Correct Answer: C.

1•110! l "! II I O!I• 1• . "! •I •1•1 • • 1 1• I• "! •I •11 O!I I .

~
Previous Score Report
https://t.me/USMLENBME2CK Lab Values Calculator Help Pause
Exam Section : Item 37 of 50 National Board of Medical Examiners
■ Mark Emergency Medicine Self-Assessment
...
A) Complete blood count with differential
B) CT scan of the lower extremity with contrast
C) Empiric antibiotic therapy;
D) Incision and drainage of the affected area
E) Needle aspiration
Correct Answer: C.

Cellulitis presents with cutaneous erythema, warmth, and induration, often following inoculation from an injury such as an abrasion.
Systemic symptoms, including fever, tachycardia, and leukocytosis are often present. Most commonly, it is acute and results from a
bacterial infection of the skin. The two most common pathogens to cause cellulitis are Staphylococcus aureus and Streptococcus
pyogenes (group A). Clinically, the cellulitis caused by S. aureus is typically purulent while that caused by S. pyogenes is not. Treatment
of uncomplicated cellulitis, as seen in this case, is with empiric antibiotic therapy, which covers both S. aureus and S. pyogenes, such
as cephalexin. If the cellulitis demonstrates purulence or the patient has a history of prior methicillin-resistant S. aureus (MRSA)
infections, coverage of MRSA is also warranted. Potential oral antibiotic choices for MRSA coverage include doxycycline, trimethoprim-
sulfamethoxazole, or clindamycin.

Incorrect Answers: A, B, D, and E.

Complete blood count with differential (Choice A) is not necessary in the work up of uncomplicated cellulitis in an otherwise healthy
patient who is afebrile and has no vital sign instability. If the patient demonstrated any signs of sepsis, then a complete blood count with
differential and blood cultures would be indicated prior to initiating intravenous antibiotics.

CT scan of the lower extremity with contrast (Choice B) is not necessary in the management of uncomplicated cellulitis. Occasionally,
bacterial abscesses may involve the underlying soft tissues, in which case imaging may be warranted.

Incision and drainage of the affected area (Choice D) or needle aspiration (Choice E) is not necessary in this case given that there is no
evidence of an abscess on physical examination. If an abscess is present, then incision and drainage in addition to antibiotic therapy is
necessary.

Educational Objective: Empiric antibiotic therapy is the treatment of choice for uncomplicated cel lulitis. Coverage of the most common
causative organisms, Staphylococcus aureus and Streptococcus pyogenes, is vital. If the cellulitis demonstrates purulence, or the
patient has a history of prior methicillin-resistant S. aureus (MRSA) infections, then coverage of MRSA is also warranted.

.,.

r ~, ~ ~ r,
Previous Next Score Report
https://t.me/USMLENBME2CK Lab Values Calculator Help Pause
Exam Section: Item 38 of 50 National Board of Medical Examiners
■ Mark Emergency Medicine Self-Assessment

38. A 67-year-old man is brought to the emergency department because of a 45-minute history of chest heaviness, shortness of breath, and nausea.
He says he was shoveling snow when the symptoms began. He has hypertension and type 2 diabetes mellitus. His medications are lisinopri l and
metformin. He does not smoke cigarettes or drink alcohol. On arrival, he appears anxious and is in moderate distress. He is pale and diaphoretic.
His temperature is 37.1 °C (98.8°F), pulse is 85/m in, respirations are 16/min, and blood pressure is 138/75 mm Hg. Pu lse oximetry on room air
shows an oxygen saturation of 97%. The lungs are clear to auscu ltation. Card iac exam ination discloses no murmurs or rubs . An ECG is shown.
This patient most likely has a lesion of which of the following arteries?

A) Left anterior descending


B) Left circumflex
C) Posterior coronary
D) Right coronary
E) Septal perforating
Correct Answer: A.

The patient is presenting with classic signs and symptoms of acute coronary syndrome, with angina, nausea, dyspnea, anxiety, and
diaphoresis that began during exertion. Additional ly, the ECG shows marked ST-segment elevations in the precordial leads V2 through
V6. Polarization and repolarization abnormalities along these vectors correspond with injury to the myocardial territory supplied by the

r ~,
·- -• - . - - .. ·- - -• . ·- ..... - . -• ·- ... ·- - ••- .. ••-
~ ~
-
r,
...
left anterior descending coronary artery, which includes the anterior portion of the interventricular septum, the anterior left ventricle, and
- - ·-• - - - - -•· - -

Previous Next Score Report


https://t.me/USMLENBME2CK Lab Values Calculator Help Pause
Exam Section: Item 38 of 50 National Board of Medical Examiners
■ Mark Emergency Medicine Self-Assessment
• • • • • • ...
Correct Answer: A.

The patient is presenting with classic signs and symptoms of acute coronary syndrome, with angina, nausea, dyspnea, anxiety, and
diaphoresis that began during exertion. Additional ly, the ECG shows marked ST-segment elevations in the precordial leads V2 through
V6. Polarization and repolarization abnormalities along these vectors correspond with injury to the myocardial territory supplied by the
left anterior descending coronary artery, which includes the anterior portion of the interventricular septum, the anterior left ventricle, and
the anterolateral papillary muscle. The left anterior descending artery branches from the left main coronary artery and perfuses a
significant portion of the myocardium. An occlusive atherosclerotic or thromboembolic lesion in this vessel resulting in transmural
myocardial ischemia and infarction is likely underlying this patient's presentation. Treatment requires the use of antiplatelet agents (eg,
aspirin, clopidogrel) plus anticoagulants (eg, heparin), pain control, and revascularization through angioplasty, thrombolysis, or coronary
artery bypass grafting. Potential complications include papillary muscle rupture with subsequent mitral regurgitation, dysrhythmias,
cardiogenic shock, pu lmonary edema, ventricular wal l rupture, and post-infarction pericarditis.

Incorrect Answers: B, C, D, and E.

The left circumflex (Choice B) artery originates from the left maiin coronary artery and primarily provides perfusion to the lateral and
posterior walls of the left ventricle, the anterolateral papillary muscle, and some blood flow to the AV node. Infarction of this territory
may resu lt in ST-segment changes in the lateral, posterior, and/or inferior ECG leads (depending on right- or left-dominant coronary
artery circu lation).

The posterior coronary (Choice C) artery, in the setting of a right-dominant circu lation (~85% of patients), provides a majority of the
blood supply to the inferior portion of the heart. Corresponding ST-segment changes are typically found in the inferior ECG leads II, Ill,
and aVF.

The right coronary (Choice D) artery primari ly provides blood flow to the right ventricle, right atrium, sinoatrial (SA) node, atrioventricular
(AV) node, and gives rise to the posterior coronary artery in the majority of the population (~85% of patients). Corresponding ST-
segment changes are typically seen in the inferior ECG leads II, Ill, and aVF. Impaired perfusion to the SA and AV nodes can also result
in significant bradycardia or heart block.

Septal perforating (Choice E) branches of the left anterior descending artery provide most of the blood supply to the interventricular
septum. A lesion isolated to these vessels typically presents with ST-segment changes in the anterior precordial leads V1 through V3
without much effect on the lateral leads V4 through V6.

Educational Objective: ST-segment changes observed throughout the anterior and lateral precordial leads are most suggestive of a
lesion involving the left anterior descending artery, which provides blood supply to the anterior portion of the interventricular septum, the
anterior left ventricle, and the anterolateral papillary muscle.
....

r ~, ~ ~ r,
Previous Next Score Report
https://t.me/USMLENBME2CK Lab Values Calculator Help Pause
Exam Section: Item 39-40 of 50 National Board of Medical Examiners
■ Mark Emergency Medicine Self-Assessment

The following vignette applies to the next 2 items. The items in the set must be answered in sequential order. Once you click Proceed to Next Item, you will not
be able to add or change an answer.

A 73-year-old man is brought to the emergency department by paramedics because of a 4-day history of progressive shortness of breath. Paramed ics report that
during transport the patient was awake and alert. Pulse oximetry on room air showed an oxygen saturation of 88%; 4 L/min of oxygen via nasal cannula was
initiated. Diffuse wheezes were heard bilaterally and inhaled albuterol and ipratropium were administered. Medical history incl udes congestive heart failure,
chronic obstructive pulmonary disease, and type 2 diabetes mellitus. Current med ications are furosem ide, albuterol, insulin, glyburide, and tramadol. He has
smoked two packs of cigarettes daily for 50 years. On arrival, he is slow to respond and sleepy but easily aroused. His temperature is 36.9°C (98.4°F), pulse is
112/m in, respirations are 12/min, and blood pressure is 167/98 mm Hg. Pulse oximetry now shows an oxygen saturation of 95%. Pupils are normal and reactive.
Severe inspiratory and expiratory wheezes are heard on auscultation.

'I 39. Item 1 of 2


Which of the following is the most likely cause of this patient's change in mental status during transport to the hospital?

A) Circulating endotoxins
Correct Answer: B.

Hypercarbia most likely explains this patient's change in mental status during transport to the hospital. Patients with severe chronic
obstructive pulmonary disease (COPD) commonly develop hypercarbia at baseline as a result of their obstructive disease, but acute
exacerbations, which are frequently the result of respiratory tract infections, can lead to an acute worsening of the underlying
obstruction. Patients present with increased dyspnea and wheezing, which represents turbulent airflow through the constricted
bronchial tree. Concurrent cough and sputum production are also common. When the underlying obstruction is exacerbated by a
trigger, patients develop profound air trapping and are less capable of expelling carbon dioxide. On arterial blood gas analysis, this
manifests as an increased partial pressure of carbon dioxide. Severe hypercarbia can lead to lethargy and mental status changes. For
this reason, all patients with a COPD exacerbation and altered mental status should be evaluated for the presence of hypercarbia.
Treatment involves the administration of bronchodilator medications, steroids, and positive pressure ventilation. Occasionally, intubation
is required if the patient becomes obtunded. Additionally, patients with COPD tend to have a lower oxygen saturation at baseline as a
result of their underlying pulmonary parenchymal disease. During COPD exacerbations, the goal oxygen saturation should target
between 88% to 92%. Historically, administration of supplemental oxygen to raise the saturation beyond this amount has been
postulated to blunt respiratory drive, though actual evidence for this is limited. It is probable instead that the administration of
supplemental oxygen worsens ventilation-perfusion mismatch in this patient population.

Incorrect Answers: A, C, D, and E.

Circulating endotoxins (Choice A) may be present in patients with sepsis. This patient is experiencing an acute exacerbation of his
COPD dd th 'd f •.
r ~, ~ ~ r,
Previous Next Score Report
https://t.me/USMLENBME2CK Lab Values Calculator Help Pause
Exam Section : Item 39-40 of 50 National Board of Medical Examiners
■ Mark Emergency Medicine Self-Assessment

Hypercarbia most likely explains this patient's change in mental status during transport to the hospital. Patients with severe chronic
obstructive pulmonary disease (COPD) commonly develop hypercarbia at baseline as a result of their obstructive disease, but acute
exacerbations, which are frequently the result of respiratory tract infections, can lead to an acute worsening of the underlying
obstruction. Patients present with increased dyspnea and wheezing , which represents turbulent airflow through the constricted
bronchial tree. Concurrent cough and sputum production are also common. When the underlying obstruction is exacerbated by a
trigger, patients develop profound air trapping and are less capable of expelling carbon dioxide. On arterial blood gas analysis , this
manifests as an increased partial pressure of carbon dioxide. Severe hypercarbia can lead to lethargy and mental status changes. For
this reason , all patients with a COPD exacerbation and altered mental status should be evaluated for the presence of hypercarbia.
Treatment involves the administration of bronchodilator medications, steroids , and positive pressure ventilation. Occasionally, intubation
is required if the patient becomes obtunded . Additionally, patients with COPD tend to have a lower oxygen saturation at baseline as a
result of their underlying pulmonary parenchymal disease. During COPD exacerbations , the goal oxygen saturation should target
between 88% to 92%. Historically, administration of supplemental oxygen to raise the saturation beyond this amount has been
postulated to blunt respiratory drive, though actual evidence for this is limited. It is probable instead that the administration of
supplemental oxygen worsens ventilation-perfusion mismatch in this patient population.

Incorrect Answers : A , C, D, and E.

Circulating endotoxins (Choice A) may be present in patients with sepsis. This patient is experiencing an acute exacerbation of his
COPD and does not have evidence of sepsis.

Hyperglycemic nonketotic coma (Choice C) is incorrect. This patient's symptoms are primarily respiratory, thus his altered mental status
is more likely secondary to hypercarbia. A clinical history of hyperglycemia, polyuria, polydipsia , and dehydration would also be
expected.

Hypoxemia (Choice D) is incorrect. It is common for patients with COPD to have a low oxygen saturation at baseline. This patient's
oxygen saturation of 88% corresponds with a partial pressure of oxygen of approximately 55 mm Hg. While this is lower than normal, it
would be unlikely to cause altered mental status and may actually be a baseline, around-the-clock saturation for many patients with
COPD.

Medication adverse effect (Choice E) is incorrect. Insulin and glyburide can cause hypoglycemia, but this patient would likely have
demonstrated altered mental status at the time when the paramedics arrived. Tramadol can also cause lethargy, but symptoms would
have been expected at the scene.

Educational Objective: Acute exacerbations of COPD cause increased air trapping and poor ventilation resulting in progressive
hypercarbia. As well , the administration of supplemental oxygen during an acute exacerbation should target a goal oxygen saturation of
88% to 92%, as higher saturations can worsen the patient's underlying ventilation-perfusion mismatch . Severe hypercarbia can cause
lethargy and altered mental status.
r ~, ~ F' r,
Previous Next Score Report
https://t.me/USMLENBME2CK Lab Values Calculator Help Pause
Exam Section: Item 39-40 of 50 National Board of Medical Examiners
■ Mark Emergency Medicine Self-Assessment
...
'I 40. Item 2 of 2
After administration of inhaled albuterol and ipratropium, the patient's cond ition has not improved . Physical findings are unchanged . Wh ich of the
following is the most appropriate immediate next step?

A) Administer ceftriaxone and azithromycin


B) Administer naloxone
C) Administer 0.9% saline
D) Initiate noninvasive positive pressure ventilation
E) Switch from a nasal cannula to a nonrebreather mask
Correct Answer: D.

Initiate noninvasive positive pressure ventilation (NIPPV) is the most appropriate immediate next step. Patients with acute
exacerbations of chronic obstructive pu lmonary disease (COPD) often develop hypercarbia as a consequence of bronchial obstruction
and poor venti lation. Initial treatment should focus on dilating the airways with the use of ~-adrenergicagonist medications such as
albuterol. Additionally, antimuscarinic agents such as ipratropium can be given along with oral or intravenous steroids. In some
instances, hypercarbia persists despite these interventions. These patients benefit from the use of NIPPV. This type of venti lation
includes continuous positive airway pressure (CPAP) and bilevel positive airway pressure (BiPAP). In bilevel modes, both the
inspiratory and expiratory pressure can be set to augment the patient's venti lation. The expiratory pressure helps to maintain patency of
the airway during expiration in an attempt to combat the collapse of the airway that is common to patients with COPD. This type of
assisted ventilation frequently resu lts in resolution of hypercarbia, but only patients who are awake enough to cooperate with the
machine are candidates for its use.

Incorrect Answers: A, B, C, and E.

Administer ceftriaxone and azithromycin (Choice A) is incorrect. This combination of antibiotics is appropriate for the treatment of
patients who are suspected to have community acquired pneumonia. This patient presents with a COPD exacerbation and requires
NI PPV to treat his hypercarbia.

Administer naloxone (Choice B) is incorrect. Naloxone is an opiate-reversal agent that is given to patients who are thought to have
overdosed on opiates. This patient has no history of opiate use.

Administer 0.9% saline (Choice C) is not correct. This patient requires NIPPV, not intravenous fluids.

Switch from nasal cannula to nonrebreather mask (Choice E) is incorrect. A nonrebreather mask delivers more oxygen than a nasal
cannula, but this wi ll not correct the underlying obstructive ventilation problem, which is more amenable to treatment with NIPPV.
....

r ~, ~ ~ r,
Previous Next Score Report
https://t.me/USMLENBME2CK Lab Values Calculator Help Pause
Exam Section : Item 39-40 of 50 National Board of Medical Examiners
■ Mark Emergency Medicine Self-Assessment
9 A I

C) Administer 0.9% saline


D) Initiate noninvasive positive pressure ventilation
E) Switch from a nasal cannula to a nonrebreather mask
Correct Answer: D.

Initiate noninvasive positive pressure ventilation (NIPPV) is the most appropriate immediate next step. Patients with acute
exacerbations of chronic obstructive pulmonary disease (COPD) often develop hypercarbia as a consequence of bronchial obstruction
and poor ventilation. Initial treatment should focus on dilating the airways with the use of ~-adrenergicagonist medications such as
albuterol. Additionally, antimuscarinic agents such as ipratropium can be given along with oral or intravenous steroids. In some
instances, hypercarbia persists despite these interventions. These patients benefit from the use of NI PPV. This type of ventilation
includes continuous positive airway pressure (CPAP) and bilevel positive airway pressure (BiPAP). In bilevel modes, both the
inspiratory and expiratory pressure can be set to augment the patient's ventilation. The expiratory pressure helps to maintain patency of
the airway during expiration in an attempt to combat the collapse of the airway that is common to patients with COPD. This type of
assisted ventilation frequently results in resolution of hypercarbia, but only patients who are awake enough to cooperate with the
machine are candidates for its use.

Incorrect Answers : A , B, C, and E.

Administer ceftriaxone and azithromycin (Choice A) is incorrect. This combination of antibiotics is appropriate for the treatment of
patients who are suspected to have community acquired pneumonia. This patient presents with a COPD exacerbation and requires
NIPPV to treat his hypercarbia.

Administer naloxone (Choice B) is incorrect. Naloxone is an opiate-reversal agent that is given to patients who are thought to have
overdosed on opiates. This patient has no history of opiate use .

Administer 0.9% saline (Choice C) is not correct. This patient requires NIPPV, not intravenous fluids .

Switch from nasal cannula to nonrebreather mask (Choice E) is incorrect. A nonrebreather mask delivers more oxygen than a nasal
cannula, but this will not correct the underlying obstructive ventilation problem , which is more amenable to treatment with NIPPV.

Educational Objective: The ventilation deficits leading to hypercarbia that occur with COPD exacerbations should first be treated with
bronchodilator medications. Patients with respiratory distress or evidence of worsening hypercarbia should be started on NIPPV.

End of Set
...
r ~, ~ p r,
Previous Next Score Report
https://t.me/USMLENBME2CK Lab Values Calculator Help Pause
Exam Section : Item 41 of 50 National Board of Medical Examiners
■ Mark Emergency Medicine Self-Assessment

41. A 45-year-old man is brought to the emergency department because of mu ltiple episodes of vomiting during the past 30 minutes. The first episode
was nonbloody, but all subsequent episodes have been a mixture of bright red blood and a coffee-ground-like substance. He has no history of
similar symptoms. He has a history of major depressive disorder treated with fluoxetine. He drinks 12 beers daily. On arrival, he is alert and fully
oriented. He is retch ing actively in the bed. His skin is pale and diaphoretic. His tem peratu re is 37.3°C (99.1 °F), pulse is 145/min, respirations are
22/min, and blood pressure is 80/35 mm Hg. Pulse oximetry on room air shows an oxygen saturation of 96%. Abdominal examination shows
distention with a positive fluid wave; there is no tenderness, guarding, or rebound. Rectal examination shows brown stool; test of the stool for
occult blood is negative. Two large-bore intravenous catheters are inserted , and 0.9% saline (2 L) is admin istered. His pulse now is 140/min, and
blood pressure is 90/60 mm Hg. Which of the following is the most likely cause of this patient's bleeding?

A) Acid reflux
B) Coagu lopathy
C) Esophageal rupture
D) Helicobacter pylori infection
E) Varices
Correct Answer: E.

Varices secondary to cirrhosis is the most likely cause of this patient's bleeding. Major causes of upper gastrointestinal bleeding (UGIB)
include peptic ulcer disease, Mallory-Weiss tear, gastritis, and Dieulafoy lesion among others, but few etiologies cause hemorrhage as
dramatic and potentially catastrophic as ruptured esophageal varices. This patient has evidence of decompensated cirrhosis with a
distended abdomen and fluid wave indicating ascites. Both ascites and esophageal varices are consequences of portal venous
hypertension, which occurs as a result of obstruction of flow through the sinusoids within the liver. All patients with bleeding esophageal
varices should have two large-bore intravenous catheters inserted for the administration of isotonic crystalloids and packed red blood
cells (pRBCs). This patient continues to have signs of hemorrhagic shock despite adequate fluid resuscitation; thus, he should also
receive pRBCs. Additionally, patients with bleeding varices should receive intravenous antibiotics, such as ceftriaxone, an octreotide
infusion, and a proton pump inhibitor. Initial care should focus on stabilizing the patient to facilitate upper endoscopy, at which point
varices can be ligated for definitive source control and to prevent further bleeding.

Incorrect Answers: A, B, C, and D.

Acid reflux (Choice A) does not independently cause hemorrhage and is not the cause of this patient's hemorrhagic shock. Common
symptoms of acid reflux include retrosternal pain after meals or in the supine position.

Coagulopathy (Choice B) is not the cause of this patient's bleeding, although a baseline coagulopathy is common in patients with
cirrhosis. Even though the international normalized ratio (INR) may be increased in patients with cirrhosis, these patients may be prone ....
- - -- - - - - -- - - - -- - - --- --
r ~, ~ ~ r,
Previous Next Score Report
https://t.me/USMLENBME2CK Lab Values Calculator Help Pause
Exam Section : Item 41 of 50 National Board of Medical Examiners
■ Mark
. --
E) Varices
··- - .,. . .. . Emergency Medicine Self-Assessment

Correct Answer: E.

Varices secondary to cirrhosis is the most likely cause of this patient's bleeding. Major causes of upper gastrointestinal bleeding (UGIB)
include peptic ulcer disease, Mallory-Weiss tear, gastritis, and Dieulafoy lesion among others, but few etiologies cause hemorrhage as
dramatic and potentially catastrophic as ruptured esophageal varices. This patient has evidence of decompensated cirrhosis with a
distended abdomen and fluid wave indicating ascites. Both ascites and esophageal varices are consequences of portal venous
hypertension, which occurs as a result of obstruction of flow through the sinusoids within the liver. All patients with bleeding esophageal
varices should have two large-bore intravenous catheters inserted for the administration of isotonic crystalloids and packed red blood
cells (pRBCs). This patient continues to have signs of hemorrhagic shock despite adequate fluid resuscitation; thus, he should also
receive pRBCs. Additionally, patients with bleeding varices should receive intravenous antibiotics, such as ceftriaxone, an octreotide
infusion, and a proton pump inhibitor. Initial care should focus on stabilizing the patient to facilitate upper endoscopy, at which point
varices can be ligated for definitive source control and to prevent further bleeding.

Incorrect Answers: A, B, C, and D.

Acid reflux (Choice A) does not independently cause hemorrhage and is not the cause of this patient's hemorrhagic shock. Common
symptoms of acid reflux include retrosternal pain after meals or in the supine position.

Coagulopathy (Choice B) is not the cause of this patient's bleeding, although a baseline coagulopathy is common in patients with
cirrhosis. Even though the international normalized ratio (INR) may be increased in patients with cirrhosis, these patients may be prone
to either abnormal clotting or bleeding.

Esophageal rupture (Choice C) presents with severe retrosternal pain and signs of sepsis, not gastrointestinal bleeding. Esophageal
rupture may occur as a result of trauma, instrumentation during endoscopy, or from violent retching.

He/icobacter pylori infection (Choice D) causes peptic ulcer disease or gastritis, which may cause gastrointestinal bleeding, but it does
not frequently cause life-threatening hemorrhage and is more likely to present with epigastric pain and melena.

Educational Objective: Esophageal varices occur in patients with decompensated cirrhosis as a result of portal venous hypertension.
Ruptured varices can cause profound, life-threatening bleeding and should be treated as a medical emergency. All patients should have
adequate peripheral venous access and receive crystalloids, pRBCs, intravenous antibiotics, and octreotide. After initial resuscitation,
endoscopy can be performed to ligate bleeding varices.

.,.

r ~, ~ ~ r,
Previous Next Score Report
https://t.me/USMLENBME2CK Lab Values Calculator Help Pause
Exam Section: Item 42 of 50 National Board of Medical Examiners
■ Mark Emergency Medicine Self-Assessment

42. A 32-year-old woman is brought to the emergency department 2 hours after a drug overdose. Em pty bottles of diazepam and acetaminophen with
codeine were found in her room . She is somnolent. Examination shows a decreased gag reflex and decreased deep tendon reflexes bi laterally.
Which of the following is the most effective method to prevent pulmonary com plications in th is patient?

A) Administration of prophylactic antibiotics


B) Continuous observation
C) Endotracheal intubation
D) Placement of a large-bore nasogastric tube
E) Placement of the patient on her side
Correct Answer: C.

Initial examination of a patient in distress should include an evaluation of the airway, breathing, and circulation. The most common
reasons for urgent endotracheal intubation are an inability to protect the airway, apnea, hypoxemia, and hypercarbia, although other
reasons also exist. These include dynamic airway changes in the setting of neck trauma or hematoma, inhalational injuries, and
anaphylaxis or angioedema. This patient is lethargic with absent protective airway reflexes (eg, gag reflex), which puts her at an
increased risk for aspiration. Aspiration occurs when gastric or oropharyngeal contents enter the airway, which can lead to pneumonitis
or pneumonia. Aspiration pneumonia presents with cough, shortness of breath, pleuritic chest pain, fever, and abnormal breath sounds.
Chest x-ray will exhibit patchy infiltrates, most commonly in the right lower lobe if the patient was upright while the aspiration occurred,
or the posterior, gravity-dependent portions of the lungs if the patient was supine. Therefore, in order to prevent this patient from
aspirating and developing consequent aspiration pneumonia, rapid sequence endotracheal intubation should occur.

Incorrect Answers: A, B, D, and E.

Administration of prophylactic antibiotics (Choice A) for aspiration pneumonia may be considered in this patient once she is clinically
stable, although the evidence for prophylactic antibiotics in aspiration is mixed. Additionally, endotracheal intubation will be more
beneficial in preventing aspiration before it occurs.

Continuous observation (Choice B) is inappropriate, as this patient is unable to protect her airway and is thus at risk for significant
aspiration. Urgent endotracheal intubation should occur as soon as possible.

Placement of a large-bore nasogastric tube (Choice D) may be warranted in this patient as a way of removing the contents of her
stomach. However, placing a large-bore tube through the gastroesophageal junction in an obtunded patient without a secure airway
greatly increases the risk for aspiration. Therefore, endotracheal intubation should occur first.

r
.
~,
- - - -- - - - -- -- -- -- -- - -- - - -- -- - --- ---- - - - -- - - - --- - --
~
- - -- --
~
- - -- -- -
r,
- - - -
....

Previous Next Score Report


https://t.me/USMLENBME2CK Lab Values Calculator Help Pause
Exam Section: Item 42 of 50 National Board of Medical Examiners
■ Mark Emergency Medicine Self-Assessment
...
D) Placement of a large-bore nasogastric tube
E) Placement of the patient on her side
Correct Answer: C.

Initial examination of a patient in distress should include an evaluation of the airway, breathing, and circulation. The most common
reasons for urgent endotracheal intubation are an inability to protect the airway, apnea, hypoxemia, and hypercarbia, although other
reasons also exist. These include dynamic airway changes in the setting of neck trauma or hematoma, inhalational injuries, and
anaphylaxis or angioedema. This patient is lethargic with absent protective airway reflexes (eg, gag reflex), which puts her at an
increased risk for aspiration. Aspiration occurs when gastric or oropharyngeal contents enter the airway, which can lead to pneumonitis
or pneumonia. Aspiration pneumonia presents with cough, shortness of breath, pleuritic chest pain, fever, and abnormal breath sounds.
Chest x-ray will exhibit patchy infiltrates, most commonly in the right lower lobe if the patient was upright while the aspiration occurred,
or the posterior, gravity-dependent portions of the lungs if the patient was supine. Therefore, in order to prevent this patient from
aspirating and developing consequent aspiration pneumonia, rapid sequence endotracheal intubation should occur.

Incorrect Answers: A, B, D, and E.

Administration of prophylactic antibiotics (Choice A) for aspiration pneumonia may be considered in this patient once she is clinically
stable, although the evidence for prophylactic antibiotics in aspiration is mixed. Additionally, endotracheal intubation will be more
beneficial in preventing aspiration before it occurs.

Continuous observation (Choice B) is inappropriate, as this patient is unable to protect her airway and is thus at risk for significant
aspiration. Urgent endotracheal intubation should occur as soon as possible.

Placement of a large-bore nasogastric tube (Choice D) may be warranted in this patient as a way of removing the contents of her
stomach. However, placing a large-bore tube through the gastroesophageal junction in an obtunded patient without a secure airway
greatly increases the risk for aspiration. Therefore, endotracheal intubation should occur first.

Placement of the patient on her side (Choice E) may decrease the amount of aspiration if the patient is vomiting. However, it is a
temporary solution and does not protect the airway from the aspiration of smaller oropharyngeal contents to the degree that
endotracheal intubation does.

Educational Objective: Urgent endotracheal intubation should occur in patients that are apneic, hypoxic, hypercapnic, or are unable to
protect their airway (eg, lethargic, no gag or cough reflex). Endotracheal intubation in patients that are unable to protect their airway
decreases the risk for aspiration and consequent aspiration pneumonitis or pneumonia.

.,.

r ~, ~ ~ r,
Previous Next Score Report
https://t.me/USMLENBME2CK Lab Values Calculator Help Pause
Exam Section: Item 43 of 50 National Board of Medical Examiners
■ Mark Emergency Medicine Self-Assessment

43. A 38-year-old woman is brought to the emergency department by her husband after she had a syncopal episode at home 30 minutes ago. On
arrival, the patient is doubled over in pain. She says that she had abdominal pain about an hour before she fainted, and that it has been
continuous for the past 30 minutes. She says that she has never been pregnant, has been healthy, and is not taking any medications. Her
temperature is 37.3°C (99.2°F), pulse is 120/min, respirations are 22/min, and blood pressure is 80/40 mm Hg. Which of the following is the most
appropriate first step?

A) Acute abdominal series


B) ECG
C) Intravenous fluid resuscitation with 0.9% saline
D) Ultrasonography of the pelvis
E) Urine pregnancy test
Correct Answer: C.

Intravenous fluid resuscitation with 0.9% saline is the most appropriate first step for this patient with hypotension who recently suffered
a syncopal event. The cause of syncope is variable and may occur as a result of reflex or neural-mediated causes, ventricular
arrhythmias, or orthostasis. While syncope is often benign, it is important to obtain an ECG to evaluate for cardiac causes of syncope,
such as ventricular arrhythmias or structural heart disease including hypertrophic cardiomyopathy, but this should only be done after
appropriate fluid resuscitation has been started. In patients with hypotension, such as this patient, attempts should be made to restore
intravascular volume with the intravenous administration of isotonic crystalloids. While this patient's underlying condition causing
abdominal pain remains undifferentiated, she should receive fluid resuscitation while undergoing a workup for her abdominal pain.
Additional diagnostic studies should follow the correction of her vital sign abnormalities.

Incorrect Answers: A, B, D, and E.

Acute abdominal series (Choice A) may ultimately be utilized to further delineate the cause of this patient's abdominal pain, but it
should follow appropriate fluid resuscitation.

ECG (Choice B) should always be obtained in patients with syncope to rule out arrhythmogenic causes of syncope, but this patient's
hypotension should be corrected first.

Ultrasonography of the pelvis (Choice D) and urine pregnancy test (Choice E) may both be necessary to further differentiate the cause
of this patient's pain, but they should occur after appropriate fluid resuscitation.

Educational Objective: Syncope may be neural-mediated (eg, vasovagal syncope) but may also be secondary to orthostasis from ....
-- - - - -- - - - - - - - - -- - - - - ----- - - - -- -- - - - - --- - ----- --- -- - - - - - -- - - - - -- -- - -
r ~, ~ ~ r,
Previous Next Score Report
https://t.me/USMLENBME2CK Lab Values Calculator Help Pause
Exam Section: Item 43 of 50 National Board of Medical Examiners
■ Mark Emergency Medicine Self-Assessment
...
A) Acute abdomi nal series
B) ECG
C) Intravenous fluid resuscitation with 0.9% saline
D) Ultrasonography of the pelvis
E) Urine pregnancy test
Correct Answer: C.

Intravenous fluid resuscitation with 0.9% saline is the most appropriate first step for this patient with hypotension who recently suffered
a syncopal event. The cause of syncope is variable and may occur as a result of reflex or neural-mediated causes, ventricular
arrhythmias, or orthostasis. While syncope is often benign, it is important to obtain an ECG to evaluate for cardiac causes of syncope,
such as ventricular arrhythmias or structural heart disease including hypertrophic cardiomyopathy, but this should only be done after
appropriate fluid resuscitation has been started. In patients with hypotension, such as this patient, attempts should be made to restore
intravascular volume with the intravenous administration of isotonic crystalloids. While this patient's underlying condition causing
abdominal pain remains undifferentiated, she should receive fluid resuscitation while undergoing a workup for her abdominal pain.
Additional diagnostic studies should follow the correction of her vital sign abnormalities.

Incorrect Answers: A, B, D, and E.

Acute abdominal series (Choice A) may ultimately be utilized to further delineate the cause of this patient's abdominal pain, but it
should follow appropriate fluid resuscitation.

ECG (Choice B) should always be obtained in patients with syncope to rule out arrhythmogenic causes of syncope, but this patient's
hypotension should be corrected first.

Ultrasonography of the pelvis (Choice D) and urine pregnancy test (Choice E) may both be necessary to further differentiate the cause
of this patient's pain, but they should occur after appropriate fluid resuscitation.

Educational Objective: Syncope may be neural-mediated (eg, vasovagal syncope) but may also be secondary to orthostasis from
severe volume depletion or cardiac disease, including both structural heart disease and ventricular arrhythmias. In all patients with
syncope that is suspected to be secondary to orthostasis, and especially in those with hypotension, restoration of the intravascular
volume with isotonic crystalloids is paramount.

.,.

r ~, ~ ~ r,
Previous Next Score Report
https://t.me/USMLENBME2CK Lab Values Calculator Help Pause
Exam Section: Item 44 of 50 National Board of Medical Examiners
■ Mark Emergency Medicine Self-Assessment

1\/l

44. A 58-year-old man is brought to the emergency department by paramedics because of a 2-hour history of palpitations and shortness of breath. His
symptoms began suddenly while he was making breakfast and are associated with light-headedness. During the past 15 minutes he has become
diaphoretic and feels more light-headed. He has not had fever, chest pain, or cough. He has hypertension, type 2 diabetes mellitus, coronary artery
disease with prior stenting, and atrial fibrillation. He takes metoprolol, lisinopril, metformin, aspirin, and clopidogrel. He has no known allergies. He
does not drink alcoholic beverages, smoke cigarettes, or use illicit drugs. On arrival, the patient appears diaphoretic and pale. His temperature is
37.1 °C (98.8°F), pulse is 158/min and irreg ular, respirations are 22/min, and blood pressure is 82/46 mm Hg. Pulse oximetry on room air shows an
oxygen saturation of 95%. Lungs are clear to auscultation. Cardiac examination discloses no further abnormalities. There is no jugular venous
distention. The remainder of the examination discloses no abnormalities. ECG is shown. Which of the following is the most appropriate initial
treatment?
A) Amiodarone therapy
B) Defi brillation
C) Diltiazem therapy ....

r ~, ~ ~ r,
Previous Next Score Report
https://t.me/USMLENBME2CK Lab Values Calculator Help Pause
44. A 58-year-old man is brought to the emergency department by paramed ics because of a 2-hour history of palpitations and shortness of breath. His
symptoms began suddenly while he was making breakfast and are associated with light-headedness. During the past 15 minutes he has become
diaphoretic and feels more light-headed. He has not had fever, chest pain, or cough. He has hypertension, type 2 diabetes mellitus, coronary artery
disease with prior stenting, and atrial fibrillation. He takes metoprolol, lisinopril, metformin, aspirin, and clopidogrel. He has no known allergies. He
does not drink alcoholic beverages, smoke cigarettes, or use illicit drugs. On arrival, the patient appears diaphoretic and pale. His temperature is
37.1 °C (98.8°F), pulse is 158/min and irreg ular, respirations are 22/min, and blood pressu re is 82/46 mm Hg. Pulse oximetry on room air shows an
oxygen saturation of 95%. Lungs are clear to auscultation. Cardiac examination discloses no further abnormalities. There is no jugular venous
distention. The remainder of the examination discloses no abnormalities. ECG is shown. Which of the following is the most appropriate initial
treatment?

A) Amiodarone therapy
B) Defibrillation
C) Diltiazem therapy
D) Hepari n therapy
E) Metoprolol therapy
F) Synchronized card ioversion ....

r ~, ~ ~ r,
Previous Next Score Report
https://t.me/USMLENBME2CK Lab Values Calculator Help Pause
Exam Section: Item 44 of 50 National Board of Medical Examiners
■ Mark Emergency Medicine Self-Assessment
...
A) Amiodarone therapy
B) Defi brillation
C) Diltiazem therapy
D) Hepari n therapy
E) Metoprolol therapy
F) Synchronized card ioversion
Correct Answer: F.

The ECG shows an irregu lar, narrow-complex arrhythmia lacking P-waves consistent with atrial fibri llation. Heart failure and atrial
fibri llation share many common risk factors, and each predisposes to the development of the other. Atrial fibrillation with rapid
ventricular response (pulse >100/min) can resu lt in reduced stroke volume and decreased cardiac output leading to symptomatic heart
failure, and potential ly hypotension and hemodynamic instability in severe cases. This patient is presenting with hemodynamically
unstable atrial fibrillation with hypotension and tachycardia. The most appropriate initial treatment is synchronized cardioversion to
quickly correct the arrhythmia and restore adequate cardiac output.

Incorrect Answers: A, B, C, D, and E.

Amiodarone therapy (Choice A), diltiazem therapy (Choice C) , and metoprolol therapy (Choice E) are appropriate therapy options for
hemodynamically stable patients presenting with atrial fibrillation with rapid ventricu lar response. Rate control with diltiazem or
metoprolol is recommended for most patients as opposed to rhythm control with amiodarone becaue of similar efficacy and fewer
adverse effects. Synchronized cardioversion is more appropriate in this hemodynamically unstable patient.

Defibril lation (Choice B) is effective at terminating atrial fibrillation with rapid ventricu lar response but carries a risk for inducing a
ventricular arrhythmia. Synchronized cardioversion is preferred to avoid this risk.

Heparin therapy (Choice D) is not indicated in the acute management of hemodynamical ly unstable atrial fibrillation. Chronic atrial
fibri llation is a risk factor for systemic thromboembolic disease, especially stroke, and patients should be counseled on long-term
antiplatelet and anticoagulation options to reduce this risk.

Educational Objective: Hemodynamically unstable atrial fibrillation with rapid ventricular response requires immediate treatment with
synchronized cardioversion to restore normal sinus rhythm and adequate cardiac output.

.,.

r ~, ~ ~ r,
Previous Next Score Report
https://t.me/USMLENBME2CK Lab Values Calculator Help Pause
Exam Section: Item 45 of 50 National Board of Medical Examiners
■ Mark Emergency Medicine Self-Assessment

45. A 44-year-old woman comes to the emergency department 2 hours after the sudden onset of epigastric pain. The patient says the pain radiates to
her back and improves when she leans forward . She rates the pain as a 9 on 10-point scale. She has vomited twice and still feels nauseated . She
has obesity and cholelithiasis. She does not take any medications and has no known allergies. She drinks two glasses of wine on the weekends
and does not smoke cigarettes or use illicit drugs. BMI is 29 kg/m 2. She appears uncomfortable. Her temperatu re is 37.3°C (99.2°F) , pulse is
96/min, respirations are 18/min, and blood pressure is 138/87 mm Hg. Pulse oximetry on room air shows an oxygen saturation of 98%. Physical
exam ination discloses tenderness to palpation of the epigastrium . The remainder of the examination discloses no abnormalities. Laboratory
studies show:
Leukocyte count 14,000/mm 3(14 x 10 9/L)
Serum
ALT 184 U/L
AST 126 U/L
Alkaline phosphatase 210 U/L
Lipase 368 U/L (N=14-280)

Which of the following is the most appropriate pharmacotherapy at this time?

A) Famotid ine
B) Hydromorphone
C) Methylprednisolone
D) N-acetylcysteine
E) Piperaci llin-tazobactam
Correct Answer: B.

Opioid analgesics, including hydromorphone, are the cornerstone of acute pancreatitis pain management. Acute pancreatitis presents
with epigastric abdominal pain that radiates to the back, along with nausea and emesis, often in a patient with a history of cholelithiasis,
alcoholism, trauma, hypertriglyceridemia, or hypercalcemia. Acute pancreatitis can be complicated by necrosis, hemorrhage, abscess,
or the formation of pseudocysts. Laboratory evaluation often shows increased serum amylase and lipase. Treatment of acute
pancreatitis is largely supportive and should include intravenous fluids, either normal saline or lactated Ringer solution, and bowel rest
in addition to pain control. The associated pain from acute pancreatitis is typically severe and often requires the use of opioid
analgesics.

Incorrect Answers: A, C, D, and E.

- ••·. ·- •• - ! - • - - -·. - • - ••· -· • ·-- - - ••••• • - ·- ·•• - •- - - • • ·- - •• • • • -


r ~, ~ ~ r,
Previous Next Score Report
https://t.me/USMLENBME2CK Lab Values Calculator Help Pause
Exam Section: Item 45 of 50 National Board of Medical Examiners
■ Mark Emergency Medicine Self-Assessment
i - :f. • • •• • • • . A

C) Methylprednisolone
D) N-acetylcysteine
E) Piperaci llin-tazobactam
Correct Answer: B.

Opioid analgesics, including hydromorphone, are the cornerstone of acute pancreatitis pain management. Acute pancreatitis presents
with epigastric abdominal pain that radiates to the back, along with nausea and emesis, often in a patient with a history of cholelithiasis,
alcoholism, trauma, hypertriglyceridemia, or hypercalcemia. Acute pancreatitis can be complicated by necrosis, hemorrhage, abscess,
or the formation of pseudocysts. Laboratory evaluation often shows increased serum amylase and lipase. Treatment of acute
pancreatitis is largely supportive and should include intravenous fluids, either normal saline or lactated Ringer solution, and bowel rest
in addition to pain control. The associated pain from acute pancreatitis is typical ly severe and often requires the use of opioid
analgesics.

Incorrect Answers: A, C, D, and E.

Famotidine (Choice A), an H 2-receptor antagonist, is used in the treatment of gastroesophageal reflux disorder and peptic ulcer
disease. This patient's symptoms, history of cholelithiasis, and increased lipase concentration are more suggestive of acute
pancreatitis.

Methylprednisolone (Choice C), a glucocorticoid, is not indicated in the treatment of acute pancreatitis. It may have a role in therapy for
fulminant liver failure, though this patient's mild increases in liver enzymes does not indicate a failing liver.

N-acetylcysteine (Choice D) is indicated for the treatment of acetaminophen overdose. There are no features of this patient's history
that suggest her current symptoms are induced by acetaminophen. As well, the patient's hepatic transaminases (AST, ALT) are only
mildly increased. In the setting of acetaminophen overdose, transaminase concentrations are often markedly increased, frequently
>1,000 U/L.

Piperacillin-tazobactam (Choice E), a broad-spectrum antibiotic, is not warranted initially in the management of acute pancreatitis.
Acute pancreatitis is caused by severe inflammation, rather than infection, and antibiotics are only necessary if there is secondary
infection of pancreatic necrosis.

Educational Objective: The primary components of acute pancreatitis management are intravenous fluids, either 0.9% saline or lactated
Ringer solution, bowel rest, and pain control. To achieve adequate pain control, opioid analgesics are typically required.

.,.

r ~, ~ ~ r,
Previous Next Score Report
https://t.me/USMLENBME2CK Lab Values Calculator Help Pause
Exam Section: Item 46 of 50 National Board of Medical Examiners
■ Mark Emergency Medicine Self-Assessment

46. A 57-year-old man is brought to the emergency department by his son 30 minutes after he had a tonic-clonic seizure that lasted 1 minute. The
patient is disheveled and has difficulty giving a coherent history. The son reports that his father has a 20-year history of alcoholism and typically
consumed 16 to 32 oz (0.5 to 1 L) of vodka daily. The patient's temperatu re is 37.8°C (100°F), pulse is 110/min, respirations are 20/min, and blood
pressure is 150/70 mm Hg. The skin is slightly jaundiced, and there are spider angiomas over the chest. Abdominal examination shows ascites.
The liver edge is palpated 5 cm below the right costal margin. Neurologic examination shows a diffuse symmetrical tremor. No asterixis is noted .
His Mini-Mental State Examination score cannot be quantified because he does not cooperate. In addition to intravenous fluids, multivitamins, folic
acid, and vitamin B 1 (thiamine), administration of which of the following is the most appropriate next step in management?

A) Atenolol
B) Diazepam
C) Haloperidol
D) Phenytoin
E) Valproic acid
Correct Answer: B.

Alcohol use disorder causes changes in the expression of central nervous system y-aminobutyric acid (GABA) and glutamate receptor
subunits, leading to sympathetic hyperactivity in the setting of alcohol withdrawal. Initially, this hyperexcitability results in tremors,
anxiety, diaphoresis, hypertension, tachycardia, and nausea. Within hou rs of the last drink, alcoholic hallucinosis (auditory or visual
hallucinations without confusion or autonomic instability) and seizures (as a resu lt of cortical hyperactivity) can occur. Delirium tremens,
the most concerning and life-threatening complication of alcohol withdrawal, involves severe confusion and disorientation, fluctuations
in consciousness, agitation, visual hallucinations, and autonomic instability (fluctuations in heart rate and blood pressure with
hyperthermia). As a result of their activating effect on GABA receptors, benzodiazepines, such as diazepam, are effective in reducing
the psychomotor hyperactivity associated with alcohol withdrawal. Long-acting benzodiazepines are preferred because they reduce the
risk for recurrent seizures. Many centers employ a symptom-based approach to the treatment of alcohol withdrawal using clinical
assessment tools such as the Clinical Institute Withdrawal Assessment for Alcohol, revised (CIWA-Ar) scale .

Incorrect Answers: A, C, D, and E.

Atenolol (Choice A) should not be used to treat alcohol withdrawal, as its efficacy for reducing seizures is less well-characterized than
that of benzodiazepines. It may also mask the signs of hemodynamic instability that are associated with delirium tremens.

Haloperidol (Choice C) is contraindicated for most patients with alcohol withdrawal, as it lowers the threshold for seizures.

Treatment with anticonvulsants, such as phenytoin (Choice D) or valproic acid (Choice E), is not typically necessary for seizures
r ~,
----- - -- - - ---- -- - - --- ---- -- --- --- -- --- -- -- -- - -- --
~
-- ·--
~
-- - -- -- - -- -
r,
- - ----- -
....

Previous Next Score Report


https://t.me/USMLENBME2CK Lab Values Calculator Help Pause
Exam Section: Item 46 of 50 National Board of Medical Examiners
■ Mark Emergency Medicine Self-Assessment
...
A) Atenolol
B) Diazepam
C) Haloperidol
D) Phenytoin
E) Valproic acid
Correct Answer: B.

Alcohol use disorder causes changes in the expression of central nervous system y-aminobutyric acid (GABA) and glutamate receptor
subunits, leading to sympathetic hyperactivity in the setting of alcohol withdrawal. Initially, this hyperexcitability resu lts in tremors,
anxiety, diaphoresis, hypertension, tachycardia, and nausea. Within hours of the last drink, alcoholic hallucinosis (auditory or visual
hallucinations without confusion or autonomic instability) and seizures (as a result of cortical hyperactivity) can occur. Delirium tremens,
the most concerning and life-threatening complication of alcohol withdrawal, involves severe confusion and disorientation, fluctuations
in consciousness, agitation, visual hallucinations, and autonomic instability (fluctuations in heart rate and blood pressure with
hyperthermia). As a result of their activating effect on GABA receptors, benzodiazepines, such as diazepam, are effective in reducing
the psychomotor hyperactivity associated with alcohol withdrawal. Long-acting benzodiazepines are preferred because they reduce the
risk for recurrent seizures. Many centers employ a symptom-based approach to the treatment of alcohol withdrawal using clinical
assessment tools such as the Clinical Institute Withdrawal Assessment for Alcohol, revised (CIWA-Ar) scale .

Incorrect Answers: A, C, D, and E.

Atenolol (Choice A) shou ld not be used to treat alcohol withdrawal, as its efficacy for reducing seizures is less well-characterized than
that of benzodiazepines. It may also mask the signs of hemodynamic instability that are associated with delirium tremens.

Haloperidol (Choice C) is contraindicated for most patients with alcohol withdrawal, as it lowers the threshold for seizures.

Treatment with anticonvulsants, such as phenytoin (Choice D) or valproic acid (Choice E), is not typically necessary for seizures
associated with alcohol withdrawal and does not address the underlying cause of the seizures. Benzodiazepine or barbiturate doses, in
escalation, are most appropriate.

Educational Objective: Alcohol withdrawal resu lts in psychomotor hyperactivity secondary to the abrupt cessation of alcohol's effects on
the GABA receptor. As a resu lt of their activating effect on GABA receptors, benzodiazepines, such as diazepam, are effective in
reducing the effects of alcohol withdrawal.

.,.

r ~, ~ ~ r,
Previous Next Score Report
https://t.me/USMLENBME2CK Lab Values Calculator Help Pause
Exam Section : Item 47 of 50 National Board of Medical Examiners
■ Mark Emergency Medicine Self-Assessment
...

N N N
(

JN
. (

..... , ..... ,...... .. . . ... .. .. . ,.. ., ,. . . . .. ... . . .. . .. . . . .


• - ., • •••- • • •• ••• • • • • • • • • e- • ' •••• • • i. • •1 0 1 I • •
.. . . .. .... . . .
•• •• • •••• •• • • •• , t
.. .
1 1•• •• • •
.. •
.
••
.
• •

:;!! ;~!::::•;::;•;::;;I;:~; =~:;-~I:-~ o:; •::: !: o:::: •! o•!, ,:: ::••;r. II;::.; I:aII.:• 11 •:II• I:o• o:: ••: , : o;; :: I;:: :••• f:.; t;: I••:,•• I:.;•~!: •~, ::••IL. . :,:. o, ,: oooI: II,:II• •••••:,•••; •: ••:., ••:
•• • • • • • • • • • •
.
• • •••
.
I •

;:::!::::;:~~:::•:::•: ::!:,;;::::;; •: !:•• :• •;;.,:•,t ;::•: • ;:•••: •• =::: :,::: : ;,•; : : ,: : : :•.,::::: :, ,:: :•~:•• :: ~•:• •: • • ;
f .: ; :

,;:,:•,~: •:: :
t t

! : ; I

::~i;:; : :; ;:·:[.:
.,. t • • • I I I t • J l • , , , • • • • • , , • , , • o o • , o • • , , • , , I • • • •

:\j;l!\il1[]j Iii! ~
!~;;!!1~i!1!l'.1~:;'. ;1mi;i1l::ill:; ~!i!rn: [:!1 :;:]!!;; I;!:1;l[lli'.;l:'. i;[Ii:!;;\;1:::;L]~~::~; ;;; \!i
:i·. :·:· :: .. i::·:r::i:: ::i: ::
;; ;\ / ;f ?\l:/ \\ill··+i\{f =( :::l);:;l\F;\~il;;.;1;<;!).:::; ~ .. :\-1;::.].< :;::! ·. . (}/·: i. : .: . . . ~'. i .:\ ..;:::=
r : : • :- ! . . • . ..
! . -: : : : : . : ' .. .. :, '

47. A 60-year-old man is brought to the emergency department 30 minutes after a syncopal episode. The patient experienced palpitations, weakness,
and dizziness, and then collapsed and lost consciousness for approximately 1 minute. He reports a similar episode 1 week ago for which he did not
seek care. The patient has congestive heart failure and hypertension treated with furosem ide and atenolol. On arrival, his temperature is 37°C
(98.6°F), pulse is 70/min, respirations are 18/min, and blood pressure is 120/70 mm Hg. Pulse oximetry on room air shows an oxygen saturation of
98%. Auscu ltation of the chest discloses clear lungs and no murmurs. During the examination he experiences another episode of palpitations. A
rhythm strip is shown. The episode passes and he reports feeling "normal" after resolution of the palpitations. Vital signs and physical exam ination
remain unchanged after the episode. Adm inistration of which of the following is the most appropriate next step in management?

A) Amiodarone
B) Atropine
C) Calcium gluconate
D) Epinephrine
E) Lidocaine
F) Magnesium ....

r ~, ~ ~ r,
Previous Next Score Report
https://t.me/USMLENBME2CK Lab Values Calculator Help Pause
Exam Section : Item 47 of 50 National Board of Medical Examiners
■ Mark ___ Emergency Medicine Self-Assessment

Correct Answer: F.

The ECG shows torsades de pointes , a dangerous form of polymorphic ventricular tachycardia characterized by a sinusoidal pattern
with cyclic variation in amplitude. Episodes are typically brief and resolve spontaneously, but patients are at risk for progressing to
ventricular fibrillation , hemodynamic collapse, and death. Patients with prolongation of the Q-T interval (which may be familial or
acquired, such as with adverse effects of several medications) ,are susceptible to developing torsades de pointes. Medications that are
associated with torsades de pointes include class IA and Ill antij arrhythmic agents, macrolide and fluoroquinolone antibiotics,
methadone, antipsychotics, antidepressants, antiemetics, and diuretics such as furosemide. Hypomagnesemia, hypokalemia, and
hypocalcemia also increase risk. The patient presentation typically involves palpitations, syncope , lightheadedness, tachycardia , and
pallor. Immediate treatment requires intravenous magnesium administration , which inhibits calcium influx into myocardial cells and
suppresses early afterdepolarizations to interrupt the arrhythmia. Any potential QT-prolonging medications should be discontinued, and
consultation with a cardiologist or electrophysiologist is recommended to determine if long-term pharmacologic therapy or an
implantable cardiac device is indicated. Hemodynamically unstable patients generally require cardioversion (which may not be possible
given the polymorphic nature of the arrhythmia) or defibrillation .

Incorrect Answers : A , B, C, D, and E.

Amiodarone (Choice A) is an antiarrhythmic agent that is commonly used to treat ventricular arrhythmias ; however, it causes QT-
prolongation and can worsen torsades de pointes. Amiodarone should be avoided in this patient.

Atropine (Choice B) is an anticholinergic agent used in the treatment of bradyarrhythmias because of its muscarinic inhibition. Atropine
increases heart rate and shortens the Q-T interval but is not recommended in the treatment of torsades de pointes. Magnesium
administration is standard therapy.

Calcium gluconate (Choice C) may be considered in addition to magnesium in the treatment of torsades de pointes , as hypocalcemia
increases the risk for the arrhythmia as well. Magnesium should be administered first.

Epinephrine (Choice D) is used in the treatment of numerous cardiac arrhythmias as a result of its positive inotropic and chronotropic
effects. Increasing the heart rate shortens the Q-T interval and may provide a benefit; however, magnesium has been demonstrated as
the most effective initial therapy.

Lidocaine (Choice E) may be considered as an adjunct to magnesium for persistent torsades de pointes as it does not prolong the Q-T
interval. Magnesium should be administered first.

Educational Objective: The initial treatment of torsades de pointes is intravenous magnesium therapy, which inhibits calcium influx into
myocardial cells and decreases the frequency of early afterdepolarizations that propagate the arrhythmia.

r ~, ~ F' r,
Previous Next Score Report
https://t.me/USMLENBME2CK Lab Values Calculator Help Pause
Exam Section: Item 48 of 50 National Board of Medical Examiners
■ Mark Emergency Medicine Self-Assessment

.. . ... - .
l • • .•,
~4

~ . ffiiiE
. ,, •
~ t
,.

f1l4f
._,._. ,..
-
.._ ,. ~ r -.-a-

.• IU
~

-., -
,4 -,i ,._ ..........
......
~

i r: -
"" "
-• If i,.

~-
--
t t ,~ --
' ~ . ,1.a,i

.. "
'

48. A 62-year-old woman is brought to the emergency department because of a 2-hour history of severe chest pain. She has hypertension and type 2
diabetes mellitus . Her medications are insulin, lisinopri l, and hydrochlorothiazide. During the triage process, she suddenly collapses. The patient is
apneic and pulseless. A rhythm strip is shown. After initiating high-quality cardiopulmonary resuscitation, which of the following is the most
appropriate next step in management?
A) Administration of amiodarone
B) Administration of epinephrine
C) Administration of vasopressin
D) Defibrillation
E) Intubation and mechanical ventilation
Correct Answer: D.

The patient is presenting with cardiac arrest, and cardiopulmonary resuscitation (CPR) has been initiated appropriately. The most
important factors that improve the chance of survival from cardiac arrest are early activation of the emergency response system (this
patient was already present in the emergency department), immediate initiation of high-quality chest compressions with minimal
interruptions, early defibrillation (for shockable rhythms), and early initiation of advanced cardiac life support (ACLS) protocols. The
rhythm strip shows ventricular fibrillation, and the most appropriate next step in management is defibrillation. Defibrillation should be
administered to patients presenting without a pulse that exhibit either ventricular fibril lation or ventricular tachycardia on the rhythm
strip. Pulseless electrical activity (PEA) and asystole do not respond to defibrillation. Delay in initiating chest compressions and in
administering the first defibrillation shock in the setting of a shockable rhythm are associated with a decreased likelihood of achieving
return of spontaneous circulation.

Incorrect Answers: A, B, C, and E.

Administration of amiodarone (Choice A) and epinephrine (Choice B) are recommended in the ACLS protocol after the initiation of high-

r ~ -- -··- -
- , -- -- -- - -- - -- -- . ... - ·-·· -- - -····-··-- - -- - - --r
~ ~
--- ,-- -- ....

Previous Next Score Report


https://t.me/USMLENBME2CK Lab Values Calculator Help Pause
Exam Section : Item 48 of 50 National Board of Medical Examiners
■ Mark Emergency Medicine Self-Assessment
i .. • . . .. • • - • - • . -

C) Administration of vasopressin
D) Defibrillation
E) Intubation and mechanical ventilation
Correct Answer: D.

The patient is presenting with cardiac arrest, and cardiopulmonary resuscitation (CPR) has been initiated appropriately. The most
important factors that improve the chance of survival from cardiac arrest are early activation of the emergency response system (this
patient was already present in the emergency department), immediate initiation of high-quality chest compressions with minimal
interruptions, early defibrillation (for shockable rhythms), and early initiation of advanced cardiac life support (ACLS) protocols. The
rhythm strip shows ventricular fibrillation , and the most appropriate next step in management is defibrillation. Defibrillation should be
administered to patients presenting without a pulse that exhibit either ventricular fibrillation or ventricular tachycardia on the rhythm
strip. Pulseless electrical activity (PEA) and asystole do not respond to defibrillation. Delay in initiating chest compressions and in
administering the first defibrillation shock in the setting of a shockable rhythm are associated with a decreased likelihood of achieving
return of spontaneous circulation .

Incorrect Answers : A , B, C, and E.

Administration of amiodarone (Choice A) and epinephrine (Choice B) are recommended in the ACLS protocol after the initiation of high-
quality chest compressions and defibrillation for shockable rhythms. Epinephrine is recommended for shockable and non-shockable
rhythms. Amiodarone may be used for ventricular tachycardia or ventricular fibrillation .

Administration of vasopressin (Choice C) may be considered if epinephrine is not available. Chest compressions and early defibrillation
are more effective than pharmacologic interventions and should be performed first.

Intubation and mechanical ventilation (Choice E) are recommended in ACLS protocols for securing the airway and ensuring adequate
oxygenation and ventilation . Maintaining perfusion pressure and attempting to restore cardiac function take priority over advanced
airway management in CPR. Basic airway and breathing management including bag-valve-mask ventilation and oxygenation should be
prioritized.

Educational Objective: The most important factors that promote survival in the setting of cardiac arrest are early activation of the
emergency response system , immediate initiation of high-quality chest compressions with minimal interruptions, early defibrillation (for
shockable rhythms), and early initiation of ACLS protocols.

...
r ~, ~ p r,
Previous Next Score Report
https://t.me/USMLENBME2CK Lab Values Calculator Help Pause
Exam Section: Item 49 of 50 National Board of Medical Examiners
■ Mark Emergency Medicine Self-Assessment

49. A 61-year-old African American man with previously stable angina pectoris comes to the emergency department because of a 2-day history of
worsening chest pain. The pain occurs at rest or with minimal exertion. He rates the pain as an 8 on a 10-point scale. Nitroglycerin has provided
only moderate relief. He has not had diaphoresis, nausea, or dyspnea. He has coronary artery disease, type 2 diabetes mellitus, and hypertension.
He has no history of liver disease or bleeding problems. In addition to nitroglycerin, medications include glyburide, valsartan, and
hydrochlorothiazide. He is 178 cm (5 ft 10 in) tal l and weighs 85 kg (188 lb); BM I is 27 kg/m 2. His temperatu re is 37.3°C (99.1 °F), pu lse is 87/min,
respirations are 18/min, and blood pressure is 130/68 mm Hg. Pulse oximetry on room air shows an oxygen saturation of 93%. Physical
exam ination discloses no abnormalities. A chest x-ray shows minimal cardiomegaly. An ECG shows T-wave inversions in the anterior leads not
seen on previous ECGs. Serum cardiac enzyme concentrations are within the reference ranges. The patient is given 3 L of oxygen via nasal
cannula. Morphine (4 mg) is administered, and the patient is pain-free. In addition to admin istration of aspirin, which of the following is the most
appropriate next step?

A) Administration of heparin
B) Administration of recombi nant tissue plasminogen activator
C) Intra-aortic balloon counterpulsation
D) Referral to a cardiac surgeon
E) Ventilation-perfusion lung scans
Correct Answer: A.

Administration of heparin is the next most appropriate step. This 61-year-old patient with a history of stable angina but with new onset
chest pain at rest and T-wave inversions in the anterior leads most likely has unstable angina. There are three primary categories of
acute coronary syndrome (ACS): unstable angina (UA), non-ST-elevation myocardial infarction (NSTEMI), and ST-elevation myocardial
infarction (STEMI). A STEMI presents with chest pain, ST elevations on ECG, and positive cardiac biomarkers (eg, troponin). Treatment
is with thrombolytics or immediate cardiac catheterization. UA and NSTEMI are similar conditions and present with ECG changes such
as T-wave inversions or ST segment depressions and chest pain but are differentiated by the presence (NSTEMI) or absence (UA) of
increased cardiac biomarkers. Regardless, UA and NSTEMI are treated similarly with dual antiplatelet therapy and full dose
anticoagulation. Appropriate anticoagulant choices include either unfractionated heparin or low molecular weight heparin. Patients are
then typically risk stratified using various scoring models to determine the urgency of cardiac catheterization, but most will undergo
catheterization within 24 hours.

Incorrect Answers: B, C, D, and E.

Administration of recombinant tissue plasminogen activator (Choice B) is appropriate for patients with a STEMI who do not have
immediate access to a cardiac catheterization suite. Thrombolytics are not given for UA or NSTEMI.

r ~,
- - -- - - -- - - - -- - - - -- - - - - -- - --- -- -- -- -- - -- - - - - - -- -
- -
~ ~
- -- - - - - - - -- --
r,
---- -- - - . ....

Previous Next Score Report


https://t.me/USMLENBME2CK Lab Values Calculator Help Pause
Exam Section: Item 49 of 50 National Board of Medical Examiners
■ Mark Emergency Medicine Self-Assessment
• - • • • I • • I • I I• I

E) Ventilation-perfusion lung scans


Correct Answer: A.

Administration of heparin is the next most appropriate step. This 61-year-old patient with a history of stable angina but with new onset
chest pain at rest and T-wave inversions in the anterior leads most likely has unstable angina. There are three primary categories of
acute coronary syndrome (ACS): unstable angina (UA), non-ST-elevation myocardial infarction (NSTEMI), and ST-elevation myocardial
infarction (STEMI). A STEMI presents with chest pain, ST elevations on ECG, and positive cardiac biomarkers (eg, troponin). Treatment
is with thrombolytics or immediate cardiac catheterization. UA and NSTEMI are similar conditions and present with ECG changes such
as T-wave inversions or ST segment depressions and chest pain but are differentiated by the presence (NSTEMI) or absence (UA) of
increased cardiac biomarkers. Regardless, UA and NSTEMI are treated similarly with dual antiplatelet therapy and full dose
anticoagulation. Appropriate anticoagulant choices include either unfractionated heparin or low molecular weight heparin. Patients are
then typically risk stratified using various scoring models to determine the urgency of cardiac catheterization, but most will undergo
catheterization within 24 hours.

Incorrect Answers: B, C, D, and E.

Administration of recombinant tissue plasminogen activator (Choice B) is appropriate for patients with a STEMI who do not have
immediate access to a cardiac catheterization suite. Thrombolytics are not given for UA or NSTEMI.

Intra-aortic balloon counterpu lsation (Choice C) is used as an adjunctive measure in patients with cardiogenic shock secondary to ACS.
This patient does not have evidence of cardiogenic shock.

Referral to a cardiac surgeon (Choice D) is not appropriate at this time. The patient should first be treated with anticoagu lants and
antiplatelets fol lowed by cardiac catheterization. In patients with multivessel coronary disease not amenable to cardiac stent placement,
coronary artery bypass graft surgery is an alternative option.

Ventilation-perfusion lung scan (Choice E) is one imaging modality used to diagnose a pulmonary embolism (PE). While this patient's
symptoms could be secondary to a PE, his known cardiac disease, worsening chest pain, and new ECG changes are more consistent
with unstable angina.

Educational Objective: Unstable angina is one type of acute coronary syndrome and is typified by new onset chest pain in conjunction
with ECG changes, but with normal cardiac biomarkers. Both UA and NSTEMI are treated simi larly with fu ll dose anticoagulation and
dual antiplatelet therapy followed by cardiac catheterization.

.,.

r ~, ~ ~ r,
Previous Next Score Report
https://t.me/USMLENBME2CK Lab Values Calculator Help Pause
Exam Section: Item 50 of 50 National Board of Medical Examiners
■ Mark Emergency Medicine Self-Assessment
...

I~ , . A A - - . , J ~ ~ / ' , , ~ ~ ~ \ N,.N~,,--A ~.....-V...,


II

I~~~~
50. A 75-year-old man is brought to the emergency department because of a 3-day history of progressive fatigue and generalized weakness. He says
that th is morning, he felt too weak to get out of bed and had shortness of breath at rest. He has type 2 diabetes mellitus and hypertension. His
medications are glipizide and amlodipine. He has no known allergies. The patient is awake and in mild distress. He is able to follow commands.
BMI is 21 kg/m 2. His temperature is 37°C (98 .6°F), pulse is 156/m in and irreg ular, respirations are 22/min, and blood pressu re is 82/48 mm Hg.
Pulse oximetry on room air shows an oxygen saturation of 94%. Auscultation of the lungs discloses bilateral basi lar crackles. The remainder of the
examination shows no abnormalities. An ECG is shown. Which of the following is the most appropriate next step in management?
A) Administration of diltiazem
B) Administration of 0.9% saline
C) Echocardiography
D) Electrical card ioversion
....

r ~, ~ ~ r,
Previous Next Score Report
https://t.me/USMLENBME2CK Lab Values Calculator Help Pause
I~,.AA--.,J~~A,L,~1,,,___,v.,\.."•'J,. '--~~\N,.N~_,,, { ~.....-V...,
II
1

I~~~~
50. A 75-year-old man is brought to the emergency department because of a 3-day history of progressive fatigue and generalized weakness. He says
that th is morning, he felt too weak to get out of bed and had shortness of breath at rest. He has type 2 diabetes mellitus and hypertension. His
medications are glipizide and amlodipine. He has no known allergies. The patient is awake and in mild distress. He is able to follow commands.
BMI is 21 kg/m 2. His temperature is 37°C (98 .6°F), pulse is 156/m in and irreg ular, respirations are 22/min, and blood pressu re is 82/48 mm Hg.
Pulse oximetry on room air shows an oxygen saturation of 94%. Auscultation of the lungs discloses bilateral basi lar crackles. The remainder of the
examination shows no abnormalities. An ECG is shown. Which of the following is the most appropriate next step in management?
A) Administration of diltiazem
B) Administration of 0.9% saline
C) Echocardiography
D) Electrical card ioversion
E) X-ray of the chest ....

r ~, ~ ~ r,
Previous Next Score Report
https://t.me/USMLENBME2CK Lab Values Calculator Help Pause
Exam Section: Item 50 of 50 National Board of Medical Examiners
■ Mark Emergency Medicine Self-Assessment
I - • - '• •• • •" • • , ■- •I• • • - • I - ■ I • • • - I I I • - I• - • ■- • • . - - - • e- • -
examination shows no abnormalities. An ECG is shown. Which of the following is the most appropriate next step in management?
A) Administration of diltiazem
B) Administration of 0.9% saline
C) Echocardiography
D) Electrical cardioversion
E) X-ray of the chest
Correct Answer: D.

The ECG shows an irregular, narrow-complex arrhythmia lacking P-waves consistent with atrial fibrillation. Heart failure and atrial
fibrillation share many common risk factors, and each predisposes to development of the other. Atrial fibrillation with rapid ventricular
response (pulse >100/min) can result in reduced stroke volume and decreased cardiac output leading to symptomatic heart failure, and
potentially hypotension and hemodynamic instability in severe cases. This patient is presenting with hemodynamically unstable atrial
fibrillation with hypotension and tachycardia. The most appropriate initial treatment is synchronized cardioversion to quickly correct the
arrhythmia and restore adequate cardiac output.

Incorrect Answers: A, B, C, and E.

Administration of diltiazem (Choice A) would be appropriate for a hemodynamically stable patient presenting with atrial fibrillation with
rapid ventricular response. The presence of hypotension makes electrical cardioversion the more appropriate initial therapy for this
patient.

Administration of 0.9% saline (Choice B) is not appropriate at this time and may worsen the patient's tachyarrhythmia-induced heart
failure, as he has evidence of pulmonary edema on physical examination with pulmonary crackles. Intravenous fluids may be
considered if the patient remains hypotensive after the restoration of normal sinus rhythm.

Echocardiography (Choice C) and x-ray of the chest (Choice E) should be included in the eventual diagnostic work-up to investigate the
cause of the patient's arrhythmia, but these studies should not delay prompt correction of the arrhythmia with synchronized
cardioversion in this hemodynamically unstable patient.

Educational Objective: Hemodynamically unstable atrial fibrillation with rapid ventricular response requires immediate treatment with
synchronized cardioversion to restore normal sinus rhythm and adequate cardiac output.

.,.

r ~, ~ ~ r,
Previous Next Score Report
https://t.me/USMLENBME2CK Lab Values Calculator Help Pause

You might also like